Vous êtes sur la page 1sur 171

Dr.

Gopika Sreedhar 18-06-2018

Hai all, these are the latest question papers


which I referred. Please confirm the answers
given along with the questions before referring,
I didn’t make any corrections instead just copied
the papers and some don’t have answers.

This is a small contribution to all my fellow


doctors within my little knowledge.

Once again, a big thank you to all those who


supported me and made my exam a success.
Best of luck to all those who are heading for
exams.
GOD BLESS….
Dr. Gopika Sreedhar
(my MOH exam date: 10/06/2018)

- - Questions almost what I remember - -

1. Patient upper anterior x-ray showing a notch in inter dental


born between two central incisors, it's due to

B. Eruption cyst

C. Supernumerory tooth

2. Indirect retainer
mostly needed in A.
Class4

B.
Class 1
C.
Class 3

D. Class 3 with modification 1

3. Child came to the clinic with his nanny and u notice very
large scar in his forehead, u will

a-Take history f scar from nanny and work

b-Take history from child and work

c-Call parent to take history and work

d-No work without parent

4. Ph of
caoh A- 6

B-8
C-
12.5
D-9
5. Principle of GTR

A. Space creation and wound stabilisation

b. Surface biomodification and wound stabilisation


C. Bone remodeling and intranarrow penetration

6. Mature tooth with deep caries with pain


A. RCT

B. Pulpotomy
C. Pulp capping

7. Inflamed pulp and we remove


3mm pulp pathology A. Partial
Pulpotomy***
B. Pulpotomy

C. Direct pulp
capping D.
Indirect pulp
capping
8. Inflamed pulp with periapical lesion

A. Pulpotomy
B. Direct pulp capping

C. Indirect pulp capping


D. Pulpectomy with znoe**

9. Proxy brush used with embrassure


A. Type 1

B.
Type 2
C.
Type 3
D.
Type 4

10. Most common types of occlusicon which is easy to fabricate


A. Mutually protected
B. Unilateral

D. Bilateral balanced occlusion

11. Patient with 5 years old denture has a severe gag reflex,
upon history he says he had the same symptoms in the first few
days of the denture delievery and it went all alone:

a. patient has severe gag reflex.


b. Patient has underlying systemic condition.****
c. Denture is overextended

12. Simpifil type for canal


enlargement by NiTi in: a.Universal
protaper

B.
Reciproca
l c.
Revers S

D. Light speed rotary****

13. What is the copper ratio that


eliminates gamma phase 2: a 2%cooper

b
4%copper
c 10%
copper d
13%
copper

14. Connects haversian system to osteocyte


A. Canaliculi

B. Volkmans canal

C. Interstitial lamelli

15. Which solution can


corrosion tools A. Sodium
hypochlorite
B. Iodophor
16. Gutta percha disinfectant

A. H2O2
B. 5.2% NAOCL

17. In a clinical research trial we primarily need to :


Get written subject in the patient's own native language.
B. verbally write subject briefly
C. tell patients that they are part of study.
18. 51 year female came to u with complete denture 2 years
ago. now suffering from burning sensation u check the
denture there was perfect seal and occlusal rest in a
position ......

the burning sensation from


a. viral infection

B. Menopause***

C. vitamin deficiency

19. Contraindication of epinephrine


to pt. have? A. Diabetic

B.
Hypoparathyroid
ism C.
Hyperparathyro
dism D.
Hyperthyrodism

20. 10 years child with congenital heart disease came for


extraction of his lower 1st molar, the antibiotic of choice for
prevention of infective endocarditis is:
21. A. Ampicillin 30 mg /kg orally 1hour before procedure.
22. B. cephalexin 50mg/kg orally 1hour before procedure
23. C. clindamycin 20mg/kg orally 1hour before procedure.
24. D. Amoxicillin 50mg/kg orally 1hour before procedure.

21. Patient with perio problem lost some of his tooth and have
chances of loosing again, which type of denture to be used
22. A. Interim denture
23. B. Transitional denture **
24. C. Immediate denture

22. A case with picture renal transplantation and patient has


sore throat and burning?
A. Leukoplakia

C. Pseudomembranou
s candidiasis C. Erythe
D. matous candidiasis
23. Characteristics of NUG

A. Non contagious
B. Pseudomembranous
24.

A. Residual cyst

B. Lateral periodontal cyst

C. Dentigrous cyst

25.

A. Picture of hemangioma

26. Picture of submandibular salivary gland

27. Picture of root caries

28. Minimum Crown


root ratio
A. 1:1

B.
1:2
C.
2:3

29. Function of propofol


A. Increase intraocular pressure

B. Decrease intracranial pressure

C. Hyperventilation
30. Common Intracanal medicament
A. Caoh

31. What ib the property of nickel


titanium wirre A. Rigidity

B. Low coefficient of friction


C. Shape memory***

32. 25 Yr patient deep pit and fissures, mild to moderate


caries risk. Which of the following not used for treatment

A. Sealant
B. Composite

C. Amalgam

33. Shoeing in complex amalgam restoration


A 1-1.5

B 2-3
C 0.5-1
D 1.5-2
34. Fluoride concentration in water

A. 1ppm

B. 2ppm
C. 3ppm

35. To avoid fracture of teeth after bicuspidization procedure


the treatment should be??
a. splinting
b. bone with resin

c. extract and
implant D.
crown
36. When you try to seat a crown on tooth you find open gap at
the margin, you will

A. Remake a new Crown***


B. Reduce inner surface of a crown
C. Fill the gap with composite

37. Under GA treatment pedo patient having mesial and distal


carries on 2 adjacent teeth A. Stainless steel Crown
38. Good implant material

A. Titanium
B. Zirconium

C. Co-cr

39. Which techniques will you use to anesthestize soft and


hard tissues of mandibular molars in one injection

A. Akinosi method

B. IANB
C. Gowgates

40. During clinical examination prior to compelete denture


constriction the dentist ask pt to say Ahh he noticed that the left
half of soft palate is not moving the indicate the injury of cranial
nerve

A. Vagus nerve (X)


B. Facial nerve

C. Glossopharyngeal

41. Orthognathic surgery,you plan to use 2 mm screw,the


drill size to make a hole is...? A 1 mm

B 1.5
mm C
2mm D
2.5
mm
42. Ph of enamel

A5.5
B 6.5

C4.5

D7.5
43. Instrument which use for grasping a tissue when remove thick
epulis fissuratum:
a- Allis forceps.

b- Addison forcep.

c- Curved hemostat.

d- Stilli forceps

44. G V black classification what number


represent blade angle A. no 1

B. no
2 C.
no 3
D. no
4
45. Class 2, long face indicated for extraction premolar?

A. when flaring central incisor


B. not indicated

C. for deep bite

46. An old patient had a complete denture, he came to you after


delivery complaining that the lower denture moves from the ridge
when he just moves his tongue, you put a (pip) paste and
checked the denture and there is no any areas of pressure or
over extension no occlusal discrepancy, what is the problem?

A. Under extension of borders


B. Over extension of borders

C. High occlusal plane


D. Cramped tongue

47. Question about vitality test

48. The following Structures opening


into Middle meatus: A. nasolacrimal duct

B. posterior ethmoidal sinus and maxillary sinus *


D. sphenoid sinus

49. Although it is a caries detecting test, synder


test is used primarily to A.amount of saliva
B.number of acid producing microorganisms

C.ph in saliva

D.undetecting caries

50. Tooth present in 10yr old child

51. Complete denture patient tissue over mandibular Ridge is


easily retractable, how to manage
A. Minus surgical procedures to correct
B. Impression will replace tissue

52. Subgingiv
al scaler A.
Gracey curette\
B. Universal
curette

53. canal filling using betta phase condensation gp:


54. A. Thermafill

B. Mc
spadden C.
Obtra II***

D. simplifill

55. Anomalis during initiation and profilration of


tooth germ will lead to: a. Amelogenasis imberfecta.

b. Dentinogenasis
imberfecta. c. Dentinal
dysplasia.

d. Oligodontia.

55. 7yr old child Mesial and distal root fracture while extracting
lower primary second molar,how to manage?
56. A. Visualise and leave
57. B. Visualise and carefully remove
58. C. Remove after 1 week

56. Substantivity is the property to release when required


from the oral structures,is the property of which material

A. flouride
B. chlorhexidine gluconate

57. Main action of


aspirin is:-A. platelet
aggregation

B.
prothrombi
n C.
fibrenogin
D. thromboplastin

58. Not commonly used


in endo:
A-ice

B-electric
pulp test C-
hot water
D-percussion
59. Patency file is:

A. small file used beyond the apex


B. small rotary file
C. small file used with irrigant for recapitulation

60. Before placement of fissure sealant what


should you do? A. clean the surface with
polishing paste****

B. clean with
polishing burs C.
occlusal réduction
61. Gingivectomy indication for all except :
62. A. Extensive alveolar removal
63. B. Eliminate supra bony pocket
64. C. Eliminate periodontal abscess

62. Born exposure diagnosed by ( not remembering full


options, but answered this) A. Cortical root exposure
63. Fracture in which one cortical plate is broken (something
like that):

A. Greenstick fracture

B. Unilateral condyle fracture


C. Bilateral condyle fracture

D. Body fracture

64. Post graduate student use MTA the prognosis


depends on prevent A. Immediate future
B. Disturbance during closure of wound
65. Degree of saturation

A. Chroma

B. Value
C. Hue

66. Missing lateral incisor adjacent tooth is healthy


and non restored A. Implant

B.
FPD
C.
RPD

67. According to new technology finishing and polishing


after amalgam filling A. 24 hrs

B. 6
hrs C.
2 days
D. 2
hrs

68. Young patient comes with subcondylar fracture, during


examination you notice class l occlusion. X-Ray showed gross
condylar displacement. What's the management :
69. occlusion. X-Ray showed gross condylar displacement. What's
the management :
70. A. Open reduction followed by physiotherapy
71. B. Closed reduction followed by physiotherapy ***
72. C. nothing to do

69. Full length


root viewed A.
Parallel
technique***

B. Bisecting technique

70. Xray needed to find correct position of foreign bodies or


impacted tooth in Mouth A. Tube shift techniques**
B. Bisecting angle
C. Paralleling techniques

D. Two films against each other (something like that)

71. You want to make impression to patient and try to control


fluid but bleeding doesn't stop and provisional restoration to be
made, what to do

A. Epinephrine retraction code**

B. Wait for some days

C. Take impression with sodium chloride and polyether

72. Question about remineralisation

73. Blood glucose levels in patients detected by


finger prick method A. 100-200

B. 110-150**
C. 40-100 D. 400-500
74. Name of dental carry system

A. DMF

B. MDF

C. MTD

75. best describe for prepared canal:

A. apical part is the narrowest in cross section**


B. parallel wall terminates at apical collar

C. parallel wall few millimeters apical collar

EXAM -2
1. gates glidden drill
a. for coronal preparation of the canal
b. numbered from 9 to 20
c. for smoothening of line angles
d. gets entangled with dentin in root canal
2. radiograph of dentigerous cyst.. impacted 3rd molar..coronally
attached radiolucency
3. radiograph of ameloblastoma...multilocular at the angle of
mandible
4. pt came to a dentist with a tooth which got trauma
2yrs back,8yrs patient central incisor, xray and radiographs
made the dentist take up pulp revascularization what is the
present condition of pulp
a.open apex vital
b.closed apex
necrosed c.primary
tooth necrosed
d.open apex non
vital**
5. right time for 1st orthodontic screening
a. 6yrs
b. 7yrs
c. 8to 9 yrs
d. 3yrs
6. most crucial phase for stoppage of
thumb sucking habit a. decidious
dentition
b. early
mixed c.
late
mixed
d. pernanent dentition
7. patient with mesial distal and buccal caries on decidio
central incisor. dentist decided metal crown placement. what is
the next step in management
a. selection of metal crown before caries
removal **
b. selection of metal crown after caries
removal
c. does not matter before or after
removal of caries d. none of the
above
8. the fluoride supplement most importantly depends upon
a. temperature of location
b. age**
c. fluoridated water intake
d. form of supplement
9. dentist wants to prevent displacement of restoration of a
short walled prepared cavity. what is the most desired step that
would lead to this
a. placement of retentive pins
b. placement of retentive grooves**
c. retentive undercuts of
internal line angles d. . flaring
of cavosurface margin
10. ph of caoh cement
a. 12
b. 12.5
c. 7
d. 10.5

11. what is the management of the tooth with fraCture of


crown upto the margin of alveolar crest
a. extraction
b. crown lengthening followed by post and core
c. subgingival tooth preparation to create ferrule effect followed
by post and core
d. orthodontic extrusion followed by post and core12. forcep used
for disimpaction of
fractured nasal bone
a . rows disimpaction forecep
b. william hayton forcep
c. walsham forecep
d. adsons forcep
13. which canal have the rarest probability of
finding a second canal
a. mesiobuccal root of maxillary second molar
b. distal root of mandibular
first molar
c. distobuccal root of.max
first molar d. mandibular
central incisor
14. type of cementum in the coronal 2/3rd of the root
a. cellular extrinsic
b. cellular intrinsic
c. acellular extrinsic
d. acellular intrinsic
15. intraoral picture of hiv patient came to clinic for follow up
with complain of burning sensation in the mouth
showing white coloured lesions on the uvula
and soft palate.. a. acute herpetic lesions
b. acute pseomembranous
candidiasis c. candida
infected leukoplakia
d. lichen planus
16. max 3rd molar tooth goes into the upper space while
extraction.what should be done a. observe and follow up
b. ct followed by removal
under g.a.**
c. no treatment just inform
the patient
d. explore the same site after 3weeks and remove the tooth
17. blacks instrument formula 8 - 40- 16- 14
what is the length of the blade
a. 8
b.40
c.16
d. 14
18. picture of upper arch..arrows on space between later
incisor and canine.identify
a. leeway space
b. mixed
dentition space c.
space of louis
d. primate space
19. a 35yrs old patient came for a routine checkup. on
radiography a small radiolucency with scalloped margins was
seen periapical to the the tooth. no symptoms ,no pain on
percussion, associated tooth vital.
a. stafnes bone
cyst b.static
bone cyst
c.unicameral
bone cyst d.
simple bone cyst
20. a radioopaque radiolucent lesion was found during
radiographic examination of the patient. clinic examination
reveal no expansion of the cortical plates no pain. the aspirate
was a straw coloured yellow fluid. it was confirmed to be
ossifying fibroma.what is the treatment
a. no treatment
b. wait till the growth ceases followed by smoothing of
expansion
c. enucleation
d. marsupialization
21. an amalgam tooth restoration got fractured at isthmus .
what is the most probable cause
a. improper
trituration b.
high occlusion
c. flaring of the cavosurface
margin
d. over finishing
22. a young patient came with number of restoration and
large number of new caries in the mouth. on examination the
gingiva appeared red and much swollen with a probing depth of
2 to4 mm. what is the peridontal diagnosis of the patient?
a. high caries index
b. gingivitis
c. juvenile
peridontitis d.
chronic
periodontitis
23. what is the most coomon risk factor of fracture of anterior
tooth
a. amelogenesis imperfecta
b. fluorosis
c. proclination
d. dentinogenesis imperfecta
24. why is premolar mostly at high risk of perforation during
the endodontic treatment a. it have more coronally placed
furcation
b. presence of concavity on the immediate coronal
part of root surface *
c. wider canals with less surrounding dentin
d. less mesiodistal diameter
25. fracture case
pt recieves blow on the face causing orbital rim movement
open bite, movement at fronto zygomatic suture, orbital
hyperteleorism, ecchymosis.
type of fracture?
a. le fort 1
b. le fort
c. lefort 3
d. isolated zygomatic fracture

28. patient came up with multiple caries in his mouth..on


examination 5 endodontically treated tooth and multiple areas
of restoration were seen.what is the caries index of the patient
a. moderate caries
risk
b. high caries
risk*
c. mild caries risk
d. periodontal evaluation is necessary
29. what is the maximum safe dose of local anaethetic for a
65 yrs old patient of 80 kgs. with 2% of lignocaine with
1:100000 epinephrine
a. 478
b. i dont remember options ...some in 500 **some in 600 n 300s
30. one simple question on tongue tie when is it treated
a. limited tongue movement
b. tongue touching palate
c. hyper mobility of tongue muscles
d. depressed tongue
31. patient came with rpd providing replacement to missing
lower molars. on examination it was found that it is an incorrect
rpd design with no rest or support. what is this type of rpd
flangeless **
open faced lingual
buccal
32. bur used for making v shaped rest on the cingulum of canine
a. round bur
b. tapered
c. fissure
d. inverted cone**
33. which cement cause maximum
pulpal tissue irritation a. calcium
hydroxide
b. zinc oxide eugenol
cement c. zinc
phosphate cement*
d. zinc
polycarboxylate c
34. child came with tetracycline stains on his central
incisors and molars what is the probable age at which
tetracycline was prescribed to this patient a. 1yr**
b..
2y c.
6yrs
d. 5
yrs
35. niti wire used in ortho ..
reason? something like that =》 i
chose shape memory
36. most common malignant tumour of minor salivary gland=
acinic cell
37. carbohydrate leading to dental caries depends on
a. frequency of intake**
b. amount of "
c. form of "
d. duration
38. compound added in alginate to initiate the
reaction
a. soduim phosphate
b. pottasium
phosphate c.
sodium sulphate
d. calcuim sulphate

39. presence of microbes beneath the pit and fissure


sealant will..
a. arrest the growth of bacteria**
b. protect the growth of
bacteria c. induce
growth of bacteria
d. reduce the growth of surface bacteria
40. cell rich layer
a.contai n mast cell and
fibroblasts b. c fibres
c. mitotic activity...i chose this
41. pt came with a complain of heaviness related to maxillary
premolar area. radiography revealed vital tooth. both the
premolars appear normal but show positive percussion test. your
diagnosis
a. chronic general periodontitis
b. lodgement of extraoral substance in
gingival pocket c. maxillary sinusitis
d. dental space infection
42. after providing adequate inferior alveolar block for
extraction of mandibular 1st molar. patient still complains of
sensation.which nerve will you consider for the next block lingual
nerve
mylohyoid nerve
mental nerve
posterior
alveolar nerve
43. odontogenic lesion which does not show any
effect in radiograph is a. acute apical periodontitis
b. odontogenic
myxema c.
chronic
periodontitis d.
alveolar osteitis*
44. bone of
1st arch is a.
hyoid
b.maxill
c.mandi
ble
d. mental tubercle

45. growth of mandible begins at


a.4 to 8 weeks intrauterine
b. 8 to10wks iu
c. 10 to 12 weeks
d. in third trimester
46. crystals responsible for formation of enamel
a. hydroxyapetite
b
fluoropatite
c. calcium
oxide
47. needle gauge for fnac19gauge

48. when does there exist a gap between dentist and


treatment
a.resources need and treatment**
b. wide distribution of
resources c. needs
meet demands
d...
49. after bicuspidsation what is the most step to maintain the
integrity of the tooth parts
a. full crowns**
b. short span
bridge c. long
span bridge d.
none of the
above
50. patient came with yellowish coloured tooth.
radiographically reduced surface enamel thickness with large
pulp chamber is seen. your diagnosis
a. amelogenesis imperfecta
b.dentinogenesis
imperfecta c. regional
odontodysplasia d. dental
fluorosis
51. one of the criteria for impression material is its
hydrophilic ability to replicate the tissue details even in
presence of some moisture. which of the following impression
material is highly hydrophilic
a.
polysulphi
de b.
polyether*
c. addition silicon
d. condensation silicon
52. question about bull eye lesion
-> erythema multiformi
53. which of the following files have
positive rake angle a. k file
b. k
reamer
c.
protaper
d. pro
files
54. which of the following is used for
subgingival scaling a. hoe
b.
sickle
c.
grace
y
d. interproximal
55. conc of chlorohexidine in mouthwash
a. 1.2
b. 0.12
c. 12
d. 0.22
56. increased stress on periodontally
compromised tooth is called a. primary trauma
from occlusion * not sure
b. secondary " " "
57. adults are less prone to caries progression as compared to
children. this is due to the fact..
a. adults have hard tooth
b. age related deposition of
sclerotic dentin c. children Eat
sugars
d. adults are more prone to caries progression
58. MTA is most commonly used root canal sealant. mta is
a. mineral tetra oxi aggregate
b. mineral tri oxiaggregate
c. magnesuim trioxi amide
d. metal tetracycline and acid
59. veneer is
contraindicated in a.
fluorosis
b. short
tooth c.
bruxism
d. mouth breathers
60. fluoride supplementation in children with given level of
water fluoridation depends upon
a. temperature
b. type of
supplement c.
age of patient
d. gender
61. question about indirect retainer
62. on denture delivery day the denture wearer patient have a
space between upper and lower denture equal to insertion of a
wax knife when the other side is occluded.
the vertical relation and centric relation was devoid of any
discrepancy during the try in procedure. what is the next step
a. rebase
b. reline the denture
c. chair side selective grinding
d. articulating the denture again to check occlusion
63. long case on geographic tongue
64. ceramometallic crown preparation margin
65. maryland bridges are
a. conventional fpd
b. short span fpd
c. resin bonded fpd
d. long span fpd
66. minor connector are connected to
major connector a. at 90 degree angle
b. acute angle
c. obtuse angle
d. 180 degree joint
67. when porcelain tooth are placed opposite to natural
tooth it causes
a. wear of porcelain tooth
b. wear of natural
tooth c. no wear
of either tooth
d. fracture of the whole crown
68. system B uses
a. warm gp condensation
b. lateral gp condensation
c. cold condensation
d. single gp cone technique
69. a file have a diameter of 0.2 at the tip
how much should it be cut to get the diameter at tip of file
number 35
70. best root
canal sealer is a.
sodium
hypochlorite
b. calcuim
hydroxide c.
edta
d. rc prep
71. which of the following are effective against hepatitis b
infection
1. iodophor and sodium hypochloride
2. . formaldehyde
3. ethylene oxide
4. glutera
ldehyde a.
1 ,2 and 3
b. 2 3 4
c. 1 and 2
only* d.
3only
72. radiograph of compound odontoma
73. increase in copper in amalgam alloy causes
a. delayed expansion
b. elimination of gamma 2 phase***
c. increase in tarnish n corrosion
d. all of the above
74. periodontal attachment loss is
measured from a. gingival margin to
pocket depth
b.cej to pocket depth***
c. cej to the gingival sulcus
d. gingival margin to alveolar bone
75. mandibular 1st premolar is most likely to get pulp exposure
during cavity preparation because
a. small tooth
b. large pulp
chamber c.
typical
morphology d.
both a n b

76. type of bone best suited for implant placement


a. type 1
b. type 2
c. type 3
d. type 4
77. main cell responsible for periodontal modelling
a. osteoclast
b.
osteoblast
c.
fibroblast
d.odontob
last
78. forecep used for removing bony prominence during
osteoplasty procedure a. bone rongeurs
b. bone file *
c. periosteal
elevators d.
osteotomy
forceps
79. the property of antagonising the clasp to prevent
dislodgement in occusal direction is called
a.
resistance
b.
retention
c.
reciprocati
on d.
sealing
80. which property of wrought wire is better than its
counterparts for construction of clasp
a. retention
b. less stress on
abutment tooth
c. shape memory
d . flexibility
81. primary support area for upper complete denture is
a. rugae
b. alveolar ridge
c. hard palate
d. buccal flanges
82. Hutchinsons triad is specific to
a. tuberculosis
b. primary
syphillis c.
congenital
syphillis d.
gardners
syndrome
83-Clamp for partially erupted molar
14a
151s
53r
84-Grade II furcation involvement treatment of choice:
A- GTR (guided tissue regeneration)
B- full flap with
curettage C-
periodontal
therapy
85-Cutting edge of an ideal
instrument should be a. Parallel to
long axis
b. Perpendicular to long axis
c. Perpendicular to shank
d. 3 mm from long axis
86-PDI classification for missing teeth in upper and lower arch
including canines:
a)class 1
b)class 2
c)class 3
d)class 4
87- Rideal walker test
a. Used for selection of stainless
steel crownsa B. Used for
selection of restorations C. Used
for selection of shade
D. Used for disinfection
88-What is the blade length of cutting instrument with the
following formula 10-85-8-14 :
A-10
B-85
C-8
D-14
89-To remain stable , a rubber dam clamp must contact the anchor
tooth gingival to the
height of contour. Which other criterion must the clamp satisfy?
a) All four points must be sharp
b) All four points must contact the tooth
c) The bow must be directed to the distal side of the tooth
90-Severly stained maxillary central with smal distal caries in 20
yr old what is the best
managment:
A- pfm crown
B-all ceramic crown
C-veneer ceramic
91-Which of the following conditions is highly indicated for the short
therapy of DOTS and
is directly observed once in the clinic:
A) Tuber
culosis. b)
AIDS
C.) Syphillis
92-Fluoride in dental office do not cause
fluorosis because : A Very little amount of
fluoride
B Tooth already
calcified C) Saliva
wash the fluoride
93-process of removal of one root of a tooth to save the other by
leaving the crown intact
a. root resection
b-. root
amputation c.
bicuspidizatio
n
EXAM -3
1. Parapost technique(serrated))
2. Acromegaly—enlarge mandible class 3 haypparathyrodism
Shoeing of amalgam—(1.5-2 mm)
3. Positive rake angle :
a) k file b) k reamer
4. Fibrous dysplasia
5. Brown line on premolar, best way to reach diagnosis: a)
leave and return after 6 months for extension of
lesion****
6. Shape of max central and lateral access cavity
7. Epidemiology(( survey disease \ general natural
survey\etiology and history of disease ))
8. Lefort 1, severe bleeding, which artery in injured?
hemorrhage
9. Leukoplakia
10. Raspberry like lesion associated with
denture((papillary hyperplasia))
11. IANB needle length? 25mm
12. Pit and fissure probe? : a)blunt b)non forceful blunt
probing****
13. Renal patient , dental clearance on : a) non dialysis
day***
14. Provisional crown, occlusal high. We manage n
reduce occlusal but it became thin, why:
A)dueto over eruption of opposing tooth****
15. Denture, whemever tongue elevate, denture moves
up:
a)under extension
b) cramped tongue
c)over extension
d)improper occlusal preparation

EXAM -4
1- A class II RPD diagram, showing an error in the diagram itself. And the premolars
were mobile. The error in?

1- Major connector*** They were too sub-gingival away from the back of
teeth, becuase it needs ligual plate.
2- indirect retainer
3- stress breaking release kind of that
4- the metal mush
The kind of bacteria in the saliva?
1-Actinomycyces
2-Kind of Vironella!!!! can't recall the name!!!!
3-Staph
4-Strept
After doing an electro-surgry on the gingiva, you have a width of gingival pocket
depth of ???
1- 3 all around the surface.*****
2- 3 all around the proximal.

Do your research. 3- 2......


4- 2......

The distance between 2 implants....3mm


The maxillary division enter Foramen
1- Ovale
2-rotundum
3-spinosum
4-........

The amount of amalgam ingestion per day? 3, the same as in the files
dentifrices-1000
Flouride in ppm-1
A long case, sever pain on the cheek on touching or eating
...
1- Trigeminal Neuralgia*****
2-Ramsay Haunt syndrome ( do your research)
3-
4-
A question about the ****extended flanges after 24 hours from wearig the denture
back to the dr. The same in the files
An ulcer on the lower labial mucosa....
1-Over-contoured flanges********
2-Over-extended flanges
3-......
4-........
What do we call an incidence of population divided on the diseased population?
wasn't mathematical at all
1-Prevail*******
2-...
3-....
4-...
The outcome of a disease in a group of diseased people?
Etiological
Risk
......
Prognostic****
A diagram showing a porcelain upper teeth opposing lower natural teeth.....
*******Abrasion
EBA, the same as in the files....
Unfilled****
.......
Primer and bonding agent
...........
A question about the Halogen light, what to do???? can't recall, but i read it
somewhere in the files...
Fix the battery
don't do anything******
......
.......
A very tricky easy q about the FREEWAY space, been written in a way to think its the
leeway space, because they've wrote "in each arch"!!! but they've asked about the
FREEWAY space... Read carefully
Do your research. As in the files.
A question about the infiltration in the upper first molar
*****ٍ Supra-periosteal
A VIII patient.......Hemophilia A
vvHow to treat the hypersesitivity? Wasn't an actual treatment, but more to the
releasing of it......Can't recall the answer, but the same as in the files 1- Root planning
2- Open up the tubules to release the intra-pulpal pressure*****

EXAM -5

1- feature that describe the energy absorbed by a material before deforming. -


toughness **
2-crown to root ratio - 2:3
3-linchen planes histpothalogical characteristic.
4-best radio for proximal caries - bitewing
5-- 8years old patient with negative behavior you use for immobilize extremities
Posey straps
Soft belts
Papoose board***

6-most recurrent cyst—(OKC))


7-complete blood count test
8-bicondylar fracture in child without displacement or malocclusion
9- Submandibular gland open into
Beside lingual frenum***
Floor of mouth with 100/200 small ducts
Pharnx

10- Glazed crown in cementation stage ..there was proximal space with other
tooth..how to close?a-
add procaline
b-add composite**
c-add porcaline and composite
d- leave as it will drift and close space
11-milkiy appearance of porcelain –(( over firing))
12-best pit and fissure treatment – ((sealant))
13-child 4 years old with oral candidiasis treatment((Nystatin oral suspension))
14- Lining of maxillary sinus
A. Para keratinized
((B. seudo stratified ciliates volume ear epithelium with goblet cell****))

15-question about apexogenisis


16-question about IANB
17-how much anaesthesia in 1 carpule of 2% lid with epi1:100000-(2.4)
18-best and first endodontic test - cold ?
19-question about symptomatic apical periodontitis
20-question about Herpatic gingivostomatis
21-management of herpatic HSV
22-effect lidocaine toxicity on the heart – ((Tachycardia))
23-which bur to prepare the groove in a 3/4 crown(( taperd fissure))
24-asymptomatic tooth with vital pulp, distal caries and some incical
chipping restoration type - ?PFM
25-purple lesion on crown and root of max incisor - external resorption ?
26-what causes white flakes on incisors after removing oath brackets -
enamel hypoplasia - food debris ?
27-which tooth root mostly displaced in max sinus
28- best material for condylar graft
29- difference between cartilage and bone growth patterns
30-gracey and universal curets differences
31-area specific scaler - gracey
32-primary use of a dental wax
33-what to check last during try in: -aesthetics -occlusion contact ??
34-plaque consist of - bacteria
35- bacteria for initiation and progression of bacteria
36-question about a SCC on tongue poorly differentiated ... - bad prognosis
+ high recurrence
37-electricl pulp test affect - nerve impulse ?38-main pulp neural cells
39-anestheisa is dependent on - lipid solubility
40-functional appliance - bionator
41-what to study before placing a functional appliance - skeletal age ? - dental
development stage
42-patient with a newly erupted molar - pit and fissure sealant
43-patient had PSAB and still feels pain, need endo, what to anaesthetise next -
buccal infiltration ? - palatal
44-best impression material - PVS
45-most biologically acceptable base - CAOH
46-patient after ortho treatment had an external resorption- CAOH
47-question about Cohort study
48-disadvantage of Mcspedden technique
49-ehtical question about informed written consent before start of a
medical study
50-question about ectodermal dysplasia + skin pigmentation - Amelogensis
imperfecta
51-question about crown and root hemisection
52-what to do after bicuspidization - crowning ?
53-when to manage accidental root perforation
54-question about he focal infection theory
55-question about sclerotic dentin protecting the pulp
56-how long to keep topical anaesthesia -1min ? - 2min
57-question involving rpd + medial undercut which wire
58-patient with RPD after 5 years all is good but denture displaced away
form tissue when depressed distally, indirect retainer is good-
management ? - rebase ? - duplicate
59-when to perform surgery after dialysis
60-definetion of maintenance after treatment
61-when to give antibiotic
62-principles of GTR - space creation and won stabilization
63-when to stop thumb sucking habit - early mixed dentition
64-pacifier effect on dentition -posterior cross bite + anterior openbite
65-endo irrigation solution
66- misdiagnosed in bicuspid area - mental foramen
67-patient with difficulty in phonetics - lower anterior teeth set higher than
lip line? - using smaller size mold for maxillary
68- in developed countries caries risk is reduced due to - artificial
water fluoridation
69-question about adding butmaide.. to acrylic to make a stronger denture for
the patient - high impact denture ?
70-question about winged clamp and rubber dam placement- placed together
71-question about conscious sedation
72-best anticarogenic sugar substitute
73-ovate pontic used for - max anterior teeth
74-question about pericoronitis
75-quesiton about raising a flap
76-injury to which area will cause lip paralysis - mental foramen area
77-when to do biopsy
78-after placement of temporary crown we did occlusal check and found
thinning of the crown whats the reason - inadequate tooth removal.
79-when to do lingual frenum management - limit tongue movement
80- advantage of RPI clasp - esthetic
81-a week after replacing amalgam with high zinc patient comes back with
pain the reason - delayed expansion
82-pulp polyp is usually - hyperplastic
83-which systemic disease cause gagging in denture wearer - ??
84-missing 13 which abutments used - 11,12,14
85-caries detecting dye - propylene glycol
86-amount of amalgam daily in

EXAM – 6

1-During try in of PFM bridge restoration the patient said he feels as if a seed is stuck
between his teeth this indicates:
A- Tight proximal contacts**
B-over extended margins
C-under extended margins
2- A small part of a periodontal curette wad broken and displaced in gingival
sulcus. how to retrieve it?
a- Schwartz periotriever**
b- tweezer
3-Difference between Gracey currete and universal :
1-Gracey for specific tooth area , universal for all surfaces
2-Gracey has one side cutting , universal is both sides cutting
3-Gracey cutting end offset angle is 70 currete , universal is 85
4-gracey is semicircular cross section , universal is triangular
A-1 and 2**
B-2 and 3
C-2,3, and 4
D-1,2 and 3
4-water irrigation device:
a- dilute bacterial products**
b- remove plaque
c- prevent attatchment
5-Surfactant usage
Increase surface energy**
Decrease surface energy
6-Difference between standardized and conventional gutta percha??
Standrized : cold lateral condensation with warm vertical compaction
Conventional = additional cone for masterpoint technique
7-Instrument which use for grasping a tissue when remove thick epulis fissuratum:
a- Allis forceps.**
b- Addison forcep.
c- Curved hemostat.
d- Stilli forceps
8- pt with badly decayed upper 6 with mild pain, in x-ray there are series of radio-
opaque lines
a. paget's disease
b. osteosarcoma

c. garre's osteomyelitis**
d. fibrous dysplasia
9-Taper of preparation
10-two minutes after placement of the rubber dam clamp on lower molar the adjacent
become swollen and red
Hyperactive immun reaction to rubber dam
11-Clamp for partially erupted molar
14a**
151s
53r
12-Pt with attrition what is seen in xray :
A-Pulp obliteration
B-hypercementosis **
C-external resorption
13-Recommendation for use of fluoride toothpaste for child under 3years old:
Recommended**
Limited
Toxic
14-Patient with Burning mouth syndrome , female has hot flushes, they show you
some test results: Vitamen B12 is and Ca slightly low among others, cause of BMS is?
a. Pagets
b. menopause. **
15-Grade II furcation involvement treatment of choice:
A- GTR (guided tissue regeneration)**
B- full flap with curettage
C-periodontal therapy
16-Surgery and recountouring under immediate denture, type of suture:
1) horizontal mattress
2) vertical mattress
3) interrupted**
4) figure of 8
17-Cutting edge of an ideal instrument should be
a. Parallel to long axis
b. Perpendicular to long axis**
c. Perpendicular to shank
d. 3 mm from long axis
18-Porcelain appears chalky cloudy color, what happened?
a.) Over firing**
b.) Under firing
c.) Excessive moisture
19-PDI classification for missing teeth in upper and lower arch including canines:
a)class 1
b)class 2**
c)class 3
d)class 4
20-Time to establish gingivitis by days:
5-7
7-14
3-5
14-21**
21-The movement of water across a selectively permeable membrane which needs
energy to be accomplished is called:
a. Osmosis.
b. Active transport.**
c. Filtration.
d. Diffusion.
22-Pt feel sever pain in upper mouth pain is radiated to ear and eye after you check
no caries when you press on 1st molar pt feel pain ? Neuralgia
Maxillary sinusitis**
23- levator superior palpebral which nerve
Facial
Trochlear
Ophtalmic
Oculomotor**
24-Child come to clinic with total reverse of upper anterior teeth
sometimes child discease his chin & lateral cephalometric give this
results Sna 80
Snb 82 Anb -2 what is diagnosis?
A-Class I malocclusion with skeletal class III B-
class III malocclusion with skeletal class I C-
class III malocclusion with skeletal class II D-
class III malocclusion with skeletal class III**
25-Pt cleft palate we start expansion the max :
1- first three months of normal max growth
2-after eruption of primary teeth
3-before three months from bone graft **
26-Best way to disinfect impression
A.Autoclave
B.Uv chamber
C.Disinfectant
D.Chemical sterilizer**
27-Spedding principle:
a. Used for selection of stainless steel
crowns** B. Used for selection of restorations
C. Used for selection of shade
D. Used for selection of sealant
28-Badly decay primary first molar possibly involving four walls. Best
restoration A- stainless steel crown**

29-Increased depth & rate of respiration is called:


a. Cheyne stokes breathing
b. Hyperventilation**
c. Hypoventilation
30-Periapical x ray
A patient had an endo trt on 47, 6 months ago with a temporary filling now he have
pain on biting. On the radiograph there is RL refered to 47 and on examination we
found a narrow deep pocket on the buccal and lingual side only. The 46 have a good
amalgam restoration from 15 years ago. What is your diagnosis:
A- microleakage in 47 B-
vertical root fracture **
C- i can’t remember
31-Panaromic showing super numerary teeth and many unerupted teeth. Diagnosis
Ectodermal dyplasia
32-Periapical xray to a missed maxillary central with a odontogenic tissue
ressembling to a compound odontoma i cant find a similar x ray
33-Main Usage of sodium hypochloride
In dental clinic
34-Patient with mild pain present with a 15 treated endo from 3 years on radio graph
rl related to 15 : 1 acute apical abcess 2 chronic apical abcess 3 acute apical
periodontitis4 chronic apical periodontitis
35-7 y.o with thumb sucking, how to start treatment:
a- conseling**
b- psychiatric
c- orthodontic
36-What is the blade length of cutting instrument with the following formula 10-85-8-
14 :
A-10
B-85
C-8**
D-14
37-To remain stable , a rubber dam clamp must contact the anchor tooth gingival to
the height of contour. Which other criterion must the clamp satisfy?
a) All four points must be sharp
b) All four points must contact the tooth**
c) The bow must be directed to the distal side of the tooth
38-What anesthesia has most vasoconstrictor action?
A. Cocaine**
B. Tetracaine
C. Procaine
D. Articaine
39-During taking biopsy from posterior third of tongue, how to hold the anterior
part of the tongue:
a- towel**
b-appliance
c- no need
40- atraumatic restorative treatment, is defined as “a dental caries treatment
procedure involving the removal of soft, demineralized tooth tissue using hand
instrument alone, followed by restoration of the tooth with an adhesive restorative
material:
Alternative restorative treatment
41-Cementosis and ankylosis of the teeth is common features of:
1. Cherubism
2. Osteomyelitis
3. Paget's disease **
42-Anterolateral area of hard palate submucosa contains:
• mucous glands
• serous glands
• adipose tissue**
43-Severly stained maxillary central with smal distal caries in 20 yr old what is
the best managment:
A- pfm crown B-all
ceramic crown C-
veneer ceramic**
44-Difference btw chronic and acute apical periodontitis :sinus tract
45-Type of bone best for implant ?
type 1
type 2**
type 3
46-LEAST Implant Success in bone:
a. type1
b. type2
c. type3
D. type 4 **
47-carbohydrates affect caries mostly by:
a. frequency**
b. quantity
c. consistency
48-Which of the following conditions is highly indicated for the short therapy of DOTS
and is directly observed once in the clinic:
A) Tuberculosis. **
b-HIV.
C) H1N1.
D) Mental illness
49-Lateral condylar guidance :
A. L=H+12/8
B. L=H/12+8
C. L=H+8/12
D. L=H/8+12**
50-pregnant women in second trimester came to your clinic , what is the best local
anesthesia ;
1- articain
2- lidocaine **
3- bupivacaine
51-Fluoride in dental office do not cause fluorosis
because : A Very little amount of fluoride
B Tooth already calcified**
C Saliva wash the fluoride
52-Attachement level is the
distance from:
a-CEJ to pocket depth**
b-CEJ to mucogingival junction
53-Associated with renal failure :
A- hyperthyroidism
B-hyperparathyroidism
C-secondary hyperthyroidism
D-secondary hyperparathyroidism**
54-process of removal of one root of a tooth to save the other by leaving the crown
intact
a. root resction
b. root amputation **
c. bicuspidization
55-surfaces visible in FOTI
A. All surfaces of all teeth**
b. all surfaces of anterior teet
c, proximal surface of anterior
d. proximal surface of posterior
56- the needle is parallel to occlusal plane during anesthesia:
A- gow gates technique
B- vazirani akinozi technique**
C- IANB

EXAM - 7

1. Bacteria in Osteomyelitis,
A. staphylococcus***
B. Streptococcus
2. Long qstn clinical presentation, with opg & occlusal view___ buccal bifurcation
cyst
3..qstn related to fracture of condyle
4.Regarding needle in vazironi Akinosi
techniques
A. 25 guage short needle
B. 25 guage long needle***
5 .Residual cyst image_ [opg]
6.qstn related to osseo integration in implant
7.controll tooth no TOP
cold 5sec,
heat 3sec,
EPT 15sec
Test tooth severe pain on Top
Cold no response
Heat no response
Ept no response
Diagnosis a. Reversible pulpitis
B. Irreversible pulptis
C. Vital pulp
D. Necrosed ***
8.qstn related to fracture of ethmoid bone
9 .Management of coronal 3rd # in pedo
10. Most effective preventive messure for pit and fissure __sealants
11. Long qstn, (attrition , pulpal obliteration, sibling lso has same problem) __
A. dentinogenesis imperfecta Osteogensis imperfectia
B. Amelogenesis imperfecta
12 . Patient with severe pain & fluctuate swelling,patient not allowed to touch the
tooth emergency management?
A. Incision and drainage
B. Complete debridement, incision & drainage
13 .systemic Antibiotc indicated in___
A. cellulitis **

14. Management of thumb sucking__early mixed dentition


15. Activation angle of blade__45 - 90
16. Maryland bridge__resin bonded
17. La used in pregnancy___lidocaine
18. Action of vasoconstrictor in la_
A. increase duration & increase intensity of
LA** B. Decrease intensity of LA
19. Resorption due to ortho management___caoh2 dressing
20 .patient with trauma of tooth, non lingering pain to cold, tenderness on percussion
diagnosis
A. Asymptomatic irreversible pulpitis, symptomatic apical periodontist B.
Asymptomatic reversible pulpitis symptomatic apical periodontist C.
Symptomatic reversible pulpitis, symptomatic apical periodontist
21 crown 10mm, root 15mm, asking about crown, root ratio___ 2;3
22. Qstn related to opg
23.Last step before cementation of crown____ polishing of metal
24. shape of acess openg of pm___oval
25 .long Clincal qstn regarding veneer
26.Gingivectomy__ to eliminate pseudo pocket
27.Dentinal changes in deep caries
28. 2nd best xray for implant
29. Qstn related to Step back technique
30. Dibetic and hypertensive patient has lesion
1 *2 cm on lateral border of tongue, type of
biopsy A. excision
B. Incision
31.missg 6, tilted 7, planning for fpd, best management
Ortho extrusion of 7
32. Material used in preventive resin__
33. Smoking & non smoking gingivts__less intense in smokers
34. steps of sterilization
35. qst related to Green stick fracture
36. Clean and dry cavty
37.Utility wax main ingredient
A. Paraffin***
B. Gumdammer
38. Force applied to pdl during orthodontic treatment___ light force
39. Curing light 450nm what is the decision nothing to do
40. Instrument used in gingival bevellg in class 2 ____GMT
41. Qstn related epoxy resin, type of impression material ____poly ether
42. Abcess
incision A. no 11
blade*** B. No 12
43.patient on warfarin about to do surgery for him, which of following is more
important
A. Pt***
B. Ptt
44. principle of GTR
A. Space creation & wound stabilization ***
B. Root biomodification & wound stabilization
45. intrusion type of resorption ____Inflammatory resorption
46. . long case history[ fever, malaise, gingivitis] ___Hsv
47. . Discoid cleoid instrument___ Amalgam carving42.long qstn about addison
disease
43.recent advantage of Diagnodent
44. Qst related to Fibro optic
A. Early detection of superficial caries
B. Deep caries
C. Tooth fracture
45.qstn related to flap
46. Most retentive crown__Full metl crow
47.most favourable taper of rpd __
A. 0.010**
B. 0.020
48. Ameloblastoma follow up__10years
49. property of gic__fluride release
50. Water fluoridation___1 ppm
51. Decreased alkaline phosphates, ealry loss of lower anterior
52. Long qstn related to Papillary hyperplasia

EXAM – 8

1...Quadlock devise
-Fixed dentoalveolar expansion @
-Fixed facial alveolar expansion
2...ANUG
-Pseudomembranous @
-Vesciles
3 ...gingivectomy
-Subrapeiodontal abcess
-Invasive alveolar surgery
4 when do first time use toothbrush
- when first tooth erupt
- when 2 years old
5 image with caries on molar with periodontal recessions and caries under CEJ what
type of caries
-occlussial
-proximal
- root caries @
6 fibrous dysplasia
- enucleation ??

EXAM – 9
1-Cementum in coronal 2/3 have:
d. Acellular intrinsic fiber
c . Cellular mixed fibers
b. Acellular extrinsic fiber **
d. Intermediate cementum
Proxy brush with which type of embrassure: type 2
3.Streptococcus mutans initiates caries but Lactobacilli progress caries to
cavitation

4.After appling porcelain over metal and firing cycles , porcelain found cloudy and
milky what is the cause
-over firing
-under firing ***
-over opacities
5. broken cusp upper premolar not carious not restored with cold water has brief
pain what’s the pulp status irreversibly inflamed reversibly infamed
innervated A delta fibers >> my answer ( no reason :D)
6.With two wall defect (osseous crater) better use freezed
dried bone decalcified cortical freezed bone
decalcified cancellous freezed bone
7- Multiple sebaceous glands cysts and supernumerary teeth and
other manifestations -gardner’s syndrome
8- To increase bone density in graft site
-bone crib
-Devascularized cortical and cancellious bone
-vascularized cortical and cancelous
9- bout epidermolysis bullosa on skin .. What is the cause :
a.hypophosphatasia
b.Amelogenesis imperfecta ****
c,osteiogenesis imperfecta
d.Dentenognesis imperfecta
amelogenesis imperfecta ??? it ‘s said hypoplastic teeth in mosby book so I chose
it dentinigenesis imperfecta Osteogenesis imperfecta
10- Dental bud at witch weak intra uterine 4 - 5 - 6 - 8
11- two central incisor in 4 years boy intruded 4-5 mm to follicle of permanent whats
the ttt
-leave and observe
-reposition and splint
-remove as quickly as possible **
12- disinfect gutta percha
-naocl **
-hot oven
-autoclave
13- child with multiple RL in lower mandible in ramus area expanding jaw and
making lower 7 follicle become advanced under lower 6 -cherubism -fibrous
dysplasisa
14- Facial nerve supply which
-buccinator **
-med pterygoid
-lateral pterygoid
15- cranial nerve sensory for orofacial area
-VII
-trigiminal V **
-IV
16- dental floss to -drisrubt plaque formed
17- brushing is to
dirupt plaque matrix formed **
-remove remaing food on teeth

18- mother calls u saying her child swallowed 50 mg fluoride paste what to do

-leave and observe manifestations -have acidic drinks and go to emergency -have
calcium and go to emergency **
19- muscles of long midface to short midface
-increase -stronger -weaker **
-the same
20- to re endo and remove silver points by
using hemostat or pliers
21- in lower lateral incisor with 1 mm remaining tooth structure over gingiva what is
used -custom made post w core
-carbon fiber post with composite core
-threaded post with amalgam
22- facial nerve supplies
-buccinator **
-lat pterygoid
-med pterygoid
-masseter

23- functional appliance


bionator **
-bite block
two other options

24 – to preserve root from peroforation and filling material integrity what should
be used to remove gutta percha from root for post application

-drill** -heat
chemical solvent

25- posterior bite block for treatment


of -anterior cross bite with deep bite
26- child with pain in lower E and
dentist removed 1-5 mm of pulp
what is this called
-pulpetomy
-partial pulpetomy
-pulpectomy
-apexification

27- picture of upper jaw only and


arrows mesial to upper
primary canines
-primate space

28- Bacteria grown in plaque communicate with each other


through
-quorum sensing *** not signaling
-signaling
-maturation
29-difference between the alveolar epithelium and the gingiva epithelium is a-
Absence of stratum spinosum b-Absence of stratum granulomatous C-
Absence of stratum cornium **

31. case about female patient came after receiving denture complaining
about inflammation in her lower anterior area under denture with some
ulceration asking about what type of ridge she has
-flabby ridge
-khife edge ridge ** my answer
-flat ridge
32.most caries prone surface >> Buccal surface of maxillary posteriors
(maxillary first molars)

33-clamp used for partially erupted tooth?

a. #14 A. b. #W4A.

EXAM – 10

1.which compartment contains body fluids ..intestitial , intercellular ,


intravascular , transition compartment

2.adverse effect of cyclosporins ..staining , gingival hyperplasia , mucosal


discoloration

3.long span bridge ..high strength and high rigidity*


4.limitation of using EPT..open apex, preganacy , narrow ya curve canals
_5.before application of pit and fissure apply..phosphoric acid **, 3% hydrogen
peroxide
6.Before applying fissure sealents in primary cavity .apply varnish , calcium
hydroxide , clean n Dry
_simplifil question..universal Protaper
_ substansitivity..chlorhexidine Gluconate
_medicine Causing gingival enlargement ..phenytoin
_ which is not pulpal test ..percusion , EPT
_long question about missing 38 carious 36 same as file..mutlilicular
radiolucency..ameliblastoma
_  Lesion at junction between hard and soft palate and surrounded with
pseudo:
1. Epithelium-hyperplasia in salivary gland. = Adenomatoid Hyperplasia
2. Necrotizing sialometaplasi***

_melonotic pigmentation kn skin and renal problem..addison disease


_copper lesion , hearing loss, notch incisor..congenital syhilitis
_question about simple bone cyst long case same as file.
_firm fixed nodes ..SCC
_Patient came hspitle fracture of mandibular symphysis wire used..6 inch 26
guage
_dentifrices ..1000ppm
_gates galliden bur used ..enlargement of coronal structure
_defination clinical attachment level..cej to pocket depth , cej to mucogingival
_how can we check periodontal disease progress...by attachment level
_differentiate between endodontic and nonendodontic periapical
lesion..radiogeaphic appearnce , EPT
_differentaite between vital and nonvital pulp ..EPT
EXAM – 11

1. case about class III malocclusion , what is the treatment ---> Advancement
of Maxillary Anterior (incisors)

2. class III restoration , which bevel ---> short bevel long irregular* , hallow
smthng {i dont remmber the options}

3. fluoride content in dentifrices --> 1000

4. pt feels burning sensation after injecting diazepam due to?


5. a.presence of propelene glycol******
6. b.presence of adrenaline

7. Pic of protruded upper and lower central incisors , after treatment its back to
position , what type of movement used -->

8. in a patient 9 yr when will the diastema closes ---> eruption od permenant


canine

9. Q about REATAINER in fpd and rpd

10. restorative material to be glazed to avoid dehydration ---> GIC

11. CASE (same like files ) ---> Trigeminal nuralgia

12. CASE pt have renal problem , and confusion ..... ---> hyperparathyroidism
11. raiograph ,Pt with blue sclera teeth , with multiple fracturs ---> DI , OI { i
choose DI but i think its OI since the patient have multiple fractures}

12. when scalining , the angle b/w scaler and facial surface of tooth ---> 45:90

13.case about patient surffering from severe pain , awaken him at 2 am cant
sleep ---> irreversible pulpitis

14. epithelization of gingiva ---> 7-14***, 5-21

15. DI occurs in ---> histodifferentiation

16. amalgam in retrograde ---> zinc free ,** copper free , { i dont remember the
rest i chhose zinc free plz check}

17. radiograph , pt having RCT on 47 and amalgam on 46 , with pocket 6-7 mm


related to 47 ---> vertical root fracture

18. before filling in pedo ---> cavity varnish

19. attrition of upper and lower bicuspids , what will u see in the radiograph --->
hypercementosis { not sure }

20. full arch extraction , next step ---> interupted suture , ... { there was long
options i choose the one with intrupted sutures}

21. determining working length in endo rule ---> curved file to locate canal * ,
bisecting angle parallel

22. 20 file ---> 0.2 mm

23. pain definition ---> unpleasant sensation ....

24. etiological factor definition ---> most accurate factors that decide or confirm the out come


disease present in high population count

25. cohort group definition ---> group of people in the study followed up

26. pic white discoloration due to ortho treatment , patient want esthatic, we
will use all except ---> coservative composite* { all othere options were veneer}

27. how to icrease ZO working time ---> a.add eugenol drop , b. mix on cold
glass slap ((, c. mix on paper pad { i choose eugenol drop not sure }

28. prognosis decrese ---> necrotic pulp with periapical lesion

29. proliferation of tissue under CD ---> surgical excision

30. amount of vasoconstrictor in LA ---> 36mg

31. 1st thing to do when removing Rubber dam ---> cut septal

32. tug back --->

33. indirect pulp capping used when ---> reversible pulpitis

34. most common ---> cleft lip and palate


35. case about missing 18,17,16 , and missing 24, 26 , there is carious 25 to be
extracted , which class is it after extraction of 25 --->

a.class I mod I

b.class I mod II

c.class II mod I ** { plz check }

d.class II mod II

36. imediate denture purpose

37. rest function

38. diagodont- queantitve \superfacial surfACE

39. most affected cell by radiosensitive --->basel cell mucosa forgot the options :(

40. anterolateral in palate ---> taste buds , Sebacous

41. after GA ....... for restorative treatment, pt is preped and anasthesied ,what
s next step ---> take raiographs , give prophylactic AB

42. analgesic for child pt with renal disease ---> acetamenophen (check the dose )

43. deflection of FPD (dont remember options)

44. pic of implant analogue

45. calcium hydroxide used b/w visits ---> antimicrobial affect

46. cleft palate occur in ---> 8-10 week IUL

47. gingivectomy indication all axcept ---> excessive osseous problem

48. amalgam cavosurface ---> 90'

49. open margin crown ---> remake

50. pulp polyp associted with ---> a.reversible pulpitis b.chornic irreversible
puplitis c.necrotic pulp d.non-vital { i choose b but am not sure}

51. enamel can withstand forces ---> a.enamel tufts , b. enamel lamella , c. g..
enamel { check the answer } Gnareld enamel

52. autoclave ----> moist heat under pressure

53. cell wall biosynthesis ---> penecillin

54. composite matrix ----> BisGma

55. long Q about defeciency in factor VIII ----> Hemophelia A

56. substantivity .... (like files ) ---> chlorohexidine glugonate

57. in apicectomy the angle should be ---> acute


58. hypertensive pt 210/100 ----> reschedual the appointment

59. bacteria comunicate with each other ---> courum sensing

60. case about condyle displacment best ---> reverse town

61. how many time should complete denture be washed ---> a. after every meal
* b. twice c. thrice { plz check }

62. angular chelitis in pt wearing CD since 5 years ---> high intercuspal distance

63. thumb sucking causes ----> ant openbite post crossbite

64.single implant consideration ---> antirotation coronal somthing , { i dont


know the ansewr }

65. most common benign tumor of salivary gland ---> pleomorphic adenoid

66. case about CONDENSING OSTITIS

67.function of vasoconstrictor ---> reduce toxicity

68. pt cleft palate and lip , pit lip , anodontia ..----> van der waund

69. case about fracture in nose , which forcep ---> walsham's forcep

70. addind surfactant to irrigation ---> reduce surface tension

71.case of avulsed tooth , how long do u splint ---> a.1* b.2 c.3 d.4

72. ceramic primer ---> silane coupling agent

73 & 74. drug causes gingival enlargment ---> phenytoin {this q was asked twice}

75. subgingival inst ----> gracy

76.what make priority to private clinic than community ---> need assistant
outcome { in files they chhose assistant but i choose the outcome so check it }

77. with acute pericoronitis , managment ---> rinsing swap antibiotic

78. fluoride used in clinic ---> duraphat

79. mother call ....... asking about avulsed tooth ---> cool milk

anesthesia not working on inflammation due to :-


A- low ph***
B- inflammation dilute anesthesia
C- increase blood circulation

80. prefered root for abutment ----> multiroot

81. sudden sharp pain which nerve ---> A

82. pt with paralysis of left side for 15 days includes eye , upper lower lip,
anterior third of tongue cannot raise eyebrows , which injury of facial nerve -->
a.injury to parotid gland**

b.chondra tympani

c.upper part of facial nerve ** (not sure)

84.pt with deep pit and fissure 25 yr have high careis risk ---> sealant

85. best treatment to prevent caries ---> sealant

86. pt have white spot on buccal surface of molar , caries risk ---> high*,
moderate, mild

87. using fluoride in dentifrices, supplement dose not cause florosis ---> teeth
already calcified

EXAM – 12

1.simplifil apical seal used with which system:


A. Protaper
B. Reciprocal
C. Revo S
Ligtspeed ****
2.what kind of periodontal probe is used in the furcation area?
A. WHO.
B. Naber’s probe
C. UNC 15
D. Michigan

3.what is the radiographic tech being appropriate if used properly while


planning for implant?
A. occlusal
B. periapical
C. Panoramic**
D. CT
E. MRI

4.fiber optic diagnosis is


A. Quantitative- Diagodent
B. Qualitative
C. Qualitative and quantitative

5.Pt. presented to u having root recession he has pain when putting


probe gently on the root what is the diagnosis:
A. Dentin hypersensitivity.
B. Reversible pulpitis.
C. Irreversible pulpitis.
D. Apical Periodontitis

6.what is the mostly observed tissue response of successfully following


oral hygiene instructions?
A. reduced pocket size
B. reduced plaque index
C. reduced bleeding
D.
N reduction in swelling
7.70 yrs. Old male pt. Comes to restore his badly decayed upper second
molar .chance of involving pulp by the infection from dentin compared
to young patient?
A.Progress slowly in adult *
B.Adult suffers less pain compaired to young
C.More progreesing in old patient than young
D.higher pulpal involvementhavhe less pulp affection

8.Patient with roughness on skin and shiny palms widening of pdl space
but with no ridge
restoration and there is a bilateral destruction in angle of mandibular
bone ,what is your
diagnosis:
A.Neoplasm
B.Scleroderma
C.Hyperparathyroidism
D.Aggressive periodontitis

9.diameter of number 20 gutta percha


A. 0.20mm***
B. 2.0mm
C. 020mm

10. how long is the rinsing time of chlorhexidine in


mouth wash to be effective A. 15 s
B.
30 s
C. 40
s D.
50 s

11. concentratoion of
chlorohyxidin mouthwash is A. 1.2%
B. .
12%
C.
2.1%

12 .What is the important biomechanical propeRty required for


single missing tooth implant A. abutment made up on titanium
alloy===**
B. abutment should be made in two parts
C. abutment should restrict the coronal rotation

13. Class 2 malocclusion with long faceorthodontics


extract upper per molar
14. A. relieve incisor flaring
B. make incisor large
C. extraction isn't indicated**

15. What different between center of the


growth and site of growth?
A.
Independe
nt B.
Centered
C. The center of growth is rapid
16. Radiographic criteria used for evaluating
success endo therapy A. reduction of size
periapical lesion
B. no response to percission and palpation test
C. extension of sealer cement
through laterla canal D. non above

17. Avulsed tooth


best medium A.pt
saliva
B.milk at room
temprature C.milk
at cold temprature
===D.saline
18. Oral manifestations of
HIV in a child A.parotitis
B.kaposis sarcoma C.herpes gingivostomatitis
D.candidiasis
18.Spedding principle:
A. Used for selection of stainless steel crowns
B. Used for selection of restorations
C. Used for selection of shade
D. Used for selection of sealant
19.Mature tooth with caries expo of pulp treatment by
A. RCT
B. Pulpectomy
C. pulpotomy

20.identify the lesion(long question)

A. Radicular cyst
B. Nasopalatine duct cyst
C. Dentigerous cyst
21. Long question about submandibular sialolith and somewhat same
pic

22 identify the local anesthetic technique

23.Impression material most stiff is:


A. Poly ether
B. Alginate
C. Poly vinyl siloxane
D. Agar-Agar
24.matrix and patrix part of (question was not direct like this)
A.Clasp
B.Attachment
C.Major connector
25.Disadvantage of full thickness
mucoperiosteal flap A. Delayed
secondary healing B. Scar tissue
formation====
C. interdental papilla integrity
26.Periodontal flap in which epithelial lining of periodontal pocket
gets converted into attached gingiva?
A. Modified Widman flap,
B. Apically positioned flap,
C. Undisplaced flap,
27.Functional appliance
A. Bionater
B. Posterior bite plane
C. Head gear
28.4yr old with 5mm intrusion of upper incisor which touching the
permanent follicle what do you do?
1. Wait and see if erupts
2. Extract carefully**
3. Crown lengthening
4. Reposition manually and splint

29. Reciprocal arm of cast partial denture


A resist lateral movement of the prosthesis
B resist potential orthodontic movement of the abutment
C counteracts the force produce by clasp ***
30.what is the most effective powerful hemostat with heavy bleeding
after extraction
A. Cotton soaked with epinephrine
B.Oxidized cellulose***
C.gelatin
D.hemcan
31.Order of placing of winged clamp type rubber dam
A.clamp before dam
B.clamp aftr dam
C.clamp and dam together
32.patient with multilocular radiolucency in mandible angle and
multiple fractures
A.myloma
B.osteomyelitis
C.hyperparathyroidism===
D.florid hypoplasia
33.According to CDC , to clean instrument before sterilization we use :
A. chemical
B. ultrasonic cleaners===
C. blood dissolving solution
D. hard brush

34.avulsed tooth remain for 60 min what should do :


A. immerse in sadium hypo
B. in sadium flouride
C. in sadium flouride then hypochlorite *
35.The main advantage of immersion technique is?
A. Prevent the impression from distortions
B. Helps all the surface of the impression
to be disinfected 36primary teeth
occlusion that develop CIII:-A. Mesial step
more than 2 mm
B. Distal step more than 2mm
C. End on
D. Edge on
37. patient came after avulsed tooth managed what's you
appropriate time of splinting: A. 1-2weeks.
B. 2-3
weeks C.
3-4
weeks
38. A 60 year old woman comes to the clinic with complaint of
angular cheilitis and she has been wearing the same denture for last
many years. What is the reason for this?
A. Decreased Vertical
dimension B. over
extended denture flange
C. under extended
denture flange
D. something about wrong eccentric position
39. A young 14 year old patient with excessive plaque and calculus
and u decide to do scaling for him.what is the best ultrasonic to be
used which moves in an elliptical motion
A. piezoelectric
B. ultrasonic
C. magnetostrictive
D. sonic
40.Retention of veneer
A. preparation of tooth
B. Micro mechanical due to etching enamel & veneer’**
C. mechanical retention
D. groove
41 upper molar anesthesia in child
42.Tuberosity technique for block :
A. P.s.a nerve
B. m.s.a nerve
C. maxillary nerve
43.43 years old patient is scheduled for extraction of grossly decade
maxillary premolar, clinical history reveals that the patient has
increase in atypical plasma esterase which of the following local
anesthesia agent maybe safely administered to the patient: A.
procaine HCL with 1/200,000
B. Propoxican HCL with 1/50,000
C. Prilocaine HCL with 1/280.000
D. Procaine HCL without adrenalin
44.Material for Chair Side relining of denture?
A. Soft liner
B. Light cure acrylic resin
C. Wax
D. Acrylic
45.Angulation of the face of blade of the scaler to stone during
sharpening :
A. 50-60°
B. 70-80°
C. 100-110°
46.Decalcificiation in case of fixed orthodontic appliances occurs most
commonly on which surface of tooth?
A. Lingual
B. Inter proximal
C. Just behind the brackets
D. Around the brackets
47.Pt. Had undergone radiotherapy before 10 months. To make
impression of removable partial
denture what is type of material
A. plaster of Paris
B. Rubber based
C. compoundd. ZOE
D. Alginate**
E. agar-agar
48.pt came with sever pain on his first permenant molar &with routine
examination
dentist found white wrinkled ulcer on buccal mucosa ask pt ‫ ا‬say
that tobacco consumer since 10years what to do
A. refer to pathologist to take biopsy
B. follow up after 2weeks
C. topical anaeshesia &follow up
D. Its just a tobacco pouch no treatment needed
49.bad taste and smell in the patient's mouth, there are bubbles in the
retiner cervical area,
diagnosis is:
A. loosening of the retainer
B. more occlusal forces
C. food imapction under the retainer
D. breakage between connector and retainer
50 .Pt underwent renal transplantation ions 3years ago he white non
scrap able lesion on the
lateral side of tongue appeared corrugated and he has shaggy and
frayed whats your dxs?
A. hyper plastic candidiasis
B. idiopathic leukoplakia
C. lichen planus
D. hairy leukoplakia===
51.what does Enamel bonding agent (EBA) consist of:
A. Unfilled resin
B. Primer and bonding agent
C. A mixture of resins in an acetone or
ethanol solvent D. A wetting agent and
resins 52.Indirect retainer placed?
A. Near the fulcrum line
B. Near gingiva.
C. Near edentulous area.
D. away from the fulcrum line
53.when patient says ‘ahh’ half of the soft palate is not moving.which
nerve we test:
A. Glossopharyngeal
B. Vagus
C. Hypoglossal
54.Most destructive finish line:
A.shouldeR
B.chamfer
C.feather edg ( most conservative )
55.Important sign of fracture in the
body of mandible A. Upward shift
B. Parasthesia of lower lip
C. Medial shift
56.The most destructive occlusal interference is
A. Centric
B. Working
C. Non working
D. protrusive
57.The usual cause of contacting or clicking posterior teeth :
A. Decreased vertical dimension
B. Increased vertical dimension
58.if the crown accessibilty is less during endo and limited removal of
pulp and debris can cause A. Crown fracture
B. Crown perforation
C. Discoloration
59.8 year Patient came to your clinic has impairedhearing, upon
examination his mouth you found copper color lesion , notchedincisor
and mass on the occlusal surface of the molars . this patient has :
A. Congenital syphilis
B. Gardner's syndrome
C. Turners hypoplasia
60 most common organism which causes caries
A. Lactobacillus
B. Streptococcus mutans
C. Actinomycetes
61.Anticariogenic sugar substitute is :
A. Xylitol
B. Mannitol
C. Sorbitol
62.Carbohydrate effect on caries by
A. Duration
B. Form
C. Type
D. frequency
63.pt.has upper right 6 endodonticaly treated tooth with small MOD
caries.The best treatment:
A. Gold crown
B. MOD gold inlay
C. MOD gold onlay
64 Dental implant are successfully with minimum failure:
A. pre maxilla area
B. post.area of maxillary arch
C. mandible between mental foramen**
D. buccal shelf of the mandible
65.antibiotics prescription is recommended when:
A. diffuse rapid spreading infection
B. acute localized infection
C. chronic infection
66.Treatment of acute pericoronitis?
A. antibiotic only
B. Reduce the opposing tooth and sub gingival
C. rinse with antiseptic*
D. no treatment
67.After serial extraction what type of sutures??
A. fig 8
B. interrupted***
C. horizontal mattress
D. vertical mattress
68 somewhat same pic of tori with long question
69.Which of the following statement is true regarding dental calculus:
A. It is composed entirely of inorganic material.
B. It is mineralized dental plaque.
70.dentist at the end of the day want to pour alginate imp
quickly how can he do that
A. Increase powder/water ratio
B. Hot water
C. Slurry water
D. Increase thickness
71.(pic was there.but I couldn’t find exact pic in google)fracture of
upper denture in midline from ant to pps and patient said that it
was broken before 2 or 3 times n by
examination presence of inflammation on
residual ridge n sever bone resorption Wat
causes the fracture
A. Ill fitting denture n thick frenum
B. Unbalanced occlusion
C. Thin denture
72.the extracanal is present in which root
of 1st max. Molar A. palatal root
B. mesiobuccal root
C. distobuccal root
D. mesiopalatal root
73.42. the use of low speed hand piece in removal of soft caries in
children is better than high
speed because
A. less vibration
B. less pulp exposure.
C. better than high speed
74.1.After gingivectomy surface
epithelisation occurs in A. 3 days
B. 5-14==
C. 14-21
D. Over a month
75.After trituration of amalgam condensation must be
A. after (3_4)min.at least in order to remove excess mercury
B. vertically
C. with little pressure
D. immediatly.
76.Sharp pain is due to which type of fibers?
A. A delta fibers
B. B delta fibers.
C. C delta fibers.
77.which laser can we use instead of halogen composit light
A. Co2
B. Nad yag
C. Argon
78.Device that used to detect fissure caries with electrical resistance
A. Laser
B. Fluorescence
C. Electric caries measurement
79.If you did two holes of rubber dam too close what will happen :
A. Difficulty of putting dam interdentally
B. Stretching of dam will happen and
subsequent leakage **
C. Wrinkling of dam
80.The depth of cavity prep. for composite in posterior:
A. Limited to enamel.
B. 0.5 mm. in dentin.
C. Depends on caries extension.
D. Depends on tooth discoloration.
E. 0.2 mm. in dentin.
81.whats the meaning of attachment level
A. from gingival margin to depth of pocket
B. from marginal groove mucogingival line
C. from dentino cementum junction to
mucogingival line D. From cemento
enamel junction to mucogingival line
82.patient came with ulcer on the dorsum of tongue. Lab report says
that Poorly differentiated squamous cell carcinoma. What does that
mean? A. Good prognosis , high recurrence
B. Bad prognosis , high recurrence
C. Bad prognosis, low recurrence
D. Good prognosis , low recurrence
83.PRR indicated in
A. Shallow caries involving entire fussure
B. shallow caries involving half and
other half deep C. caries involving
only half of fissure
84.Development of mandible from which cartilage lateral
to pharyngeal arches A. Meckel’s cartilage
B. Reichertz cartilage
C. Thyroid cartilage
85.time of curing in very small class 3 composite restoration
A. 15 sec
B. 20 sec
C. 10 sec
D. 5 sec
86.Decrease the effect of acid etching on the pulp the three length
of filed and reamers that u work by them :(question was entirely
different)
A. 20_26_29
B. 21_25_32
C. 20_25_32
D. 21_25_31
87.A patient made for himself a complete denture. After a few days he
comes to you complaining from pain and white spots on the residual
ridge and you do relief in that area and give him ointment. After a few
days he comes again complaining the same but in another area. The
main cause is:
A. Uneven pressure on the crest of alveolar ridge.
B. Rough tissue contacting surface of denture
C. Increase vertical dimension
D. Absence of balancing occlusion
88.Female come with mass on left neck, slow growing before 6 years,
the first surgeon said it is a harmless sialodenitis, now CT scan show
mass on submandibular gland, your diagnosis:
A. sialodinitis
B. pleomorphic adenoma======
C. adenoid cystic carcinoma
89 What is the goal of maintenance therapy
A. To prevent recurrance of disease
B. Evaluate tissue response
90.upper 8 Impaction to avoid tear
of gingival flap A. Adequate size of
flap
B. Flap include greater palatine nerve
C. Strong retraction of flap margin
91.After bleaching, want to restore a tooth with composite. You
don't want to compromise bonding. How long should you wait?
A. 24hrs
B. A week
C. After 2 weeks
D. Another material for restoration
92.Increase in the chance of fracture of anterior teeth if?
A. Caries*
B. Protrusive anterior
C. Weak enamel
93.scammons curve of growth-which attains the highest 1st??
A.Neural
B.Genital
C.Lymphoid
94.During which period of intra uterine life the primary tooth bud is
formed:
A. 4weeks
B. 5weeks
C. 6 weeks
x
EXAM – 13

62­ caries detecting system used flurescence for recording teeth images
DIOFTI
1.vazirani akinosi technique ­trismus case
2.twice ianb given.failed.how to manage?­gow gates
3.lower lip malignancy which is most common?
a.Mec*** b.acinic cell ca c.acc d.low grade tumor
4.canine palatal impaction vs buccal impaction ratio
2:1***
5 .2mm plate drill hole size
1.5mm***
6.pic of missing 11; 12 has mesial caries .pt needs restoration of 11.no mobility, healthy perio,no 
other systemic disease. What is best diagnostic image for 12?
A.periapical** b.bitewing c.panorama d.occlusal
7.warfarin inr on day of surgery 
8.ideal amount flouride in water. 
0.5 to 0.8**
9.shade guide for cement during porcelain veneer restoration. Use cement base lighter then porcelain
10.trauma case, ant teeth move as one segment diagnosis? 
Alveolar bone#**
11.vanderwoude syndrome ­congenital lip pit**
12osteogenesis imperfecta 
13.prognosis after rct determined by 
Decrease in size of radioluecency ***
14.main component of inlay casting wax
Paraffin wax**
15.newly erupted (pic) teeth stained groove no catch best managed by?
A.Flouride n 6 month follow up **
B.sealant
C.PRR
D.composite
16.which one gives good glossy finish after restoration? 
Microfill composite**
17.best radiograph for implant 
CT , periapical, panoramic views,** MRI
18.technique of anesthesia in hemophilia pt 
Intraosseous
19. During upper 3rd molar extraction tooth pushed posteriorly and superiorly unable to visualise,
management? Admit Ct and extract under GA
20. Gingivectomy in nifidipine induced gingival enlargement, is done to,
Eliminate pseudo pocket 
21. Shape of two rooted maxillary premolar access cavity?
Oval
22.maxillary premolar extraction forceps  Universial 150

23.  Diabetic patient came to clinic with pain, swelling & enlarged mandible, on radiograph it 
showed moth eaten appearance, your diagnosis is:
a) Acute osteomyelitis.
b) Chronic suppurative osteomyelitis.***
c) Focal sclerosing osteomyelitis.
d) Diffuse sclerosing osteomyelitis. (cotton wool appearance).

237. Patient suffering from pain in the area of the mandibular molars with paresthesia 
( numbness ) inthe lower lip. By clinical and radiographic examination your diagnosis:
A) Acute osteomyelitis. ***
b­ossifyig fibroma 
c­osteosarcoma
 
24.fractured subgingival curette removed with
Schwartz periotriever
25.class 5 restoration excess removed using
A.Knife B.carver **
26.semi adjustable arcon articulator example 
A.dentatus
B.denar 5 a
C.denar mark 2**
D.gnathoscope
27.pseudo class 3 management? 
A.retraction of upper
B. Retraction of lower *
C. Lip bumper
D.
28.bone of nasal septum ­vomer**
29.mandible formed from?
Meckels cartilage **
30.pt with upper n lower denture having anterior ridge with white ulcerations came to dentist. 
Correction was done.patient was comfortable. Again came with some problem in another area. 
Cause for lacerations? 
Uneven pressure on ridge
31.pt.came for extraction on examination there's a white patch with ulceration on buccal 
mucosa.pt is tobacco chewer.best management? 
Referral to surgeon for biopsy 
32.nerve supply of palatal mucosa of maxillary first premolar? 
Greater palatine nerve

33. /Radiopacity attached to root of mandibular molar: 
a. Ossifying fibroma.
b. Hypercementosis. ***
c. Periapical cemental dysplasia.

2/ Radiopacity attached to root of mandibular molar (vital non­carious ):
1. Ossifying fibroma.
2. Hypercementosis.
3. Periapical cemental dysplasia.++

34.chronic maxillary sinusitis microbiology? 
A.predominantly aerobic
B. predominantly anaerobic 
C. Mixed aerobic n anaerobic**
D.80% anaerobic 20%aerobic

35.motor supply of scalp.
Temporal br of facial 

36.dis adv of full thickness flap.
papillary integrity

macrophages are found in:
a­ phase of scar formation 
b. late phase of inflammation **
c. early phase of inflammation
d. phase of repair

37.cyst surrounding impacted 3rd molar
Dentigerous cyst
38.structure that oppose retentive arm in rpd
Reciprocal arm
39.caries progrssion in old pt vs young patient
Generalised sclerosing by age
40.acute pericoronitis management?
A. removal of flap
B.removal of flap n antibiotic *
C.extration
D.
41. Management of fractured root between middle and apical 3rd with large gap between 
segment
1 rct of coronal with surgical removal of apical*
2 splint
3 rct of coronal and splint 

42.conservative method of tooth whitening? 
Bleaching 

43. The principal function of an indirect retainer is to
a) stabilize against lateral movement.
b) prevent settling of the major connector.
c) minimize movement of the base away from supporting tissue.**
d) restrict tissue ward movement of the distal extension base of the partial denture 

44.side of the bristle actively participate in?
Modified still man
45.overhanging restoration mainly affect? 
Periodontal health
46.child with lower canine abscess with no crowding best managed by?
Extract both canine without space maintainer 
47.trauma case.missing 11.fracture of 12 fracture of buccal plate .restoration using?
A. Acrylic rpd**
B.tooth supported fpd
C. conventional fpd
D.metallic rpd
48.biologic width­ base of the pocket to cej
49.wrought wire clasp advantage over cast wire clasp
Less irritation to abutment 
50.student taking xray of lower molar in female patient induces gag.what is the cause of gag 
reflex?
lingual nerve

51. Single bilateral edentulous area anterior to remaining natural teeth
Kennedy class 1 Class 4

52. Dis adv of akers clasp
caries
caries detecting system used flurescence for recording teeth images
A-diagnodent
53.difoti advantage  B.dofti*

Determine caries activity

54.perforation at root bifurcation. Material used?
MTA
55.mentally and physically disabled , pouching of food in cheek
Autism
56. In a denture lower 3rd molar set in occlusal plane. 
A.post 1/3rd of retro molar pad 
B. Ant2/3rd of retro molar pad **
C.class 1
D.class2
57.curing light 450nm wavelength Wat is ur decision? 
Working normal**

58.scarlet fever, white lesion.scraped.red bleeding area.diagnosis?
Candidiasis
59. after he quit smoking, minor aphtous ulcer appear, what is the cause:
A­ allergy
B­ dilation of blood vessel
c­ nicotine out of blood stream***
D ­ non all
60.2wall defect what is the best graft to treat this defect? 
Cancellous FDBA
61.sjogrens syndrome
EXAM – 15

1. best xray for proximal teeth ? bitewing 
2.7 years old pt came with fracture in subcondylar area. 
the other side have class 1 malocclusion , fracture side have class 2 malocclusion . manual 
reposition it shows class 1 . whats you tt?
 A. open reduction . 
b closed reduction. 
c . follow up .
3. instrument with number 15 . 83. 8. 14 . which is for length of balde ? 8
4 . 3 years under general anasthesia , xray show small caries proximal area D and E , tt ? 
A. composite 
B. amalgam 
C.stainless crown
D. pulpotomy with stainless
5.convergence of walls amalgam
6 . pt 17 years started forming open bite , no sucking finger habit . 
A. cleidocranial dysplasia 
B. eagle syndrom
C. treacher collins 
D .plummer vinson
7. pt had radiotherapy 4 years back . edentulous, need CD . 
which material to use for impression 
A. compound 
B. plaster paris
C.zoe *
8. standard and conventional gp difference
9. space between 2 implant ? 3mm
10. non resorbable suture , most used in oral surgery .
11. pic of fibroma
12. GP : 70 % ZOE 30% gp
13. why use low speed bur for temporary teeth ?
14. dentist romoves all caries in a tooth , than saw a small red point bleeding lighly . 
A . direct pulp capping 
B. indirect 
C. pulpotomy
15. Percentage of copper in amalgam which reduces gamma 2 phase?
16. crucial need for appliance to stop thumb socking 
A. temporary dentition
B. early mixed dentition 
C. late mixed 
D. permanent dentition
17. 
diabetic patient suffering 15 days came with swelling , erythema , pain in the area of right 
mandibular molars ( no mobility) . xray ; moth eaten appearance . diagnosis ? 
which osteomyelitis? Acute osteomyelitis
18. oxygen flow rate per minute . 5-9 L/min
19. 7 years old , intruded max ant teeth after fall . tt
20. Pt have calculus and u want remove using device have elleptical motion .. Which is it...?
A.ultrasonic
B.piezo
C.hand instrument
D.magnetic
21. Brush method with side parts of bristle activated ? Stillman
22. During 3/4th crown preparation on pm bur used to add retentive grooves Taperd fissure
23. character of irreversible pulpitis ?
24. Most potent vasodilator?
a.cocaine
b.procaine
c.tetracaine
d.atricaine
25. The substance in local anaesthesia cartridge responsible for prevent oxidation of 
vasoconstrictor: A.sodium chloride solution B.sodium metasalphate C.sodium salphate
26. bionator
27.When you give sedative inhalation for patient to prevent hypoxia u give :
A­95% oxygen and 5% nitrouse oxide 
B­90% oxygen and 10%nitrous oxide 
C­85%oxygen and 15%nitrous oxide 
D­100% oxygen and zero nitrous oxide
28.patient came to hospital with gun shot ,,the surgeon will make fixation by : a­ christian's 
technique b­ keen's technique
29. Pt. got gun shot , question about graft used for condyle ?
Poaterir ilic
Siliography
30. best xray for parotid
31. swelling when eating . sialothitis
32. second best xray for planning and fixing implant Mri
33. 2 layers of varnish under amalgam .
34.class 1 malocclusion , the vertical dimension
A. 1 mm less in rest position 
B. 5 mm
C. 6mm
34. which part of periodontal tissue regenerate the last ? Cementum

EXAM – 16
. pt. with white spot on his tooth and tooth not cavitated tell ptt to check up after?
A. 3 months
B. 4­6 months***
C. 7­9 months
D. 11­12 months
B????
2.Distance between patient and cephalometric
5 feet *****
3.patient with anaphylactic shock due to pencillin 
1­ 0.5­1epinephren of 1\10000 Im
2­ 0.5­1 1\1000 adrenalin im ******
3­ 200 mg hydrocortisone intravenous
4­ Other option not related
4.patient came to clinic with wrinkled skin and white shiny hair, with pegged laterals teeth : 
A) ectodermal dysplasia******
B) cleidocranial dysplasia 
C) pteygz jogheurs syndrome
D) Gardner syndrome
5.Which material used in one­visit pulpectomy:
a. MTA (Mineral Trioxide Aggregate)***
b. Ca (OH) 2 + CMCP
C. Formocresol
D. Zn oxide
6.Critical ph of saliva at which demineralization of enamel begins :
A­5­­5.5 ***
B­4­­­4.5
C­6­­­6.5
7. best describes caries progression?
a. Dynamic process with demineralization and remineralization***
Other 2 option I can’t recall
8.Color choice for cervical third for full coverage
A­highest chroma****
B­thick enamel
C­highest value
D­lowest hue
9.The concentration of household bleach is:
5.25%***
10.If tooth or root is pushed during surgical extraction into max. sinus:
a. Leave it and inform the Pt.
b. Remove it as soon as possible. ***
11.Post graduated student usesMTA the prognosis depends on prevent:
a. immediate suture.
b. disturbance during closure of wound. *** 
c. using a flab
12.8 yrs old pt has lower primary canine space infection what is the ttt?
1. Extract one canine without space maintainer
2. Extarct canine and band and loop
3. Extract both canines without space maintainer**
4. Extract booth canines with lingual arch holding bar
13.to make v shaped groove in canine what bur should be used according to some standard:
a)round
b)tapered
c)fissure
d)inverted cone**
14. Patient came to clinic with wrinkled skin and white shiny hair, with pegged laterals teeth : 
A) ectodermal dysplasia****
B) cleidocranial dysplasia 
C) pteygz jogheurs syndrome
D) Gardner syndrome
15.Saturation :
A chroma***
B hue
C value
16.Dentinogenesis imperfecta occurs in which stage.
A.histodifferntiation***
B.morphodiffrentiation
c.apposition
17.After ortho pain upper canine , x ray resorption canine root
a. Apply CaoH at the site of resorption***
b. Do RCT in a single visit.
c. Extract the tooth & reimplant it.
d. Extract the tooth & do implantation.
18. Pt. come with severe pain, no response to pulp test when you do percussion the patient 
jump, diagnosis is: (No periapical change in radiograph) 
1.symptomatic reversible pulpitis .
2. Symptomatic irreversible pulpitis .
3. Asymptomatic apical periodontitis and asymptomatic irreversible pulpitis. **
19.hiv posatavaly test 
a. elisa *****
b. westron blot
Other 2 option I forgot
20.Access opening of lower canine is 
Triangle 
Square
Ovale*****
Trapizoidal
21.water irrigation device:
1. Prevent plaque formation
2. Completely removes plaque
3. Dilute bacterial toxins***
22.By radiometry u found the halogen light cure is 450 what u will do
a. change battery
b. change the lamb
c. don't make Any change.****
23. pt with missing 4 ant teeth and need fpd what abutment you choose
a. 2 canines
b. right canine and left canine and premolar
c. left canine and right canine and premolar
d. 2canines and 2 premolars**
24.A tooth with fracture cusp dentine involves what is the status of pulp in this case?
a. un inflamed healthy pulp**
b reversibly inflamed
c. irreversibly inflamed
d. innervated with A delta fiber
25.best material for impression of flappy ridge :
a­ plaster of Paris*****
b­ ZnOE
c­ agar agar
d­ compound
26.Active ingredient of Hemodent:
a. Ferric sulphate
b. Zinc phosphate
c. aluminum chloride****
d. ferric chloride
27.the CBCT have the following property 
a­ best to show TMJ disk***
b­ expose the patient to large amount of x­ray 
c­ use for routine radiographic examination
28.access cavity for lower 2nd molar:
1­triangular with base towards mesial******
2­triangular with base towards distal
3­oval with base towards mesial 
4­oval with base towards distal
29.A root seldom has 2 pulp canals
A. mesiobuccal root of upper molar
B. mesiobuccal of lower molar
C. distobuccal root of upper molar******
30.The organism that rarely found in newborn mouth:
a. Streptococcus mutant. ***
b. Streptococcus salivarius. 
c. Spirochaeta.
d. e­coil.
e. Skin bacteria.
31. Fluoride water supplementary and we want to give systemic fluroide what to ask pt
A.Age*****
32. The common concentration of Fluoride in over the counter dentifrices in ppm is?
1000 ppm****
33. Cavity etching before applying GIC is:
1. Polyacrylic acid 10 seconds.****
2. Polyacrylic acid 60 seconds.
3. Phosphoric acid 10 seconds.
4. Phosphoric acid 60 seconds
34. Patient had bulimia and had lesion in palatal surface in upper teeth with recurrent vomiting. 
What is the type of lesion
1. attrition
2. abrasion
3. erosion*****
35. according to Kennedy class of bilateraly single edentulous space found anteriorly
class 4 ****
36. treatment oral and pharynx candida. 
Fluconazol***
37.Which transformation of oral mucosa should be taken seriously
dysplasia***
Metaplasia
hyperplasia
neoplasia

38. CATAR:
1.One wall defect
2. Two***
3.Three
39. PT. Who has iron deficiency anemia difficulty in swallowing with examination of barium 
sulphate. 
A. Geographic tongue. 
B. Burning mouth syndrome.
C.plummer vinson syndrome. ****
D.diabetic patient.
40. time in days to established gingivitis
14­21 days****
41. colour of complex seen in a 20 year old healthy periodontium
Black complex
Purple complex******
orange complex
red complex
42.Long case of removal of epilis fissuratum. What forcep you use
­ stillies forcep
­Adson forcep
­Allis forcep******
43.Best implant material
Titanium*****
Platinum
palladium
45.Brushing technique Active side of bristles 
Modified stillman****
Bass
Charters
46. Needle for vazirani akinosi technique
25 gauge long needle***
47. Continuous condensation technique in gp filling is: 
a. obtura I.
b. obtura II.
c. ultrafill.
d. System B.*****

EXAM – 17

. After infection with chicken pox, Isolation period should be:
a. When fever subsides
b. After one week
c. When the vesicles are crusted****
d. Until carter stage is last
2. Primary role of epinephrine of local anesthesia during apical surgery: 
a. Reduce systemic absorption so reduce toxicity****
b. Increase duration
c. Two other options were about alpha & beta receptors, I can’t remember
3. Streptococcus Mutans affects which tooth surfaces:
a. pits and fissures**** 
b. smooth surfaces
c. root surface
4. Most common intracanal medicament:
a. CaOH****
b. Formocresol
5. Angle of blade while scaling:
a. 45­90****
b. 70­80
c. 100­110
6. To disinfect impression material, use:
a. Formaldehyde
b. Acetaldehyde
c. Gluteraldehyde****
7. Amalgam free of gamma 2 is:
a. 2% cu
b. 5% cu
c. 10% cu
d. 13% cu****
8. What's most difficult in impaction surgery
a. Mesio angular
b. Disto angular****
c. Horizontal
d. Vertical
9. After removal of impacted 3rd lower molar, pt complaining from parasthesia, why?
a. Irritation of the mandibular nerve during extraction**** 
b. Broke mandible
c. Hemorrhage from the socket
10. The instrument use in suturing of 3rd molar:
a. Stillis forceps****
b. Addison forceps
11. Brushing technique Active side of bristles 
a. Modified stillman****
b. Bass
c. Charters
12. Which material is used in one­visit pulpectomy:
a. MTA (Mineral Trioxide Aggregate)****
b. Ca (OH) 2 + CMCP
c. Formocresol
d. Zn oxide
13. Patient had bulimia and had lesion in palatal surface in upper teeth with recurrent vomiting. 
What is the type of lesion:
a. Attrition
b. Abrasion
c. Erosion****
14. Conventional GI cement has an advantage comparing to other GI types:
a. Fast Setting 
b. Shelf life
c. Strength
d. F concentration**
15. The three length of file and reamers that you work by them:
a. 20_26_29
b. 21_25_32
c. 20_25_32
d. 21_25_31****
16. Mucocele is treated by :
a. Marsupalization 
b. Excision****
17. Restoration that is contraindicated in mouth breather pt: 
a. GIC****
b. Compomer
c. Direct composite restoration
d. Indirect composite restoration
18. Amalgam used in retrograde filling is:
a. High copper 
b. Zinc free****
c. Mercury free
d. Antibiotic treated
19. Pt came to clinic with avulsed central incisor, 30 min ago, stored in milk, PDL was good, 
splint for how many weeks:
a. One
b. Two****
c. Three
d. Four
20. Proxy brush is used with which type of embrasure:
a. Type 1
b. Type 2****
c. Type 3
d. Type 4
21. After GTR (guided tissue regeneration) what type of dressing should be used:
a. Eugenol dressing
b. Non eugenol dressing****
c. Antibiotic dressing
22. First sign to show if there's lidocaine toxicity:
a. Bradycardia
b. Tachycardia****
c. Cardiac fibrillation
23. A disease occur in 350 persons out of 1000, the prevalence of the disease in 7 years will be:
a. 35
b. 50
c. 380
d. 2450****
24. PDI classification for missing teeth in upper and lower arch including canines:
a. cl1
b. cl2****
c. cl3
d. cl4
25. Dental material classification:
a. Ceramics, polymers and composite
b. Ceramics, polymers and alginate.
c. Ceramics, polymers and cement
d. Ceramics, metals, polymers, composites****
26. Bacteria for infective endocarditic:
a. S. aureus ****
b. S. virridans
c. Actinomyces
27. Most used as anticariogenic: 
Xylotol****
28. Arrange the steps of cleft palate management: 
a. Measures to adjust speech. 
b. Establish way for nursing and feeding. 
c. Cosmetic closure. 
d. Prevent collapse of two halves.
I forgot the options, but I think the order is B < D < A < C
29. First line treatment of 3 years old child with oral candidiasis:
a. Nystatin oral suspension****
b. Fluconazole systemic
c. Ketoconazole systemic
30. Serial extraction for rapid eruption in patient 11 years:
a. Mandibular permanent first premolar
b. Mandibular permanent second premolar
c. Deciduous canine***
d. Deciduous first premolar
31. (Pic of floating teeth) pt with mental confusion, renal calculi, and high alkaline phosphatase:
a. Hyperthyroidism
b. Hyperparathyroidism****
c. Hypoparathyroidism
d. Hypothyroidism
32. Pt has facial asymmetry, what type of x­ray used:
a. Anterior posterior object
b. Orthotomograthy
c. CBCT**
33. Which is easiest to stop gingival bleeding in class II cavity preparation:
a. Electrical cautery
b. Retraction cord after matrix band
c. Pressure with wet cotton pallet****
34. Local anesthesia technique used to block the buccal, lingual and mylohyoid nerves:
a. Gow­Gates Technique****
b. Vazirani­Akinosi Technique
c. Coronoid approach
d. IAN Block
35. Why acrylic teeth are used NOT porcelain in dentures:
a. Acrylic resist staining more
b. Acrylic bond to denture base more ****
c. Acrylic have high compressive strength
d. Acrylic resist wearing more than porcelain
36. 25 years old pregnant patient has bleeding on probing on papilla of anterior area of the 
maxilla:
a. Giant cell granuloma
b. Pyogenic granuloma****
37. Bacterial spores used as a test for autoclave, it is considered:
a. Chemical test
b. Physical test
c. Biological test****
d. Sterilization test
38. What is not a denture surface?
a. Polished surface
b. Vestibular surface****
c. Occlusal surface
d. Impression surface
39. Pt 9 years old, Apexogenesis is done to his upper central incisor, when to judge that the 
treatment done is successful:
a. Root complete development
b. Tooth is asymptomatic****
c. Tooth responds normally to vitality test
d. Inflammation confined to the coronal pulp
40. Pt has necrotic pulp in upper central incisor, not closed apex, what is the best treatment: 
a. Calcium hydroxide 
b. Apexification with GP filling 
c. Calcific barrier ****
d. RCT + GP filling
41. Most suitable to describe NITI:
a. Rigidity
b. Low coefficient of friction
c. Shape memory****
42. Bacteria for initiation and progression of caries: << Streptococcus and lactubacillus
43. Advantage of GIC: << Release fluoride
44. Bacteria cause endo failure: << E. feacalis
45. Pterygopalatine ganglion is anatomically related to which nerve: 
a. Maxillary nerve ****
b. Facial nerve
46. Which fluoride can be applied professionally by dentist:
a. Act
b. Duraphat****
c. Gel tin
d. Prevident
47. .Interproximal bone parallel to:
a. Gingival margin
b. CEJ***
c. PDL
d. DEJ
48. Extraction after dialysis
a. 1 hr after hemo dialysis
b. 1 day before ****
c. 1 day after
d. 1 week after
49. Type of cementum in coronal 2/3rd:
a. Acellular afibrillar
b. Acellular extrinsic fibre****
c. Acellular intrinsic fibers
d. Cellular mixed stratified
50. Difference between PRR and class I filling:
a. Caries extension****
b. Caries depth
51. Best location and size of root perforation that is favorable:
a. Small perforation below height of bone crest
b. Large perforation below height of bone crest
c. Small perforation at height of bone crest****
d. Large perforation at height of bone crest
52. Pt complaining from bleeding while brushing, on examination, swollen red gingival, shiny and 
loss of stibling, bleeding on probing with 3­4mm probing depth without attachment lost:
a. Acute gingivitis****
b. Chronic gingivitis
c. Periodontitis
d. ANUG
53. The access opening for upper central and lateral incisor is:
a. Triangular****
b. Rectangular
c. Square
d. Oval
54. The most used material for home bleaching: 
a. Hydrogen peroxide****
b. Sodium perborate
55. Food low cariogenic potential, the following should be characteristic 
a. Low buffering capacity 
b. Ph higher than 3
c. Contain mineral**** 
d. Contain protein
56. Pt with denture complains of poor retention, when you press on palate you see bubbles in 
posterior extension due to:
a. Over extension
b. Over post damming
c. Under post damming****
d. Under extension of post dam
57. When to select the composite shade for restoration:
a. Before rubber dam placement****
b. After cavity
c. After bonding
58. Sharpening of curette, you put its cutting edge to stone at which angle:
a. 70­80
b. 80­90 
c. 60­70
d. 100­ 110****
59. Mechanism of mandible growth:
a. Apposition and intra membranous modeling
b. Interstitial and endochondral
c. Apposition and endochondral ****
d. Interstitial and intra membranous
60. Which deciduous tooth cause crowding in lower anterior region if early extracted:
a. Primary mandibular first molar 
b. Primary mandibular second molar**** 
c. Primary maxillary first molar
d. Primary maxillary second molar
61. Pt. needs complete denture, you take impression with irreversible hydrocolloid (alginate) & 
poured it after more than 15 min. the cast appears soft & chalky the reason is:
a. Dehydration of the impression****
b. Expansion of the impression
c. Immerse the impression in disinfectant
62. Measuring pulp vitality:
a. Heat
b. Ozone O3
c. Carbon dioxide
d. Laser Doppler**
63. Dye that is used with toludene blue to differentiate between cancer and normal cells
a. Methylene blue
b. Congo red
c. Lugol****
64. Pt with painless, firm, bilateral enlargement on lower mandibular region, on x­ray, multilocular
radiolucency with displacement of the lower 2nd molar. What is the diagnosis:
a. Crhon’s Disease
b. Behçet's Disease
c. Odontogenic Keratocyst
d. Cherubism**

65. Pt came to the clinic after 3 days of composite restoration was done, complaining from color 
was changed and not matching the adjacent teeth. What you will do:
a. NO treatment
b. Add layer of composite
c. Apply bonding agent 
d. Resurfacing**
66. Success rate of implants
a. 85% to 94%
b. 95%
c. more than 95%**
67. Pic of teeth with erosion
68. Pic of patient wearing CD, what was the problem?.. I chose mid line shift
69. Pic of intraoral lesion, the options were:
a. Fibroma
b. Epilus Fissuratum
c. Flappy Ridge
d. Papillary Hyperplasia
70. Q about mesio­buccal cusp crack in upper molar, what’s your management?.. Sorry I forgot 
the options
71. Q about how new technology/methods helped in detecting caries? Diagnoden
72. Pt came to clinic with multiple missing (non­erupted) permanent teeth, what type of x­ray you 
will take?
EXAM – 18

1.L.A. given to pregnant lady.
a)procaine
b)Prilocaine
C) lidocaine ***
2.pic of facebow.type of facebow
a)condyle
b)ear
C)kinetimic*****
3.dental floss used:
a)remove calculus
b) remove overhang
c)disrupt the plaque
4.During working on a patient you noticed a furcation perforation,what is the best material to 
manage this;
a)calcium hydroxide
b)MTA****
5.Length of the needle when injected to the mucosa in Inferior Alveolar block:
a)two­third of the needle****
b)one­third of the needle
6.Technique will you use to anesthetize soft and hard tissuse of mandibular with one injection:
a)Inferior Alveolar nerve block
b)Gowgates*****
c)vazirani akinosi 
7.pic supernumerary teeth
8. After finish class V glass ionomer cement we do finishing with:
a) Pumice slurry.
b)Aluminum­oxide disc.*****
c)Prophy paste with fluoride
9.In acid etching for fissure sealant:
a)Longer time for permanent teeth
b)Longer time for primary teeth****
c)Same for both primary and permanent
d)Longer in amelogensis imperfect
10.Hemophilia A child has tooth 58 resorbed but distal root not premolar tooth cusp tip has been 
showing patient has discomfort from tooth 58.what is the treatment:
a)extraction
b)no intervation****
c)pulpotomy
11.What is the copper ratio that eliminates gamma phase 2:
a)13%****
b)10%
c)2%
12. Cavity etching before applying GIC is :
a) Phosphoric acid 10 seconds.
b ) Polyacrylic acid 60 seconds.
c ) Polyacrylic acid 10 seconds.********
d ) Phosphoric acid 60 seconds.
13.Patient with missing upper 12 and 21.what kennedy classification:
a)class 1 modification 1
b)class 2 modification 1
c)class 3 modification 1
d)class 4 modifiction 1
14.When all the teeth is missing except the two canine,according to kennedy classification is:
a)class 1
b)class 2
c)class 3****
15.PDI classification for missing teeth in upper and lower arch including canines:
a)class 1
b)class 2*****
c)class 3
d)class 4
16.During composite filling MO in premolar there was overhang restoration to avoid this:
a)semi sectional matric with wedge*****
b) full tofflemire matric with wedge
c)celluloid matric
17. Caries susceptible surface in a high caries patient
a)Facial surfaces of maxillary posteriors****
b)Facial surfaces of max anteriors
c)Palatal surfaces of max posteriors
d)Palatal surfaces of max anteriors
18.In recent DIAGNODENT is used for detection for specifically diagnosis that cannot be done by
another device(something like this):
a)proximal
b)deep
c)superifical
d)bacteria invasion
19. ­Material used in oral cavity binds to all surface and has ability to release ions and decrease 
microbial effect:
a)Flouride***
b) chlorhexidine mouthwash
20.an old patient came to clinic completely edentulous need to make complete denture what is 
xray of choice:
a)bitewing
b)periapical
c)panomaric radiograph****
d)CBCT
21.to make v shaped groove in canine what bur should be used according to some standard:
a)round
b)tapered
c)fissure
d)inverted cone
22.sensory innveration to orofacial:
a)facial N
b)occulumotor N
c)abducens N
d)trigeminal N
23. While performing cranial nerve examination you notice that the patient is unable to raise his 
eyebrows, hold eyelids closed, symmetrically smile or evert his lower lip. This may indicate:
a. Trigeminal nerve problem.
b. Facial nerve problem.****
c. Oculomotor nerve problem.
d. Trochlear nerve problem.
24.Desired resorption in ortho tooth movement:
a)undermine resorption
b)alveolar resorption
c)hyalination
d)facial resorption
25. pt. came to dental clinic for extraction.after extraction bleeding profusely when check 
haemological problem in lab test they found that factor VIII ( 8 ) is less 10 % what’s the 
diagnosis: 
a­ Hemophilia A.****
b­ Hemophilia B.
26. 12 yr old child bleed easily with minor trauma have high bleeding sight elevated time 
coagulation and fragile capillary he suspected to have
a)Hemophilia 
b)Thrombocytopenia**** 
c)Vit.k. deficiency
27. Female come with mass on left neck, slow growing before 6 years, the first surgeon said it is 
a harmless sialodenitis, now CT scan show mass on submandibular gland, your diagnosis:
A. sialodinitis
B. pleomorphic adenoma****
C. adenoid cystic carcinoma.
28.incidence infected with ceratin disease 350 in 1000 population.what will be prevalence after 
seven years:
a)35
b)50
c)350
d)2,450***
29. Any attribute, characteristic or exposure of an individual that increases the likelihood of 
developing a disease or injury.
a)confounding factor 
b)prognostic 
c)etiological 
d)risk factor***
30. A female patient came to your clinic with dry lips and mouth and bilateral submandibular 
oedema and ocular dryness. Diagnosis is:
a) Polymorphic adenoma.
b) sialotitis
c.)salivary stone
d) Sjogren's syndrome****
31. During Extraction tooth fall in patient mouth retrieve it with 
a) college pliers 
b) Russian forceps****
32. pituitary gland: growth hormone
33. Separate the tooth from the middle of molar and preserve the tooth as two half premolar this 
procedure called:
A­tooth Hemisection 
B­ tooth Bisection****
34.tooth has radicular cyst what will be the state of pulp:
a)vital tooth
b)caries tooth
c)non­vital tooth******
35.few week after FPD pt complain of bad odour no pain what could be the cause:
a)loosing retainer
b)open margin
c)food accumulation in between retainer
36.The important property of cement under amalgam : 
a) High modulus of elasticity ***
b) Low modulus of elasticity 
c) modulus of elasticity not important 
d) High modulus of elasticity and low tensile strength
37. With long span FPD, unit should have:
a) Low strength
b) Low rigidity
C) High strength
D) High compressive and high rigidity***
38. old patient has discomfort in premolar , in x­ray there is abrupt (sudden) midway canal 
dissapear , why :
a) secondory dentin apically 
b) hypertropic calcification in apical part 
c) bifurcation apical***
d) disorganised mass at middle
39.minimum dose of adrenaline for pt with anti­depressant drug:
a)0.01­0.02****
b)0.02­0.03
c)0.03­0.04
40.remants of rest of serres:
a)dental lamina*****
b)hertwiz sheet
c)vestibular lamina
41.water is interfered during direct restoration what will happen:
a)soft resin
b)decrease space for restoration
c)change colour
d)reduce streghth of restoration
42. smear layer benefit in protecting pulp through
a)prevent toxins to reach pulp through dentinal tubules****
b)decrease acid etch effect on pulp
43.h file superior than k file:
a)positive rank angle****
b)negative rank angle
c)more flutes
d)more in diameter
44.disinfectant used in home as home bleaching:
a)NAOCL****
b)lactic acid
c)alcohol
45.nine year old child extraction of maxillary right maxillary 1st molar and left maxillary 2nd molar
what space maintainer can be used:
a)nance
b)transeptal
c)hawleys applicance with thin wire
d)no treatment, reassurances
46.most retention of palatal form when applied with retentive agent:
a)u shaped****
b)v shaped
c)flat shaped
47. elderly patient with flat ridge & uncontrolled movement, best teeth to use are:
a) 0 % cusp angulations "flat teeth" ***
b)10% 
c)20 % 
d)30%
48. Caries progression in adult less than child:
a)Difference in PH.
b) Generalized dentine sclerosis by age***
c) Increasing in organic content of tubular dentine by age
49.no caries on maxillary tooth will pressure pain radiates to ear no radiographic finding:
Maxillary sinus
50.minimum space fo erupting of primary teeth for permanent teeth:
a)3mm
b)4mm
c)6mm***
d)8mm
51. aim of most periodontal surgeries:
a)Provide direct access and visualization to diseased root ***
b)Eliminate pockets
c)Lengthening gingiva
52.shape of rest seat in RPD: 
a)spoon shape and square
b)concave****
c)convex
d)all
53.access opening for mandibular 1st molar:
a)rhomboid
b)base to buccal and triangle shape
c)trapezoid
54.base of the caries:
a)dentin
b)pulp
c)enamel
c)DEJ***
55.pic after orthodontic treatment there are white spots what is not in option for treatment:
a)veneer
b)full crown****
c)parital veener
d)composite
56.minimum conservative whitening of vital teeth:
a)bleaching
b)microbrush****
c)veener
57.avulsed tooth best medium:
a)saliva
b)water
c)HBSS*****
58. Gingivectomy contraindication in :
a)massive alveolar bone surgery ***
b) Gingival inflammation
c)gingival abscess
59. the L.A depends on
a)strength bond between drug and nerve*** 
b) strength bond between drug and it's intensity
c) bind between drug and time of removal from body
60. cancer related to gardners syndrome in which organ:
a.colon***
b.lung
c.heart
d.pancreas
61. Caries detection dye composed mainly by
a)Acid fuschin
b)Basic fuschin
c)Propylene glycol****
62. Remove thick epulis fissuratum:
a) Allis forcep***
b) Addison forcep
c) Curved hemostat
d) Stilli forcep
63. Internal resorption in maxillary central incisor can be treated with 
a)Ca(OH)2***
b) Gutta percha single sitting 
c) ZOE 
d) Silver cone
64. 2% taper of endo files means:
a) the difference from tip to D16. ***
b) 0.02 mm increase from the tip to the handle.
65. At the begining of the operation day in the clinic, you should start the 
water/air spray for three minutes in order to get rid of which type of 
microorganisms :
a) Streptococcus mutans.
b) Staphylococcus 
c)pseudomonas aurignossa****
66. The subgingival scaler to be specific area:
a) universal
b) The head should be 90% with shank
c)gracey curette****
67.function of rest in RPD:
a)support*****
b)retention
c)stability
68.antibotics prescribed for:
a)diffuse swelling***
b)periapiacal cyst
69.drugs cause ginigival enlargement:
a)phenyntoin****
b)cyclosporine
c)calcium channel blockers
70.Aker’s clasp used in RPD causes:
a)caries***
b)mobile
c)ginigival recession
71.before starting endo treatment what test should be done:
a)cold test*****
b)hot test
c)percussion
d)electric test
72.advantage of indirect over direct restoration:
a)occlusal interference
b)marginal adaptation***
c)high metal strength
73.onset of herpes simplex viruse without any treatment:
a)average 9­11 years******
b)above 10years
c)11­15 years

74.minimum time of topical LA applied:
a)30min
b)1min****
c)2min
d)3min
75. Theoretically which Xray technique gives exact length of root canal :
a)Bisecting angle 
b)PAralleling technique****
76.needle used for IAN block:
a)25gauge long needle******
b)25gauge short needle
77.young patient with multiple caries high plaque index erythema and edematous 
ginigiva,ginigiva pocket 2­4.diagnosis:
a)ginigivitis****
b)junvile periodontitis
c)periodontitis

EXAM – 19

*Iron deficiency anemia type : Microlytic 
*Patient, with amalgam containing zinc restoration for a simple class one cavity, arriving your 
clinical with pain after one month
1. Occlusal
2. Pulpal involvement
3. Delayed expansion **
*Raspberry on the palate : hyperplasia
*Loss of gag reflex : IX 
*he most accurate impression material for making the impression of an onlay cavity: 
a. Impression compound.
b. Condensation type silicone.
c. Polyvinyl siloxane ***
d. Polysulfide.
*  For Hepatitis B only Antigen s HB is to check:
Immunity((IF they said antibody (immunity))
Acute Infection** Hbs acute
Chronic Infection Hbc chronic
** Digital fiberoptic transillumination is for:
a. All surfaces**
b. Proximal
c. Pitsand fissure
d. Smooth surface 
Radiology pix of : odontoma, compound odontoma, ameloblastoma 
Disease that no need for special care and not contagious : chickenpox ( rest options were Hp c 
and congectivitis and measles ) 
Patient with missing pri 1st molar on the left and missing canine and 1st molar on the right 
( lingual arch ) 
* Where does the needle holder hold the needle during suturing?­75
a­ at the end of the needle 
b­ at 1/3 the needle from the tip 
c­ at 2/3 of the needle from the end
d­ at 1/3 of the needle from the end ***
Restoration of p&f sealant : flowable
Questions am not sure abt the answers
Which disease of special care patient lead to periodontal destruction ( i answered with down 
syndrome ) 
Space needed for eruption of pri teeth with no crowding ( 3­4­6***­8)
Trauma Intrusion of pri centrals and effected permanent buds Rx ( carful exo**­ observation­ 
ectomy)

EXAM – 20
When you give a child a gift for a good behavior this is called:
A.Positive reinforcement.**********************
B.Negative reinforcement.
2­Optima water fluoridation:
a. 0.5­0.8 mg\liter.********
b. 0.2­0.5 mg\liter.
c. 2­3 mg\liter
d. 1­5 mg\liter
3­dye that is used with toludene blue to differentiate between cancer and normal cells ( shaheen)
a. methylene blue
b. congo red
c.lugol***********
4­For recording of vertical dimention we use:
A.Willis Gauge.*****************
B.caliper.
C.Face bow
5­Avulsed tooth:
A.splint 1­2 weeks. **********************************************
B.splint 2­3weeks.
C.splint 3­4weeks.
6­bizygomatic width measured by:
A.gothic arch
B.facebow********************************
C.willis gauge
7­best feature of sealant?
A.viscosity
B.retention**********************
(sealant(pedo) >>> retention and sealer(endo) >>>viscosity)
8­Patient with hypertension with 140/100 and he has been using medicine for the past 10 years 
he wants to make a denture , what do you see ?
a.mucosal change
b.dry mouth*******************
c.gingival enlargement
d.fungal infection.
9­Dentin contains which type of collagen fiber ( shaheen )
a.Type I***************
b.Type II
c.Type III
d.Type IV
10­What is the copper ratio that eliminates gamma phase 2:
a. 2% copper
b. 4% copper
c. 10 % copper 
d. 13 % copper********
11­Difference between amelogenesis imperfecta and dentinogenesis imperfect
a.heridiatery factor.
b.brown color of enamel.
c.pulp champer and root canals.****************************
12­The test for testing the bur in which all the blades of the burs pass through 1 point called:
a. Run out.***********************************
b. Concentricity.
c. Run out and concentricity.
d. None of above.
13­patient has a symphysis area and need maxillofacial surgery what is the wire
a. 4 feet 8 gauge
b. 4 feet 28 gauge
c. 6 feet 20 gauge
d. 6 feet 26 gauge***********************************************
14­To check a perforation in the desk of the TMJ. we need: Cranial imagery.
A.Arthrography."*************************
B.Traditional tomography.
C.Computerized tomography.
15­Oral herpes caused by which virus: ( corrected file )
a­herpes simplex type 1*********************
b­ herpes simplex type 2
16­Condyle developed by :
A.Intramembranous ossification.
B.apposition and Endochondral ossification.*************
17­60 year old patient came to the clinic complaining of excessive movement of denture. On 
examination there is elevation of anterior end when u press o the distal end. TTT ( shaheeen )
A. Reline****************
B. Rebase
C. Remake
D. Denture adhesive
18­Question about Cemental dysplasia. ( shaheeen ) Clinical scenario.( how to know ) the only 
case with : Female of African­American with a radiolucency and the mandibular of the anterior 
teeth , teeth are vital )
19­Water irrigation device
A.prevent plaque formation
B.completely removes plaque
C.dilute bactetial toxins************
20­intracanal medication
a.calicum hydroxide ************************
21­Amount of daily wear of amalgam ingested in the body:
1 – 3 μgs/day of mercury.*********************
10 – 15 μgs /day of mercury.
25 μgs /day of mercury.
22­Patient comes to your clinic complaining that the denture become tight, during examination 
you notice nothing, but when the patient stand you notice that his legs are bowing (curved). What
you suspect:
a.Paget’s disease.
23­Case scenario on ( Central giant­cell granuloma ) benign tumer affect women and is more in 
the mandible , Micrograph of a central giant cell granuloma showing the characteristic giant cells 
with surrounding cells that have nuclei that are dissimilar to those in the giant cells.

24­picture of bluish swollen glazed lesion on tip of tongue and laterally in 25 years’ female:
a­ Sq. Cell
b­ Hemangioma*************
c­ Neuro fibroma
d­ Lipoma
25­Needle used for aspiration cytology
19 gauge.
26­flouride which we use in clinic doesn’t cause fluorosis;
a­teeth already calcified
27­65 year old patient complains of pain sharp and lingering in Buccal mucosa. It goes away and
comes back spontaneously at night , stimulated with cold (when going outside ), pt is 
edentulous ,has had extractions in the past. Pain is on one side
of face and does not cross the lip line:
A.trigeminal neuralgia **************
B.Bell's palsy
C.myofacial pain.
28­Porcelain shrinkage after firing:
A.1­5 %.
B.5­10 %.
C.10­20 %.***********
29­Instrument which use for grasping a tissue when remove thick epulis figuratum:
A.Allis forceps.*************
B.Adson forcep.
C.Curved hemostat.
D.Stilli forceps
30­After finish class v glass ionomer cement we do finishing with:
a.Pumice slurry.
b.Aluminum­oxide disc.**********************
31­oval radiolucency between roots of upper central incisor
A.Incisive foramen**************
B.Radicular cyst
C.Granuloma
D.Absess

32­During mentoplasty/ genioplasty , doctor should take care for injury of what nerve:
A.Lower branch of the facial nerve Mental nerve

33­After trauma a tooth becomes yellowish in color, this is due to:
A.Necrotic pulp.
B.Irreversible pulpitis.
C.Pulp is partially or completely obliterated.******************
D.Hemorrhage in the pulp.
34­Treatment of internal resorption involves:
A.Complete extirpation of the pulp to arrest the resorption process.**********************
B.Enlarging the canal apical to the resorbed area for better access.
C.Utilizing a silver cone and sealer to fill the irregularities in the resorbed area.
D.Filling the canal and defect with amalgam.
E.Sealing sodium hypochlorite in the canal to remove the inflammatory tissue necrotic in the area
of the resorption.
35­xray give real root length:
Parallel tech. *********
Bisecting tech.
36­The tip of size 20 endo file is:
0.02 mm.
0.2 mm.************
37­why calculus should be removed in perio diseases. ( shaheen )
A. To avoid plaque adherence *********************
B. Because its an etiological factor of periodontal disease
C.it irritates tissues
D.to maintain good oral hygiene

38­How will you asses oral hygiene / efficiency of oral hygiene methods prescribed to the 
patient? ( shaheen)
a. Plaque index****
b. Calculus index
39­Premature contact between upper and lower ant. teeth in eccentric occlusion while there is 
absolutely no contact on the centric occlusion. So the management is by grinding of:
A.incisal edge of ant. max. teeth.
B.Incisal edge of ant. man. teeth.
C.Inclination of ant. max. teeth lingual.****************
D.Inclination of ant man teeth.
40­Patient presented to you after fitting the immediate denture 5 – 10 months, complaining pain 
and over tissue in the mandibular, what is the diagnosis:
A.Epulis fissurment.**********
B.Hypertrophic frenum.
41­Biological width after crown lengthening 
1 mm.
2mm.
3mm.***********
4mm.
42­Wax inlay which type contain in much gradient?
Paraffin wax.***********************************
Bee wax.
43­Pt. needs complete denture u take impression with irreversible hydrocolloid & poured it after 
more than 15 min. the cast appears smooth & chalky the reason is:
a. Dehydration of the impression. ***
b. Expansion of the impression.
c. Immerse the impression in a chemical solution.
44­what is the type of wax used to verify the occlusal reduc on for full veneer
restoration
A. Onlay wax
B. Lowa wax
C. Utility wax*****************
45­Main use of dental floss:
a. Remove food debris. ( in files )****
b. Remove calculus.
c. Remove bacterial plaque. ( what I have chosen )****
46­Concentrating of acid used in etching porcelain veneer:
9.6 % hydrofluoric acid.*********
35 % phosphoric acid.
37 % phosphoric acid.
37 % hydrflouric acid.
47­Dental plaque is formed after:
a. 6 hours.*********************
b. 12 hours.
c. 24 hours.
d. 48 hours.
48­At the begining of the operation day in the clinic, you should start the water/air spray for 2 
minutes in order to get rid of which type of microorganisms :
a. pseudomonas aurignossa *****************
b. Streptococcus mutans.
c. Streptococcus salivarius
49­High mylohyoid crest in patient for complete denture, the surgeon must avoid vital structure 
which is/ during preproesthetic surgery of mylohyoid ridge reduction:
Lingual nerve.*****************
Mylohyoid nerve.
Long buccal.
IAN
50­Drug used to decrease saliva during impression taking is:
a. Cholinergic.
b. Anticholinergic.************
c. Antidiabetic.
d. Anticorticosteroid
51­Dentigerous cyst treatment >>>>> Eneculation
52­Components of the forceps >>>>> a.Handle – hing – peaks .
53­Most common malignant salivary
A.Mucoepidermoid carcinoma****************
B.Plemorphic adenoma
c .adenoid cyctic carcinoma
54­Subgingival scaling and root planning is done by:
a. Gracey Curette.***************
b. Hoe.
c. Chisel.
55­which used for special area:
a. Gracey*******************
b. universal scalar
56­10­14 yrs. pt. excessive plaque and calculus, what is the best ultrasonic to be used 
( corrected file )
A. Piezoelectric**************
B. Magnetostrictive
C. Ultrasonics
57­A 21 years old patient who has iron
deficiency anemia, difficulty in swallowing,
with examination of barium sulphate, you
found:
A.Geographical tongue.
B.Burning mouth syndrome.
C.Plummer vinson syndrome.************
D.Diabetic patient.
58­Established gingivitis >> 14­21 days
59­Minimal distance between two implants
3 mm
59­Three years old pt. came to clinic with his parents he has asymptomatic swelling bluish in 
color fluctant in midline of palatal raphe, diagnosis is :
A.Bohn's nodules.
B.Gingival cyst.
C.Lymphepithelial cyst
D.Herps semplex virus.
E.Epstein's pearls.********************
60­­5 years child with bilateral loss of deciduous molars & the anterior teeth not erupted yet, the 
space maintainer for choice is:
A.lingual arch.
B.Removable partial denture.
C. Bilateral band and loop.( I have chosen this )*
D.Non removable partial denture
61­Pedo, has trauma in 11, half an hour ago, with slight pulpal exposure, open apex, treatment 
is:
A.Pulpotomy.
B.Apexification.
C.Direct pulp capping .*************************
Extraction.
62­pt 5 y removed his upper central what to do:
A.No ttt *****************
B.Space maintainer
Crown
C.Maryland bridge
63­CONCENTRATION of sodium fluoride:
A.0.05ml daily*********
B.0.5ml twice a week
C.2 daily
64­Amount fluoride 5 year old, 0.5ppm water
fluoridation:
A.0.25***********************
B.0.5
C.1
D.1.25
65­stainless steel crown ready to cement but you find small open margin from buccal what to do
A.Crimbing wz plier 112
B.Crimbing wz plier 114********************
C.New crown
66­Jeuvenile periodontitis treatment ? >> Tetracycline
67­What does dolicocephalic means:
A.Long skull***************
B.Short skull
C.Long face
D.Short face
68­fuctional appliance:
a.bionatar*********************************************************************
B.ant post bite block
69­what make priority to private clinics than community clinics
A.Need of assistance*************************
B.Insurance
70­FROM PPE: ( Corrected file )
A.uniform
b­mask*****
71­What’s the diagnosis of this picture ( Corrected file )
A.Dens evagenation
B dens invagention********************************************
C. Crown dileceration

72­pt. came to ur clinc have an painless ulcer on the lip , which begin last 6 weeks as elevated 
border with deep center ulcer developed very quickly during first 4 weeks then
73­( slowly growing or stop growing ) have no history of truma but the pt. works outside under 
exposure of the sun. Biobsy reveals PMN & acanthotic exudate, what is the diagnosis:
a. squamous cell carcinoma.
b. keratoacanthoma.*********************************
c. verrucus cell carcinoma.
d. mucoepidermoid carcinoma.

74­Optimal & minimum crown root ratio and minimal acceptable ratio ( Corrected file )
A.1:1 and 2:3 respectively.
B­ irrelevant as long as there is no mobility.
C­ 3:2 and 1:1 respectively.
D­ 2:3 and 1:1 respectively. *************
75­pt with plasma cholinesterase deficiency , which anesthesia you use?
A.procaine
B.prilocaine**************************************
C.procaine with vasoconstricter
D.another ester type
76­Case for impacted upper bi lateral canine ( corrected file )
A. Periapical
B. Dentigerous****************
C. Residual
76­Question ( I don’t know the answer )
Qst. amelogenesis imperfecta which type of ortho brackets do u suggest:
a.invasilgn ( I have chosen this one )*
b.metalic
c.lingual bracket.
77­max denture (I don’t remember the question first time I have seen it ? ) >> extends 1­2mm 
beyond vibrating line
78­The matrix band should be above the adjacent tooth occlusal surface by: ( corrected file )
a. 1 ­ 2mm. ***
b. 2 ­ 3mm.
C. 2.5 ­ 3.5mm.
D. Below to it
79­She told me that q talking about patient restored his tooth by composite
restoration and came after one week with lighter color of composite. Lighter color
caused by???
A­ Insufficient light curing. ( I have chosen according to the file )*
B­ improper isolation.
C­ water resorption.
80_Loose enamel rods at the gingival floor of a class II amalgam cavity should be removed using
:
a. Straight chisel.
b. Hatchet.
c. Gingival curetla.
d. Gingival marginal trimmer.**************************
81_ The cement material with uniform film thickness :
a. Zinc oxide
b. Resin
c. GI
d. Zinc phosphate********************
82­Pt. with complete denture complains from tightness of denture in morning then becomes good
this due to:
a. Relif of denture. ******************
b. Lack of cheeck elastisty.
c. Poor post dam.
83­ Overcontouring of restoratn enhances
A esthetics
B cleansing action
C plaque accumulation**********************
84­ After gingivectomy surface epithelisation occurs in
A.3 days
B.5­14************
C.14­21
D.Over a month
85­ Proxy brush is used with which type of furcation ?
a­ type 1
b­ type 2
c­ type 3******
d­ type 4
Proxy brush is used with for furcation type 3 and embrasure type 2
****The rest I don’t remember
100­Last question in my exam ϋ
Seibert , 1983 classified , Class II is ?
a.buccolingual loss of tissue with normal apicocoronal ridge height
b.apicocoronal loss of tissue with normal buccolingual ridge width******************
C.combination­type defects
EXAM – 21

)patient suffered from swelling and pain at the meal time ?
a.Sialothisis **
b. Pain from Impacted 8
c. Pain from Fpd
2)During mentoplasty, doctor should take care for injury of what nerve:
a. Mental nerve.
b. Lingual nerve.
c. marginal mandibular nerve
d) . Lower branch of the facial nerve. ***

3)minimum space between 2 implant 
a.3mm**
b.4 mm
c.5mm
d.6mm
4) Gold standard for measuring malodor :
a. heliometer *
b. organoleptic
c.gas chromatography
5) Bacteria responsible for Caries initiation and progression:
streptococus mutans and lactobacilli**
6) Pic of compound odontoma
7) Treatment of Fucation type II involvement with non graft regenerative technique:
a, osseous coagulum 
b. GTR**
8)Rubberdam application:
4 contact points between clamp and tooth**
9)Discoid/cleiod instrument used for:
a. carving anatomy **
b. finishing
c. remove amalgam flushes
d.precarving…
10)Treatment ameloblastoma: 
A.hemisection of mandibule with condyle 
B.resection with free bone margin **
C.enuculation
11)long story about cyst with unerupted tooth ?
Dentigerous cyst**
12)PA x­ray for the upper ant maxilla (not clear) ?
a.supernumerary tooth*?
b.compound odontoma
c.complex odontoma
13) dye that is used with toludene blue to differentiate between cancer and normal cells
a. methylene blue
b. congo red
c.lugol****
14) .acid which is more potent cariogenic is?
a.acetic acid
b. lactic acid**
c.teichoic acid
15) disinfectant of dental chair after HBV:
1. iodoform & hypochloride
2. formaldehyde
3. ethylene oxide gas
4. 400/ ethyl alcohol/detol
a­ 1&2**
b­ 2&3
c­ 3&4
d­1&2&3
16)tt of short margin crown? Remake**
17) stainless steel crown ready to cement but you find small open margin from buccal what to do 
A. Crimbing wz plier 112 
B. Crimbing wz plier 114**
C. New crown
18) second preoperative x­ray in implant:
a.anter active computed tomography.**
b. complex computed tomography.
c.MRI
19) The aim of treatment maintenance is:
20) Study of oral health on pregnant women
a­Cohort study**
b­Cross sectional 
c­Case control
d­Observational
21) Diagodent used for caries
A.deep
B.proximal
C.superficial**
22)best time for the treatment of thumb sucking?
a.before the eruption of lower molar
b.encourge the parents to stop the habit
c.before the eruption of upper anteriors*
d.reasurre ….
23)pic of mandible where the needle touchs the condyle,name of the technique?
a­Gow gates technique**
24)case about condinsing ostitis
25) Since cavity deisn for composite should be conservative as
much as possible ; concept is acceptable?
a.Conventional amalgam**?
b.Bevel amalgam
c.Extension for prevention
d.Conservative with special preparation
26)pt with renal transplantation has lesion on palate (white dots ) Pic>>> a.pseudomembranous 
canidiasis*** 
b.erythmatous candidiasis
27)effect of water or alcohol on zinc oxide eugenol impression material
a.slow
b.delay
c.stop
d.speed***
28)to increase working time of alginate?
a.cold*
b.hot
29) Composition of gutta percha? 
70% zinc oxid, 20% gutta percha
30)function of rest in RPD?
Support**
31)The most destructive occlusal interference is
A. Centric
B. Working
C. Non working**
D protrusive
32)Pt Came with fracture ,surgeon decided to do fixation,what is the size of the wire that should 
be used?? A:4 inch 22gauge
. B:4inch 20 gauge. 
C:6 inch 26 gauge**
33) patient who has un­modified class II kennedy classification, with good periodontal condition 
and no carious lesion, the best clasp to use on the other side: 
a. Reciprocal clasp\aker's clasp. 
b. Ring clasp.
c. Embrasure clasp.**
d. gingivally approaching clasp.
34) class 2 Kennedy good oral hygiene and free of caries:
A.Circumferential clasp **
B.Ring clasp
C.Back action clasp
35) Smokers :
Oral cancer
Control
Smokers
60%
40%
Non smokers
10%
90%
What is the percentage?
13.5**
36)case about nasopalatine cyst
37) Dental materials are classified as
A. ceramic, polymers and composite
B. ceramic, polymers and alginate.
C. ceramic, polymers. Cement
D. ceramics, metals, polymers,composites**
38)case about reverible pulpitis
39) siebert ridge classification for apicocoronal loss>>>> class II**
40)role of varnish?
a. prevent discoloration*
b.prevent leakage
c. insulator Ionized
40)­anasthesia? lipid solubality of unionized ions**
41) Decalcification of enamel in ortho ?? 
A: around the bracket.**
B: under the bracket
42) ­c shaped canal found in >> mand 2nd molar
43) post extraction pain?
a. resolve after 5 day
b.treat with narcotic analgesics
c. if it persists after 3 days its alv osteoitis****

44) pt with high mastication and needs aesthetic posterior restoration, whats the best choice
a­ Zinc polycarboxilate
b­ GIC
c­ Composite with bevel
d­ Composite without bevel**
45) A tooth with fracture cusp dentine involves what is the status of pulp in this case?
a. hypersensitivity*
b. reversible 
c. irreversible
46) split dam technique used with
Single crowned tooth
Posterior molar
Fixed partial denture field***
47) nursing caries in child :
A. Lower incisor 
B. Upper incisor**
C. Lower molar
48)Pt missing 4 ant teeth and he want to replace it by Fpd,
What is the abutments for this case?
A­Rt & Lt canine
B­Rt & left canine and premolars
c­both canines and left premolar
d­both canines and right premolar***
49)t o plane the facial and the lingual wall of enamel , which enamel will use :
A. Enamel hatchet**
B. Gingival trimmer
C. Chesil
D. Plane line angle
50)picture of 8 years old pt with fractured tooth , no radiographic signs and not tender to 
percussion ,what is the treatment?
a.pulpectomy
b.partial pulpetomy with calcium hydroxide
c.pulpetomy with calcium hydroxide
d.calcium hydroxide and restoration
51) A root seldom has 2 pulp canals
A. mesiobuccal root of upper molar
B. mesiobuccal of lower molar
C. distobuccal root of upper molar**
52) Xray to see relation between impacted tooth (sorry I forgot the exact q):
1) PA*?
2) OPG**
3) occlusal
4) lateral ceph
53) Distance between cephalogram and patient 
A. 5 feet**
B. 6 feet
54) many q's about preventive resin restoration
55) What’s the name of the caries system?? 1: MTD 2:MDF 3:DMF **

EXAM – 22
.bacteria for initiation and progression of caries…. Streptococcus and bacillus
2.Avulsed tooth 30 min …soak in Naocl then NaF
3.Factor/agent responsible for initiation of caries in oral cavity.. a.strepto b. bad oral hygiene **
4.ulcer in lower labial sulcus after few days of new denture insertion .. 
A,under extended flanges
b.overcontoured flange
c.overextended flange****
d.high occlusal force
5.child came with his mother playing with instruments and becoming restless and then 
aggressive how can we manage him hyperactivity disorder :
 a. progressive management 
b. talking to him in language for children **
c. describe the steps for treatment step by step
6.sensory nerve supply to upper lip: a. facial b­ trigmenal N ****
7. After upper molar injection a colorless extraoral swelling .. inside which artery the inject was? 
Zogomatic.
.maxillary***
 facial..
8. submandibular gland calculs causing pain during meal which x ray used to detect :
Occlusal..panoramic..and Sialography.**

9.6 years old child with Bluish swelling above lower 6 whats the proper managemen:
a. incision and extraction
b.incision and induce eruption**
c.observation and small incision to expose the molar
10.which LA agent for pregnant lady in her 2nd trimester … lidocaine
11.spedding principle: Stainless steel crown selection
12.Gun shot to mandible fixation technique a.christian** b. keans
13.nefidipine induced gingival enlargement why we do gingivectomy ? ..elimination of 
pseodopocket
14. which type of dentine shiny and upper layer is calcified –sclerotic**­secondary­tertiary­
reparative 
15.Cause of pain in irreversible pulpitis..
 a. increase intrapulpal pressure****
 b. degeneration of odontoblasts 

16. a question to describe the sensation towards a stimuli to pain receptors, in the patients own 
feeling I think its just Pain.
17.LA mechanism 
a.blocking impulses**
b.increase pain threshold
c.decrease pain threshold
18.Test to differentiate between periapical abcess and periodontal abcess 
..radiograph..vitality..**
19.condition that does reach defined goals reflect effectiveness !! ethics question !!
a. controlled condition***
b.uncontrolled
c.normal
20.Molar abutment for a looong span fixed denture patient feels pain and discomfort :
a.break in connector?
b. vertical tooth # **
21.bone tempreture during implant cuse bone damage …46 C for 1 min*
22.mercury ingested daily from amalgam …1­3ug
23. % of copper amalgam which reduce gamma 2 phase ..13
24.teeth in alveolar cleft deviate to.. non cleft side Missing
25.impression disinfectant : %glutaraldehyde (but check the concentration of it )
26.truma to child with avulsed tooth put im milk since 30 min how long we splint ?
By days : 1­7 days** … 7­15 days …21 days
27.Aliminum foil test for which strilisation …ultrasonic?
28. newly postgraduate dentist took an impression to screening room how should he disinfect the
impression : a. surface only b.surface and tray c. surface and bottom and tray**
30.something regarding stage of porcelain I am sry don’t remember but I think answer was 
((dough stage !!))
31.RME (maxillary expansion) for child after few days mother called clinic that a diastema 
appears :
a.continue expansion normally ****true
b.reverse the screw
c.ask orthodontist 
32.perio weak tooth ortho movement… light
33.decalcification in ortho .. a.around brackets** b. under brackets
34. least dimensional stable.. a.metal b.plastic** c.ceramic..(I don’t remember the Question !)
35.pit and fissure Alternative restoration 
a.GIC
b.compomer
c.composite** Flowable composite
36. system depend in continuous waves:
a.Obtura1
b.obtura2
c.ultrafil
d. system b**
37. file #20 tip ? 0.2
38.occlusion in which posterior teeth are in occlusion and anterior also (if possible) 
balanced occlusion.*** centric occlusion
39.material used in oral cavity binds to all surfaces and has ability to release ions with time and 
has decrease microbial affect 
a.Gic b.flouride*
40.prophylaxis antibiotics when given .. prosthetic valve
43. pt congestive heart failure management :a. position chair in upright* ..b. treat as normal pt .. 
avoid O2
44.unresorbable suture material : silk
45.subgingival instrument … gracey
46. Xray about cleidocranial dysplasia
47.case picture has upper complete denture with porcelain lower class one kenedy with the 
anterior natural has wear defect what is called: 
a.attrition
b.erosion.
c abrasion**
48. how to make LA injection less painful :
a.topical
b.strach tissue
c.slow injection
d.gauge 25
1.a b c ***
2.a c d
3.c b d
4.a b d
49. after bracket removal discoloration whats the management ..full crown
50. central incisor with full veneer crown after sometime came need endo for same teeth ..lingual
access cavity made through the veneer crown whatis the proper way to restore the cavity? 
Remake another crown??
51.Xray with impacted lower premolar with a cyst what is the diagnosis.. dentigerous or eruption 
cyst?
52. Pt have orthodontic removable appliance after speech
Lingual alveolars voul for few days ****
Lingual alveolar for several weeks
Lingual voul for few days
Lingual voul for several weeks

53.condyle growth and formation … apposition and endocondral
54.Time for gingivitis.. 14­21 days
55.in pedo which nerve should anesthetise for upper molar.. superior posterior AN ??
56.Ghost teeth : dentinal dysplasia Regionl odontoplasia
57. Important in rotary endo.. precoronal enlargement
58. Most common malignancy in oral cavity… SCC
59.which are opposing retentive are in RPD ..reciprocal?
60.For good oral hygiene which clasp? Circumfrential
61.instrument has positive rak angle : without saying more
a.kfile*
b.reamer
c.?? but h file was not there
62.Malar bone which case? Palmman vinsion syndrome
63.Antiboitic inhibits cell wall ..penecillin
64. first formation of tooth bud ..6 weeks embry
66.most important property in fissure sealant material ..resistance
67.decay index .. DMF
68.percentage or stannous fluoride 8%
69. what is the thist occlusal refrence point ..a .nason *b.condyle. c occlusal plane

70.whicle retreatment of endo after GP removal dentist tries to insert file to working length but 
lost it?? ..ledge**
71.overhanging restoration affects.. a. contact area b. periodontal health**
72. apicectomy cut angle : a.perpendicular b. acute** c.obtuse
73. indication of maintaining good oral hygien by ..: decrease plaque index.?
74.why denture put in solution after remval in bed time : to avoid fracture
75.Most common tooth has abcess a.molar** b. premolar c.incisor d. canine
76.why caries in adult more than pedo
77.Allergy from composite because of with content : a . monomer
78.facial fracture causing list of signs I forgot but it was zygomatic complex fracture same file
79.Most common irrigation:Naocl
80.Lap pontic disadvantage.. Tissue irritation
81.measure attachment loss :
a.from free gingiva to pocket
b.from cej to pocket.**
from cej to mucogingival junction
82.patient came after 48 hrs of complex crown fracture of incisor exposed pulp management:
a.rct**
b.partial pulpotomy
c. complete pulpotomy
d.Dpc
83.pt came to clin had checkenpox dr said it affected calcifying and eruption .which stage of 
development affected :
a.proliferation
b.histodiffrentiation***
EXAM – 23
1. Blood supply to TMJ?
a. External carotid artery

2. Anesthesia for lower posterior teeth bone and tissue 
a. Gow gates ***
b. Akinose
c. Inferioor alveolar nerve block
3. Diameter of arteriole of dental pulp in micrometer
a. 35­45
b. 15­25
c. 5­10==
d. 0.3­1

4. Bacteria not found in pericoronitis
a. bacteriodes==
b. peptostreptococcus
c. haemopis
d. Prevotella

5. Management of root that got displaced into the maxillary sinus
a. Leave and observation
b. Administer antibiotic
c. Remove it as soon as possible ***

5. Pt with buccal space infection related to lower molar what is the emergency management
a. Extract the tooth and antibiotics later
b. antibiotics
c. Incision and drainage
d. Refer to maxillofacial surgeon??

7. How many carpules can you give from lidocaine 2% with 1:100 000 epinephrine to a child who 
weighs 80 kg in mg
a. 560***
b. 450

8. Uncommon pattern of mandibular permanent teeth eruption
a. Lower central before 1st molar
b. 1st molar before central
c. Lower 7 before 2nd premolar ***

9. Ideal medium for avulsed permanent 
a. HBSS

10. Best media for avulsed tooth in child
a. Cold milk ***
b. Saliva
c. water

11. 5 yrs old lost his upper central what to do?
a. Nothing ***
b. Polycarboxylate crown
c. Space maintainer
d. Resin bonded bridge
12. Maxillofacial surgeon wants to put 2mm plate what is the size of the drill he will use?
a. 1mm
b. 1.5mm ***
c. 2mm

13. Amalgam cupper concentration to eliminate gamma 2 phase?
a. 13%

14. Retention in amalgam filling 
a. Walls convergence ***
b. divergence

15. Emergency after giving LA what is the most common reason?
a. Epinephrine
b. Stress??

16. Waldeyer's ring
a. Lymphoid tissue ***
b. neural

17. Pt with gun shot wound condyle is injured and surgeon want to replace it with graft from?
a. Anterior iliac crest
b. Post iliac crest
c. Costochondral??

18. True about dentinogenesis ?
a. Dental lamina becomes clear
b. Dentinal matrix is involved in the process ??

19. Pt with dog bite in the face he is stabilized and tetanus shot given what will the maxillofacial 
do?
a. Irrigate with Hydrogen peroxide
b. Lacerate the wound so it would heal primary healing
c. Place iodine inside
d. Suture in layers ??

20. Autoclave?
a. dry
b. Wet steam sterilization ***
21. Perio furcation involvement II with recession what is the regenerative procedure without graft
a. GTR ***
b. Osseous coagulum
22. Floss 
a. Removes plaque
23. Pt with anterior cleft lip and palate at what time did the problem develop?
a. 8­11 intrauterine life ***
b. 4­6
24. File with positive rake angle
a. K file ***
b. Profile
c. Protaper
d. K reamer
25. Patient swallowed fluoride tooth paste what the best first response
a. Drink milk or similar high calcium product ??
b. Take to emergency
26. Material in caries dye
a. Propalyne glycol
27. What is a suitable combination of files
a. Yellow­10
b. Red­25 ***

28. Pt had RCT and 1 week after obturation he had persistent pain, in x­ray filling is short what to
do?
a. Give analgesic
b. Start retreament
c. Give antibiotic and analgesic 
d. Wait and observe

29. Fluoride in special need pt
a. APF gel
b. Stannous fluoride solution
c. Sodium fluoride solution
d. Fluoride varnish ***

30. File length
a. 21,25,31
31. Access cavity in upper central and lateral incisors
a. Oval ***
b. triangular
32. concentration of fluoride in water
a. 1 ppm

33. 7 year old with unilateral functional crossbite on right side due to mandible shift when 
opening? What to do
a. Unilateral maxillary expansion on right side
b. Unilateral maxillary expansion on left side
c. Wait and observe??
d. bilateral maxillary expansion

34. normal or excessive occlusal forces are placed on teeth with compromised periodontal 
attachment
a. primary occlusal trauma
b. secondary occlusal trauma ***
35. Pt 67 years old with bony lesion in the mandible lower face is enlarged lower lip is protruded ,
hypercementosis, thickened mandibular cortex, in x­ray mixed RO and Radiolucent lesion in 
mandible area
a. Paget’s disease ***
b. Fibrous dysplasia
c. Florid osseous dysplasia

36. Pt 22 yrs old with bluish dots in the palate, rash for the previous 4 days, thrombocytes count 
25 000? 
a. Leukemia 
b. Thrombocytopenia purpura ***
37. X­ray case of dentigerous cyst
38. Step before final impression
a. Special tray fabrication
39. Cause of shrink spot in crown casting?
a. High temperature
b. Wide sprue
c. Long sprue handle

40. Pt with lost lower 2 premolars history reveals that he has xerostomia how to replace his 
teeth? 
a. RDP??
b. Implant fixed bridge ??
c. Convetional bridge
d. Resin bonded bridge

41. Patient with ulceration all over his gingiva especially in the interdental papilla area, halitosis 
a. Necrotizing ulcerative gingivitis?

42. Gingival margin trimmer is used for
a. Beveling gingival margin of class II

43. Patient came for a denture very anxious …
a. Hysterical pt

44. Denture missing a component that prevents tissue wards movement which is causing 
problem the pt what is the missing part?
a. Occlusal rest

45. Most common salivary gland tumor?
a. Pleomorphic adenoma

46. Most common benign salivary gland tumor
a. Pleomorphic adenoma

47. Space mesial to upper canine in primary teeth
a. Primate spaces
48. The most important factor in oral habits causing maloclussion ?
a. frequency
b. duration??
c. force
d. direction

49. peasoreamer is most useful in 
a. removing vital tissue from thin canal
b. removing cotton from canal??
c. Preparing coronal part of the canal

50. Impression material with the highest stiffness
a. Polyether

51. The duration of anesthesia is affected by:
a. removal time
b. bond strength between the anesthesia & the nerve
c. anesthesia bond strength & removal time??
d. The duration of administration

52. Difference between Gracey and universal curette: 
a. Gracey used for cutting in specific area while universal is in any area.
b. Gracey has one cutting edge while universal has two.
c. gracey has 70 offset, universal 85
d. Section of gracey is hemicircular and in universal triangular.
1. A,b,c??
2. A,b
3. A,b,d

53. Cerebrospinal fluid can be recognized because it contains?
a. high protein
b. high glucose??
c. Beta 2 transfirrin
d. glucose oxidase

54. Rubberdam application:
a. 4 contact points between clamp and tooth Age of the patient

55. NiTi feature
a. Shape memory

56. Maxillary sinus opens in
a. Middle meatus

57. Contraindication for veneer
a. bruxism
58. Complete denture pt tissue over mandibular ridge is easily retractable how to manage?
a. Minor surgical procedure to correct??
b. Impression will replace the tissue

59. Patient with high caries risk which smooth surface of permanent teeth is prone to decay first
a. Facial of upper posterior
b. Lingual of upper posterior ??
c. Facial of upper anteriors
d. Lingual of upper anteriors

60. Pt with high caries risk, Stained questionable pits and fissures, what step do you do for 
diagnosis?
a. Follow­up after 6 months
b. Exploratory opening of the pits and fissures ***
c. Composite resin filling
d. Seal the pits and fissures

61. For composite restoration conservative design
a. Amalgam preperation
b. Beveled amalgam preperation
c. Preparation following decay ***

62. Pt with discolored proximal surface, no cavity, good oral hygiene, management?
a. Apply sealant
b. Restore
c. Fluoride ***

63. Pt with decayed #36. Teeth #16,#26,#46 are stained but sound what is the management? 
a. Apply sealant

64. Endo diagnosis case?
65. Patient with two moderately deep carious lesions, pulp response is normal what to do?
a. Place permanent restoration in one visit??
b. Place permanent restoration in two visits
c. Place temporary and then permanent in 1 week

66. Definition of reversible pulpitis
67. Pt has prolonged pain on cold, sometimes spontaneous pain diagnosis? Irreversible pulpitis
68. Concentration of NAOCL? 5.25%
69. AH plus different than AH 26? no formaldehyde release
70. Indication for doing indirect pulp capping in pedo? Caries penetrating dentine
71. Brushing technique that uses side of bristles of the brush? Modified stillman
72. Pt with calculus advised to use brush frequently why? To break plaque formation ..
73. Base of carious lesion is always toward
a. Enamel 
b. DEJ ***
74. Treatment of acute pericoronitis?
a. Antibiotic
b. Excision and antibiotic
c. Curettage and irrigation

75. Pt came with pain prolong pain on cold on a specific molar, you tried to do cold, hot, electric 
pulp test but couldn’t reproduce the symptoms what should you do for pulpal diagnosis?
a. Percussion test
b. Anesthetic test**
c. Drilling test ***
76. Gutta percha is mostly composed of
a. Zinc oxide

77. Pt lost his lower 6 and you want to replace it with a bridge, upper 6 is extruded. Where will 
interference most probably take place?
a. Protrusive movement ??
b. Working side interference
c. Non­working side interference

78. The vasoconstrictor affects LA in terms of?
a. Increase duration and intensity?

EXAM – 24

2.When is Thermo plasticized GP used


a.Canal with irregularities***
b.Curved canals
c.Where lateral condensation is difficult
d.Canals with open apex

3. What are the disadvantages of mcspadden technique in obturation:


a. Increase time
b. Increase steps.
c. Difficult in curved canals***

4. Retention of Complete denture – From Posterior Palatal Seal***

5. Posterior palatal seal is ?


a.Distance between hamular notch
b.junction of hard and soft palate***

6. 10y ingested 10mg fl. What is the immediate management


a. Drink milk withe calcium***
b. Go emergency
c. another option that is not related
(there was no option of do nothing otherwise would have chosen it)
7. Scaling in epileptic motion?
a.ultrasonic

b – magnetostrictive

c- piezoelectric
8. APF Conc
a.1.23***
b.12.3
c.123
d.0.123

9. - in upper 8 Impaction to avoid tear of gingival flap


a. Adequate size of flap
b. Flap include greater palatine nerve
c. Strong retraction of flap margin

10. 4th no. In instrument classification—> angle of blade***

11. A picture showing occlusal wear of lower teeth opposing PFM


crowned upper anterior teeth what is the diagnosis:
a.abrasion*
b.abfraction
c.erosion
d.attrition

12. Percentage to prevent phase 2 amalgam copper = 13%

13. newly erupted premolars and molar>>>


a.fissure sealants***
b.preventive
c.fluoride

13. Question on characteristics of pagets disease


14. Question on cancer related to gardners syndrome in which organ
a.colon***
b.lung
c.heart
d.pancreas

15. epoxy brush = embrassure type II***


16. Most difficut tooth to anasthesize = lower molars***
17. c shaped canal found in = mand 2nd molar***
18.Mechanism of GTR technique
19.Chlorohexidne conc. = 0.12

20.patient with swelling & diagnosis is keratocyst, to confirm the


diagnosis they decide to do fine needle aspiration what needle gauge
used
a.19
b.21
c.25
d.30

21. Tooth with external resorption


a.Necrotic***
b.irreversible pulpitis
c.reversible pulpitis

22. Pink tooth


a.internal resorption***
b.external resorption
23. The worst type of bone
a. Type I
b. Type II
c. Type III
d. Type IV***

24. exposed roots, caries free teeth but reversible pain with cold water =
Dentine hypersensitivity***

25. opg xray showing supernumerary teeth


a.ectodermal dysplasia
b.cleidocranial dysplasiac***
c. craniofacial dystosis

26. order of placing of winged clamp type rubber dam


a. clamp before dam
b. clamp aftr dam
c. clamp and dam together ***

27. Niti feature = shape memory***

28. Relative contraindication of NO2 = suppression of gag reflex (made


most sense out of all options that I cant really remember).

27. pic of palatal infiltration in anterior area anesthetizing which nerve


a.nasopalatine***
b.greater palatine

28. mercury ingested daily from amalgam = 1-3ug ***

29. how to make LA injection less painful :


a. topical
b. stretch tissue
c. slow injection
d. gauge 25
1.a b c ***
2.a c d
3.c b d
4.a b d

30. Time for gingivitis.. 14-21 days

31. Formation of cleft palates in which phase


a.6
b.8-10*** week
c.15

32. decay index = DMF


Tissu
33. Ridge Lap pontic disadvantage = e irritation***
34. fiber optic light can be used to localize the canal orifices by directing
the light to the pulp chamber, the orifices will be seen:
a. Light
b. dark ***
c. Yellow
d. White

35. Water irrigation devices is Used to:


a. prevent plaque Formation
b. removes plaque ***
c. dilute bactetial Toxins

36. In hairy tongue which papillae increase in numbe


Filliform***
Fungiform
Circumvallate

37. Flouride should be applied in


2mins
4min***
6 min

Anxiety induced nausa

Exam 25
. Hemophilia B ­ decrease in which factor? > Factor IX
2. Supply of upper 4 palatally by: 
A. Lesser palatine 
B. Greater palatine **
C. Nasopalatine
3. Simplest way to stop bleeding? 
A. Retraction cord after matrix band
B. Use cotton pellets press for some time**
4. Most important for dentist to know?
How to manage situation and control
5. Mother called dentist, her child swallowed fluoride? 
Give calcium supply and take to emergency directly
6. How to measure if periodontitis is healing? 
Decrease bleeding on probing 

7. Local anesthetics aid in reducing the flow of saliva during operative procedures by
A. blocking the cholinergic nerve endings
B. blocking innervation to major salivary glands
C. blocking efferent parasympathetic nerve pathways
D. reducing sensitivity and anxiety during tooth preparation**
8. All are preventive method except?
A. Pit and fissure
B. Fluoride tooth paste 
C. Fluoride in water 
D. GIC restorative**
9. What’s the access opening for upper lateral? > oval
10. preparation for labial surface in one plane in the preparation for metal crown is:
A) More retentive.
B) Less retentive. *** ‫أ‬
c) Less cutting of tissues
11. X­ray of impacted 3rd molar, what is diagnosis?
A. Radicular cyst 
B. Dentigerouse cyst
12. Long case at the end written multilocular giant cell, what is diagnosis?
Giant cell granuloma
13. Read about spaces in children (leeway, primate space)
Photo of child upper teeth with arrow asking what this space is? (It was between canine and 
premolar) 
A. Physical space 
B. Leeway space
C. Primate space**
14. What connect the abutment to prosthesis? 
A. Pontic
B. Rest**
15. X­ ray of lower 6 with radiopacity in root area (around root), diagnosis? 
A. Hypercementosis
B. Condensing osteitis **
16. Forceps to hold tissue while suturing? > Addison’s forceps
17. Question about scaling device used? Ichise ultrasonic scaler
18. Sub gingival scaler? > Gracey
19. Instrument to measure furcation? > Nabers
20. Irreversible hydrocolloid (alginate) impression left 15 mints then poured with stone, cast was 
milky and chalky, Cause?
I chose dehydration of impression
21. Patient came from another clinic with perforation? > use MTA
22. Why to use calcium hydroxide between visits? > Because of antimicrobial effect
23. procedure of cutting half of the tooth (mesiodistal in upper mollars )(buccolingual in lower 
molars) by removal off all necrotic tooth part with its root & leaving of the normal root is called :
a­tooth hemisection**
b­root bisection
c. root amputation
24. Type of graft from same person? > Auto graft
25. GIC advantage? > release fluoride
26. Question about amalgam? > Convergence occlusally
27. Submandibular Wharton duct secrete saliva through :
A­ floor of the mouth via 10­20 ducts
B­ venteral surface of the tongue***
28. over contoured crown? 
Accumulate plaque
29. Most surface collect plaque? > Upper buccal posterior teeth
30. Photo of pterygomandibular raphe, what is this type of anesthesia? 
A. Gow gate 
B. Alveolar block*
31. 3 x­ray with small radiolucent circles what’s the diagnosis? 
A. Osteosarcoma 
B. Ameloblastoma
32. Why to advice pt to brush his teeth? > To remove food debris
33. Pt with short RCT, retreatment, GP removed, dentist can’t locate the apex in the canal, why? 
Forgotten the options!!
34. Post preparation of upper 4, perforation occurs what is the reason? 
I chose furcation is more cervical (most reasonable option)
ou make a ledge in the canal! You want to correct this! What is the most common complicantion 
that can occur on this procedure?
a. perforation***
b. stripping
c. creation of false canal
d. apical zip
35. Patient with class 2 composite restoration came after two days complain of pain why?
Overhanging
Forgotten the other options!!
36. Severely fractured tooth due to caries lesion, tooth is vital what is the best to give retention 
and resistance? 
A. Crown with post
B. Amalgam bond ( i chose )
37. What’s the fatal dose of fluoride?
A. 345 micro 
B. 400 micro 
C. 5­10 micro ***
D. 1 gram

38. Child swallows fluoride what’s the dose and time to be injected? 
Forgotten the options!!
39. Photo of child with lesion white spots in half of palate from posterior teeth to anterior including
lateral and central, what is the diagnosis?
40. Old patient with denture with poor oral hygiene complaining of pain? Denture stomatitis
41. Cross section of wrought wire? > Round
42. Anterior posterior strap used in? 
I chose u shape (SLE)
43. Patient with caries in lower posterior you found there is sever attrition, best management? 
A. Composite restorations 
B. Crowns ( i chose )

Exam 26
1­ role of myoepithelial cells in salivary glands 
A. Secret saliva 
B. Contraction when the gland stimulated to secret saliva ***

2­ calculate mean arterial pressure systolic 140 diastolic 80
A.100** b.3 c.110 d.80

3­ stainless steel crown ready to cement but you find small open margin from buccal what to do 
A. Crimbing wz plier 112 
B. Crimbing wz plier 114**
C. New crown

4­ marginal deterioration —> mahlar scale
5­ revers bevel in inlay 
A. Retention.**** B. Resistance 

6­ biocompatible alloy when used intra orally 
A. Titanium **b. Nickel c. Steel 

7­ minimum space between implants —>3mm

8­ space on cephalometry —> 5 feet

9­ Orthognathic surgery , you plan to use 2mm screw , the drill size :
a­ 1 mm
b­ 1.5 mm***
c­ 2 mm
d­ 2.5 mm

10­ Treatment of cervical caries in old patients with a temporary restoration is best done by:
a) Glass ionomer. *** due to its chemichal bond & fluoride release
 b) Composite resitn.
c amlgam 

11­ new 7 y healthy pt came to you with multiple caires what is the xray for choice
A. 2 bitwing
B. Selective periapical and 1 bitwing 
C. Panorama 
D. Selective periapical and 2 bitwing and panorama**

12­ When u condense the amalgam after trituration:
A. Immediately**
B. After 3 min
13­ forceps for epilus fissuratum —> allis forceps
 
14­pt when to give antibiotics ( i think pt have immunity problems)
A.periapical granuloma
B. Osteitis 
C. Multiple extractions ***
D. Incision and drainage 

15­ duct open near circumferential papillae 
A. Von ebner **B. Specious gland. C. Stenson duct 

16­ What are the disadvantages of mcspadden technique in obturation: 
1. Increase time
2. Increase steps.
3. Difficult in curved canals**

17­ Caries progression in children more rapid than adult due to:
1. Difference in PH.
2. Generalized dentine sclerosis by age*
3. Increasing in organic content of tubular dentine by age

18­ Patient had class ll amalgam....... radiographically an over hanging amalgam is present. This 
is due to
Lack of matrix usage 
B. Insufficient carving
C.improper wedging ***
D.no burnishing for amalgam

19­The overhanging restoration 
A. Increase the microleakag of the restoration 
B. Affect integrity if proximal contact 
C. Affect on periodontal health*

20­ space between soldered parts 
A. 0.02. B. 0.2 ***C. 1. D. 2

21­ wall of access should be 
A. Divergent ocllusally **
b. Parallel 
c. Convergent 
22­ pits and fissure restoration affected by
A. Contamination by saliva**
B. Flow of saliva
C. Depth of cavity 
D. Strength of itch

23­ Florid in tooth past —> 900­1500

24­ hypertension and diabetic pt have ulcer 1*2 cm in lateral border of tongue what the type of 
incision 
A. External b. Internal ****c. Punch 

25­ lady has clas 3 composite with white margin no pain no caires what to do
A. Repair***. B. Add composite layer. C. Remove restoration and redo

26­ promary stress bearing area of mandible 
A­Buccal shelf area of mandible **
B­Residual ridge
C­Retromolar pad

27­ drug cause gingival enlargement 
Phenytoin 

28­ pt with sickle cell anemia don’t give —> aspirin 

29­ hard dentine removed by—> high speed

30­ 6 y with thumb sucking and teeth affected
A. Early mixed appliance 

30­ Bacteria which can invade epithelium?
A­ Porphyromonas Gingivalis***
B­ Fusobacterium
C­ Streptococcus mutans
D­ Capnocytophaga

31­ 10y ingested 10mg fl. What is the immediate management
A. Drink milk withe calcium..... *
b. Go emergency 
C. No thing to do 

32­ canine edentulous both side class —>3
33­ retorted central incisors than lateral incisor —> class 2 division 2

34­A female patient came to your clinic with dry lips and mouth and bilateral submandibular 
oedema and ocular dryness. Diagnosis is: 
a) Polymorphic adenoma.
b) salivary stone 
c) Sjögren's syndrome**

35­ class 2 Kennedy healthyteeth no caires no pdl disease what type of clasp?
A. Circumferential *
B. Ring 
Cant remember the other options 

36­ non retentive arm where to put it?
A. Near fulcrum 
B. Away from fulcrum *
C. Away from edentulous 
D. Near teeth

37­ subgingival instrument—­> gracy
38­ fracture between middle and apical third 
A. Rct for coronal part*
B. Splint
C. Extraction 

39­ pt asymptomatic diagnostic xray you find lower 8 roots near to the inferior alveolar nerve 
canal what to do?
A. Surgical extraction 
B. Resection 
C. Observation **

40­ smoking and non smoking —> less resilient than non smoker***
41­Scaling in epileptic motion?
A..ultrasonis
B.Piezoelecrtic
C..Magnetistrictive**
D.scaler

41­The infection will spread cervical in infection from:
1. Lower incisors.
2. Lower premolars.
3. Lower 2nd and 3rd molars.***
4. Upper incisors

42­ Water irrigation device
A.prevent plaque formation
B.completely removes plaque
C.dilute bactetial toxins**

43­ material used in implant —­> titanium 
44­ material make wear to enamel—­> porcelain 
45­ test to check pulp except—­> percussion 
56­ to differentiate between vital and non vital pulp
A. Electric ***b. Percussion c. Palpitation d. Xray

57­ modified ridge lap used in :
A. Upper 1***
 b. Upper 7 c. Lower6. D. Lower4
58­ infiltration—­> supraperiosteal 

59­ Patient with renal replacement and there is an image show a white spot on his soft
palate, diagnosis:
A. Pseudo membranous thrush **
B. Erythrematous candida
C. Bacterial infection
D. Viral infection

60­ what make priority to private clinics than mcommunity clinics 
A. Need of assistance **
B. Insurance 

61­ Q have supply and demand ( i don’t remember the Q or the answers at all sorry)

62­ pt will make metal framework pt class 1 modification 1 with picture next step to do relining the
tech: to take impression : 
A. Hindels tech. **
(Can’t remember the other answers)

63­pt with acute pericoronitis what is the management?
A. Antibiotics 
B.Excision and antibiotics
C. Rinsing, swap , antibiotics *
D. Analgesics

64­ crown short margin all around and fail to make it seat management 
A. Remake** b. Cement it

65­ trechercholin syndrome—­> defect in malar bone

66­ Dental materials are classified as
A. ceramic, polymers and composite
B. ceramic, polymers and alginate.
C. ceramic, polymers. Cement
D. ceramics, metals, polymers,composites*

67­ Edentulous pt cl II kenndy classification 2nd premolar used as abutment when we surving we
found mesial under cut
wt is the proper clasp used:
1/wrought wire with round cross section*
2/ wrought wire with half round cross section
3/cast clasp with round cross section
4/ cast clasp with half cross section “RPC

68­ access in lower 2
A. Oval** b. Triangle c. Square 

69­ fluoride used in dental clinic —> duraphate 
70­ new born wtith mass in upper anterior region, diagnosis
a Epstein’s pearls **
b Bohn’s nodules
c Congenital Epulis

71­ nerve sensation in tmj
72­ pt with early loss of primary teeth—> papillon lefever syndrome
73­ retromolar area surgery be careful of—> lingual nerve 
74­ in upper 8 Impaction to avoid tear of gingival flap 
A. Adequate size of flap*
B. Flap include greater palatine nerve
C. Strong retraction of flap margin 
75­ most common surgical extraction teeth—> lower 8
76­ 4th no. In instrument classification—> angle of blade

78­ when you give the pt anesthesia the hand that holding the syringe should be :
A. On Pt’s shoulder 
B. On Pt’s hand
C. Away from pt*

79­ xray give real root length 
A. Parallel tech.**
B. Bisecting tech.

80­ pt withe sever pain , swelling and no response to electric pulp test:—>
Necrotic symptomatic absess 
81­ pregnant woman with bleeding gingiva
82­ periodontal health of pt.... can’t remember 
83­ avulsed teeth best to put in —>Low fat milk
84­ mother call you her son has avulsed teeth you should advice mother to keep tooth in :
A. Cool milk **
B. Hot milk 
C. Saliva 
D. Water
85­ pt 5 y removed his upper central what to do:
A. No ttt*
B. Space maintainer
C. Crown
D. Maryland bridge 
86­ Difference between standardized and conventional gutta percha
87­ About the ISO instruments file?
A­length of handle
B­ widths of file tip*
88­ Simplifi type canal enlargement by NiTi in or NiTi rotary file is
A// universal protper*
B// reciprocal
C// revers s
89­ sharp pain on trigger —­> trigeminal neuralgia 
90­ pt with porcelain in upper what to do in lower—> porcelain 
91­ amalgam copper —>13%
92­ time for gingivitis—­> 14­21 days
93­ oval radiolucency between roots of upper central incisors 
While taking X-ray for upper right first premolar with two equal roots using
mesial slob, its lingual root will move [comparing to the zygomatic
pgrocess]:
a. distal.
A. Incisive foramen* b. Mesial. *
B. Radicular cyst c. Palatal.
C. Granuloma d. Lingual
D. Absess 
94­ pt with denture clicking—> high vertical dimensions 
95­ radiolucency between roots of lower bicuspid—­> mental foramen 
Exam 27

1)SLOB technique
2)Dentist must know emergency resuscitation procedure,ABC stand for what? Air way breathing 
circulation
3)step ladder appearance in x­ray in what disease?
 Leukemia..
sickle cell anemia…***
cerebral palsy

4)mach bands in radiograph is misdiagnosed as? Proximal caries…occlusal caries…periodontal 
pocket**

5)optimal fluoride in water? 0.5­0.8

6)used for fluoride topical application? 8% stannous fluoride

7) Lateral cephalometric radiographs of a growing patient were taken several years apart, when 
superimposed over each other, the result would show that mandible and the maxilla haad moved
during growth
a) Downward and backward
b) Downward and forward**
c) Upward and backward
d) Upward and forward

8)failure of teeth formation will affect the growth of what structure? Whole face…
maxilla..mandible…alveolar bone***

9)nystatin used for treatment of? Candida**..pain…herpes…aphthous
10)image of mesiodense and asking about cause of retained primary**
11)image of crowding and asking about the cause? Supplemental lateral incisor**
12)image of a newborne and asking about diagnosis? Natal tooth with bilateral complete cleft 
palate and lip**
­­­­­­­­­­­­­­
Exam 28 :
1. Patient came with mandibular fracture and the surgeon decides to do maxillary mandibular 
fixation. what is the size of the wire that should be used:
a. 27inch 31 gauge
b. 21 inch 27gauge 
c. 22 inch 24gauge***
2. Leeway space on each side of a lower arch:
a. 1.5**
b. 2.5
c. 3.5
d. 4.5
3. Patient came with recurrent herpetic gingivostomatitis what is the treatment
a. Acyclovir**
b. Antibiotic
c. No treatment
4. What is the property of nickel titanium wires that makes it better that others
a. Rigidity
b. Memory**
5. Simplifill type of obturation is used in which rotary system
a. Protaper***
b. Reciprocal
6. Temperature and time used in autoclave sterilization
a.121 for 20 minutes*   15 ­ 20 at (121) \ 3 min only at (134)
b. 150 for 30 minutes
c. 170 for 20 minutes
7. patient came with periradical radiolucency after a short obturation, dentist decide to do 
retreatment, what is the best approach???
a. retrograde filling approach after nonsurgical RCT
b. clean to and beyond the apex***
8. what part of the denture distributes occlusal force??
a. direct retainer
b. denture base**
c. indirect retainer 
9. brushing technique in which parts of the bristles are activated??
a. modified stillman**
b. bass 
10. ideal time for chlorohexidine mouth rinse to kill bacteria??
a. 30seconds**
b. 60 seconds
11. shape of access cavity of premolar with 2 canals 
a. ovoid **
b. trapezoid  
12. maximum dose of anesthesia in 1 carpule of 2% lidocaine 1:10000 epinephrine
With epinephrine =7 , without epinephrine = 4.4
13. what type of bone graft is used in treatment of furcation type II??
a. corticocancellous***
b. cortical
c.
14. 9 years old child drinking 0.8ppm fluoride, how much fluoride supplement does he need?
a.0ppm**
b.0.5 =0.2
c 0.25= 0.5ppm
15.optimum water fluoridation in PPM
a. 1.0**
b.10
c.100
16. optimum crown root ratio in fixed partial denture:
a. 1:1
b 2:3**
c. 2:1 
17. percentage of maxillary teeth fracture:
a. 75%**
b.25%
c. 10%
18. 16 yrs old patient came with intrusion 3yrs ago, no symptoms, no radiographic changes what 
will your approach be??
a. no treatment***
b. orthodontic treatment
c. RCT
19. you open your water air spray n allow it allowing every morning to eliminate which bacteria??
a. Pseudomonas aurignossa **
b. aueros
20. you are performing an amalgam filling on a child and his mother asks how long does this type
of filling last???
a.20 years**
b. 2years 
c. 10 years 
21. At what age does tetracycline staining occurs??
a. 1 year**
b. 6 years
c. 10 years
22. how do you disinfect gutta­percha in the clinic??
a.5%sodium hypochloride**
b. 5% hydrogen peroxide
c. autoclave
23. a doctor wants to take a final impression for a complete denture making, he asks his students
what is the step done before taking this impression??
a. special tray making**
b. jaw relation
c. surveying
24. in how many days does established gingivitis occur??
a. 7days
b. 21 days*
c. 14days 
25. Contraindication of gingivectomy???
a. gingival abcess
b. gingival enlargement
c­Alveolar **
26. what is the most important property of porcelain??
a. esthetics**
b. strength
c. stability

27. what is the type of dentine formed in response to caries?
a. tertiary dentine**
b. secondary dentine
c. primary dentine 
d. scelorotic dentine
28. cleft lip occurs as a result of 
a. malunion of medial nasal and maxillary process**
b. malunion of lateral and medial process
29. the lip is formed from
a. 2nd pharyngeal arch**
b. 
c.
30. one of the following is one is the characteristics of propofol
a. muscular** (hyperventilon)
b. hypertension 
c. analgesia
31. during preparation of class V you find out that caries has extended to the cementum, what 
type of restoration are you going to use??
a. GIC**
b. composite
c. amalgam
32. during examination you find a V shape cavity, what type of tooth wear is this??
a. abrasion
b. erosion
c. attrition
d. abfraction**
33. You want to prescribe antibiotic for your patient what is the test you will do to figure out the 
best antibiotic??
a. culture sensitivity test*
b. 
34. patient came to your clinic with carious exposure of a mature tooth what will your treatment 
be??
a. RCT**
b. pulp capping 
c. apexification
35. Streptococcus Mutans affects which tooth surfaces??
a. pits and fissures **
b. smooth surfaces
c. root surface
36. 2 years old patient came with caries in most of his teeth what is the name of this type of 
caries??
a. early childhood caries
b. incipient caries
c. nursing caries *
37. pedo patient has nursing bottle caries in all his teeth except lower anteriors, what is the 
reason for that??
a. teeth are covered by the tongue
b. salivary wash 
c. 
38. when should parents take their child to the orthodontist ???
a. after eruption of anteriors**
b. after all teeth erupt 
c. when there is crowding 
39. patient wearing complete denture came to your clinic complaining of clicking this is due to ?/
a. increased VDO**
b. protein deficiency 
c. improper setting of the teeth
40. A surgeon is performing apicectomy, how is the bur placed to promote better healing after the
surgery??
a. perpendicular to the root **
b. parallel to the root 
41. how to treat ranula??
a. excision
b. marsupialization**
c. resection
42. what is the most conservative way of treating tooth discoloration??
a. bleaching*
b. microabrasion
c. veneer
d. crown
43. what best describes post­operative pain??
a. it’s called condensing osteits if continues for 3 days**
b. can be treated by analgesics 
c. should subside after 1 day 
44. which impression can be poured twice
a. additional silicone***
b. alginate
c. condensation silicone
45. where is the opening of the submandibular gland located??
a. opposite the maxillary molars *
b. opposite to anterios
46. best irrigant used in RCT
a. sodium hypocholoride**
b. hydrogen peroxide
c. sodium fluoride
47.which material is used for denture rebasing??
a. heat cure acryl**
b. self cure acryl
50. patient complaining of pain on cold and spontaneous pain sometimes, what does this 
indicate??
a. irreversible pulpitis **
b. reversible pulpitis 
c. necrotic pulp
d. information not enough 
51. acute necrotizing ulcerative gingivitis is associated with??
a. pseudomembranous formation**
b. gingival enlargement 
52. patient came to your clinic with pain but cannot locate the pain, you decide to do anesthesia 
test where will you start??
a. maxillary molars first then anteriors** 
b. mandibular anteriors first 
53.patient came with trauma you suspect mandibular ramus fracture, which radiograph do you 
order to confirm??
a. lateral view *
b. submentovertex 
c. towns view
54.patient has premature contact in anteriors during eccentric relation but no contact during 
centric relation, you want to do selective grinding??
a. functional cusp of posteriors 
b. incisal edges of anteriors **
c. nonfunctional cusps of posterior
55. Perikymata comes from??
a. striae of retzius**
b. hunter shrugger bands 
c. enamel sheath
56. patient comes to your clinic to make a new denture, you notice that she difficulty in walking 
and enlarged mandible, her test results show elevated alkaline phosphatase, what is your 
diagnosis??
a. Pagets disease *
b. Gardner’s syndrome
57. you do RCT in 21, with conservative access cavity, tooth had only mesial caries, what will 
your final restoration be??
a. Only crown
b. post and core
c. composite and post
d.composite only***
58. differences between gracey and universal curette 
a. area specific
59. how is amalgam cavity designed??
a. 90 cavosurface angle** 
b. 60 cavosurface angle
60. in periodontal treatment patient compliance is an important factor in addition to??
a. quality and amount root debridement**
b. other answers were not related
61. maintenance phase in periodontal therapy is done for??
a. to evaluate the treatment and healing 
62. lip is formed from
a. 1st pharyngeal arch
b. 2nd pharyngeal arch**
63. characteristics of propofol
a. muscular inactivity**
b. bronchodilation
64. Property of MTA that make sit superior to others
a. good sealer **
b. radiolucency
c. contains antibiotic 
65. secretomotor supply of parotid gland 
a. glossopharyngeal nerve **
b. trigeminal nerve
c. facial nerve
66. laser used as substitute of blue curing light??
a. Argon laser 
67. patient came with a removable partial design that is wrongly designed, the RPD has no 
occlusal rest, what is this type of RPD called??
a. gum stripper 
68. a finger spring was used to correct a mild crossbite of upper lateral incisor, in which of the 
following situation is it preferable to correct a crossbite with a finger spring??
a. lingually tilted lateral incisor, 5%overbite*
b. lingually tilted lateral incisor, 50%overbite
c. labially tilted lateral incisor, 5%overbite
d. labially tilted lateral incisor, 50%overbite
69. technician places a cellulose liner before investment of a crew, what is this step done for??
a. compensates for metal expansion 
b. allow space **
70. a dentist after finishing a crown preparation makes a seating groove as a final step before the
impression, what is this used for??
a. helps seating the final restoration***
b. guides in impression taking 
71. you ask a patient for an OPG radiograph, but the radiograph she brings has obliterated 
apices of all maxillary tooth, you want to repeat the radiograph what should be done to avoid the 
first mistake?? 
a.
72. Herpetic infection is caused by??
a. Herpes simplex virus type – 1**
b. Herpes simplex virus type­1
c. Epstein barr virus
73. A Study comparing used to find cholera carriers in three different cities. Virtually all people in 
the city participated. What is the design. At the end of the study proportion of the carriers were 
calculated and compared
a. case control study
b. Cross sectional***
c. Cohort
74. long case, fine needle aspiration show yellowish shiny fluid, what will be your diagnosis??
a. Dentigerous**
b. aneurysmal bone cyst 
75. question about dentigerous cyst 
76. question about denture stomatitis 
77. question about rubber dam
78.question about case control study picture of nasopalatine injection
79. x­ray of internal resorption
80. picture of automatic torque wrench used for implants 
81. picture of nasopalatine injection
Exam 29
 Bone removal during alveoplasty
A. Bone file **
B. Osteotome
C. Rongeur
2. Maximim amount of Anesthesia 80kg patient lidocaine 2% + epinephrine 1:100,000
Answer: 7*80= 560mg
3. Complex in 18 year old healthy patient: purple
4. Patient going for ortho consultation, no crowding, history of trauma to anterior teeth
Which class? 
1. I
2. II div 1 ***
3. II div 2
4. Class 3
5. Fatal fluoride toxicity for a 3 year old with time ­ 5mg/kg**
6. Amount fluoride 5 year old, 0.5ppm water fluoridation
A. 0.25 ***
B. 0.5
C. 1
D. 1.25
7. Color saturation: chroma
8. Hold epulis fissuratum with: Allis forceps
9. What is maryland bridge?
Answer: Resin bonded
10. Child with lower canine infection needs extraction ­no crowding­, what do you do?
Answer: extract two canines without space maintainer
11. Tmj innervation: auriculotemporal nerve
12. Distance between 2 implants: 3mm
13. Crown root ratio if crown:10mm and root:15mm? 
1:1
1:2
2:3***
2:1
14. 3 yr old with 5mm intrusion of upper incisor what do you do?
1. Wait and see if erupts ***
2. Extract + space maintainer
3. Crown lengthening
4. Reposition manually and splint
15. Type of ortho force for teeth with compromised periodontium: light
16. In primary teeth, before putting filling you do: Proper cleaning and drying
17. How to get rid of blood, suction fluid waste: drain connected to sewage
18. Cleft lip result of:
Malunion of medial nasal and maxillary process
19. Maxillary growth:
Tuberosity + nasal + downward and forward
20. Best for disc perforation: arthrography
21. Child fell on chin with both condyles broken but all is normal (no malocclusion, all movements
are fine) treatment:
No treatment
22. Suspect fracture right condyle, xray? reverse town
23. Patient pain on biting (relief when open) related to tooth with big amalgam restoration, 
confirm diagnosis by? Tooth slooth
24. Patient pain on biting with deep pocket related to one rootSuspect? Vertical fracture**
25. Patient dull pain related to tooth with endo since 3 yrs ago. Short endo+ radiolucency and 
restoration leakage.. reason for failure?
Short endo + inadequate filling**
26. Child came with pain and fatigue + multiple vesicles on hard palate (last year he had the 
same and went away after one week). Diagnosis?
Herpes simplex (recurrent)
27. Streptococcus mutans in? Deep pit and fissure
28. Angle of instrument with tooth? 45­90
29. Advantage GIC in carious mouth: fluoride release
30. Restoration needs glaze to prevent dehydration?
A. GIC ***
B. Porcelain
C. Composite
31. Adult Patient with broken tooth between middle and apical third, managment?
Endo for coronal part
32. Patient adviced to brush her teeth frequently to prevent calculus, why? To break plaque layer
33. Disinfect gutta percha by: sodium hypochlorite
34. Gutta percha formed of?
70% Zno Eug and 20% gutta percha
35. Difference btwn standerdized and conventional gutta percha (forgot answers)
36. Question about the strip that changes color when autoclave, what does it mean?
A. Object subjected to steam heat
B. Object sterilized*
37. Why do we clean instruments before sterilization?
A. Less time sterilization
B. Preserve instrument integrity 
C. Kill protein**
D. Kill spores
38. What type of disease you can work with at clinic? 
A. Conjuctivitis
B. Chicken pox
C. Hepatitis C**
D. Something like chicken pox
39. Diabetic with pain and abscess and allergy to penicillin:
A. Start working and prescribe clarithramycin ***
B. Prescribe clarithramycin then start work
C and D had penicillin
40. Anesthesia with less pain
A. Topical
B. Stretch tissue
C. Inject slowly
D. Needle gauge more than 25
And there were options to choose from, i choose A,B,C**
41. What anesthesia has most vasoconstrictor action?
A. Cocaine**
B. Tetracaine
C. Procaine
D. Articaine
42. Action of smear layer in pulp protection?
A. Dsnt allow toxins reach pulp/88
B. Less effect of etching on pulp
43. Old patient with sharp severe pain on trigger and doesn't cross midline: trigeminal neuralgia
44. Which tooth has root that is mostly pushed into sinus when extraction?
Maxillary first molar
45. Which fibers are teared first when extraction?
A. Transceptal
B. Circular
D. Dentoalveolar **
E. Cementoalveolar
46. Which smooth surface is mostly affected with high caries patient?
A. Labial maxillary incisors
B. Palatal maxillary incicors
C. Buccal maxillary molars ***
D. Palatal maxillary molars
47. Tongue tie treated when? When limitation of tongue movement
48. Positive rake
A. K file*
B. K reamer
C. ProFile
D. Protaper
49. Case about patient with porcelain teeth in upper denture and now needs lower denture, what 
type of teeth we use? I chose porcelain (there were other options like porcelain+gold or acrylic)
50. Why do gingivectomy? To remove pseudo pocket
51. Why use low speed with pedo?
Less pulp exposure
52. Baby with Lesion anterior maxilla: congenital epulis
53. When to do re endo if exposed to oral fluids?
6 days (there were other options)
54. Something with epoxy resin, what type of impression?
A. Polyether**
B. Polysulfide
C. Additional silicone
55. Most difficult group of teeth to anesthesize? Mandibular molars
56. Centric relation? Bone to bone
57. Bleeding from nose with doubt of cerebrospinal fluid? 
Glucose oxidase formation
58. Patient with esthetic concern having dark central bcz of trauma treatment?
A. Crown
B. Veneer
C. endo
D. Endo+internal bleaching**
59. Most size suture material in dentistry? 3/0
60. Patient came for restorative work with Blood Pressure 200/160 treatment
A. Anti stress
B. Anesthesia without epinephrine
C. Nothing, it's just chair stress
D. Reschedule and send him for physician consult ***
61. Property of material that doesn't distort with tensile stress
A. Ductility**
B. Malliability
C. Solubility
62. Child multiple white spots, inflammed gingiva, hight plaque, probing depth 2­4mm
Periodontal diagnosis?
A. Gingivitis ***
B. Jeuvenile periodontitis
C. High caries index
63. Prophylaxis for patient with bacterial endocarditis:
A. Ortho band**
B. Routine dental cleaning
C. NON ligamentary injection
D. Suture removal
64. Amalgam copper: 13%
65. Median rhomboid glossitis with patient wearing denture cause? Candida
66. Type of approved probe for exploring pit and fissure sealant?
Blunt
67. Most common type of occlusion comfortable for patient and easy to fabricate?
A. Mutually protected**
B. Unilateral
C. Bilateral balanced
68. When xray cone is not properly pointed what happens?
A. Cone cut**
B. Overlap
C. Elongation
D. Shortening
69. Patient low caries worried about brown discoloration of premolar (picture) diagnosis? Stained
groove
70. Interdental brush with? Type II embrasure
71. Endo central minimal with mesial caries. Restoration?
A. Composite**
B. Fiberpost + mesial composite
C. Cast post/crown
72. Name of injury caused by normal/abnormal occlusion on a periodontally compromised tooth?
A. Fremitus
B. Primary occlusal trauma
C. Secondary occlusal trauma**
73. When to decide Composite shade?
A. Before rubber dam placement**
B. After cavity
C. After bonding
74. Best way to know pulp status and integrity?
A. Cold test**
B. Electric test
C. Anesthesia
D. Percussion
75. Case picture lesion, patient all fine but he had trauma to the area (he bite it traumatically). 
Diagnosis?
A. Fibroma**
B. Papilloma
C. Hematoma
76. Case 12 yr old child swelling in area 44 45 (missing 45) xray radiolucency around crown 45 +
radioopaque specks. Diagnosis
A. Dentigerous cyst**
There were other options i dnt remember
77. When sharpening curette angle? 100­110
78. Cast post and core try in
A. Seat With pressure
B. Seat Without pressure**
C. Rotate
79. Irreversible pulpitis bcz of?
Congestion of blood vessels in pulp
80. After gingivectomy, surface epithelialization occurs after?5­14
81. Difference btwn radicular cyst and ossifying osteitis
82. Compmer is? (Composite resin + polyacid i think)
83. Porous denture managemet
A. Reline
B. Rebase***
C. Remake
84. At age of 10 what teeth are in the mouth?

Exam 30
best material for direct pulp capping in children – CaOH

2. In seibert classification , apicocoronal loss of tissue ­ class ii
Seibert , 1983 classified , Class II is ? 
a.buccolingual loss of tissue with normal apicocoronal ridge height 
b. apicocoronal loss of tissue with normal buccolingual ridge width**
c. combination­type defects

3.drug causing gingival enlargement ­ PHENYTOIN**

4.Scenario asking drug for oral and esopharyngeal candidiasis ­ fluconazole**

5. bluish swelling on gums of 14 month old child ,which is fluctuant and not tender, missing teeth 
­ eruption hematoma**

6. 22 years old patient complaining of recurrence oral ulcer with history of
congenital and ocular lesion what could be the diagnosis?
A. Chron’s disease
B. Bohens syndrome
C. Behçet's disease***
D. Sjogren's Syndrome

pt. with lesion in upper lip red border and yellow base, and said he has joint 
pain and optic lesion:
• reiters syndrome**
• behcet's syndrome

7. scenario saying multiple OKCs and basal cell carcinoma ­ gorlin goltz syndrome********
Old patient had displaced teeth his father has lesion and removed long time ago panoramic x­ray
show multiple unilocular lesions in upper and lower anterior and posterior teeth with displaced 
but no resorption :
Basal cell nevus syndrome (gorlin goltz syndrome)*****
Cheribism

8. non adjustable articulator ­ class 1***, class 2, class3, class 4
Type 1- non adjustable simple hige articulator allow only vertical movement
Type 2- allow vertical and horizontal movement
Type 3- semi adjustable allow horizontal vertical and condylar movements
Type 4 fully asjustable allow 3 dimentional movement

9.which nerve is anatomically related to pterygpalatine ganglion –
 maxillary nerve********
mandibular nerve 
ophthalmic
facial Restricted coronal retention

10. important biochemical property of single implant ­ abutment made of titanium*********

11. most biocompatible base for pulp and pdl tissues ­ Zn polycarboxyllate****

12. shape access cavity of max. premolar with two roots and two canals ­ ovoid*****************

13.bone death during bone drilling for implant ­ 46 c for 1 min

14. A 21yr old boy c/o' SHIFTING TEETH' present with enlargmnt of mandible..also give history 
dat his fathr had a leison and surgically removed frm jaw..wit R/g showing radiolucent leisons on 
both side ofmandible and on anteriors too..
Keratocystic odontogenic tumor
Ameloblastoma*
Dentigerous cyst.
Radicular cyst
15. dentigerous cyst

16. after crown lengthening you wait 5­6 months before crown placement for –
 maturation of pdl************

17. evaluation of progress of improved pdl heath ­ attachment level***********

18. which of oral conditions should be of great concern?
dysplasia **
metaplasia **
hyperplasia
hypertrophy

19. Instrument used to trim excess material from gingival margin for class V ­ File 
chisel
knife
carver**

20.which one has positive rake angle­ ­ k file**, k reamer, protaper, profile

21. mass in posterior part of tongue .need to do biopsy .which instrument used to retract tongue 
while sectioning for biopsy – towel tip 

22. asthmatic attack . which LA ­ Epinephrine sub cutaneously 1:1000

23. growth of maxilla primarily by ­ inter membraneous ossification**
anatomical land mark in skeletal open bite
is:
a-mild facial convexity
24. antibiotic that inhibit cell wall biosynthesis ­ penicillin******* b-decrease height of ramus and posterior
teeth**
25. medication for juvenile periodontitis ­ tetracycline  Tx : intrusion of posterior

26.angle between face blade of universal curette and sharpening stone ­ 100 ­ 110*********
27. pic of root caries 

28. pain and swelling below tongue which increase while eating – 
submandibular sialolithiasis*************

29. thick labial frenum and wide vestibule. How to correct for placement of denture.
Vistobuloplasty **** (( if narrow frenum ))­­­ z plasty 

30. antibiotic prophylaxis for congenital heart disease for dental treatment – 
Amoxicillin 50mg/kg by wt 1 hr before procedure*******

31. best ferrule effect for fracture of tooth at alveolar crest ­ orthodontic extrusion*****

32. cause of skeletal open bite ­ discrepant in vertical ramus growth and posterior teeth

33. amount of expansion achieved by rapid maxillary expansion of helix( depend on patient 
activation ****
34. zinc containing amalgam placed subgingivally in class 2, pt came with pain after few days ­ 
delayed expansion*************

35. diameter of GG#1 in iso ­ 50***

36. pic of facebow asking which type ­ ear, kinematic, condylar, mandibular( I think kinematic 
...not sure)

37. pt under oral hypoglycemics needs extraction. did finger stick blood sugar test. what should 
be the blood sugar level to perform extraction ­
less than 85 
85 ­200( I choose this)
200 ­396

38. cusp which is non carious, not restored, brief pain when subjected to cold ,what is pulp status
­ reversibly inflamed 

39. scenario showing readings of ept, cold test, heat test and percussion of symptomatic tooth 
and control tooth. status of pulp of symptomatic tooth ­ reversible pulpitis***************

40. blade used for incision of intraoral abcess ( slightly big)­ #11****, #15, #12, #10

41. relation in which working end of instrument is placed against tooth­ adaptation********

42. water fluoridation prevents which caries ­ occlusal, interproximal, smooth surface***, pit and 
fissure??

43. procedure in which inflamed pulp tissue beneath an exposure is removed 1­3 mm deep ­ 
pulpotomy
partial pulpotomy**********
direct pulp capping 
indirect pulp capping

44. 5 yr old child lost upper left central after trauma. management?
no treatment**
space maintainer
Maryland bridge
polycarbonate crown

45.dentist wants to pour impression twice . which impression gives accurate records even when 
poured twice?
agar 
addition silicone
polyvinyl siloxane **********
polyether

46. ideal time for chlorhexidine mouth wash­ 15 sec , 30 sec**, 45 sec, 60sec

47. There is a wide gap between the delivery and demand in dental care.
a. emergency and dental treatment
B. puplic education of self dental care ***
C. resources of dental treatment 
D. distriputions of resources

48. screening test for hiv
ELISA*********
western blot
montoux
immunofluroscent assay

49. Mongolism is 
disomy21
trisomy 21*****
two more options

50. xray for caries approaching pulp
occlusal
bitewing
periapical********
opg

51. 12 yr old boy missing some teeth. xray to view all permanent teeth ­ panorama******

52. According to shillingburg whats the order of shade selection
1. Chroma. Value. Hue
2. Hue. Value. Chroma
3. Value. Chroma. Hue**
4. Value. Hue. Chrom 

53. caries progression in adult less rapid in children 
­generalised dentin sclerosis by age********
54. proxy brush for ­ type2 enmbrasure
55. dentin conditioning best by
37% phosphoric acid for 10 sec
37%phosphoric acid for 30sec
10%polyacylic acid for 10 sec***
10%polyacrylic acid for 30 sec
56. scenario of symptomatic irreversible pulpitis

57. primary objective of gingivectomy in nifedipine inducedgingival enlargement ­ eliminate 
pseudo pockets**

58. space between upper and lower teeth in physiologic rest position­ freeway space***********

59.hypernasality voice due to
palatal perforation***
torus
two more options

60.post least tendency to fracture ­ fiber post

Exam 31
1/case described and diagnosis is most likely sialadenitis but patient has allergy to Iodin 
containing what kind of diagnostic image taken:
a­ normal Sialography
b­ CT
c­ MRI***
d­ Occlusal x­ray
2/ In school a program for teaching the kids how to brush their teeth with verbal and theoretical 
teaching so they can brush by them selves what is that type of teaching:
 Demonstration**

3/ Dental caries is an endemic disease means that the disease:
1. Occurs clearly in excess of normal expectancy.
2. Is habitually present in human population.***
3. Affects a large number of countries simultaneously.
4. Exhibits a seasonal pattern.
 
4/Def of pharmacokinetic? 
Is the effect the body has on the drug include absorption distribution metabolism and excretion of
drugs .
5/Det od epidemiology?
­population survey of disease \ survey the disease \ general nature survey 

6/Anesthesia for first molar?
7/Number of canal in lower first molar?3
8/Number of canal in upper premolar?2
9/Cord colar for 4%prilocaine  with 1:200:000? yellow
8. PH of
10/Female pt. Comes with endo treated upper central with M & D caries and have incisal 
abrasion. Porcelain veneer is planned with modification to cover incisal edge. Veneer should 
end:
a. Fourth lingual 0.5 mm before centric occlusal.
B. Fourth 1.5 mm before centric occlusion**
c. Fifth 1.5 mm before centric occlusion.
11/the recurrent infection in diabetes is due to the destruction of which cell 
Neutrophils **
Leukocyte
Macrophages 
Immunoglobulin
.12/Pleomorphic adenoma its size is 1.5 x1.5 cm on posterior part of hard palat, what is ttt : 
a.Enucleation only 
b.Radiotherapy 
c.Chemotherapy and enucleation 
d.Resection of periostuim.***
13/Child come to clinic with total reverse of upper anterior teeth sometimes child discease his 
chin & lateral cephalometric give this results
Sna 80
Snb 82
Anb ­2 what is diagnosis:
A) Class l malocclusion with skeletal class lll 
B) class lll malocclusion with skeletal class l
C) class lll malocclusion with skeletal class ll
D) class lll malocclusion with skeletal class lll***
14/Paranasal fluid occurs in:all fracture exp zygomatic fracture
15/Anesthesia for 1 molar?
16/Posterior 1/3 of tongue nerve supply? Glossopharyngeal 
17/Pt loss teste in anterior part of tongue due to injury in?? 
Facial nerve **
Mental nerve 
Posterior Superior alveolar nerve
18/After ext lower second molar pt loss of sensation due to injury in??
19/Cod of anesthesia?
20/Which Bacterial cause of acute periapical periodontitis?? Polymicrobal anaerobes
21/Cell in chronic inflammatory? Chronic in ct drain to siuns tract
22/Histology of Congental syphilis??
23/‫كانت‬ ‫الخيارات‬ ‫البتوجعو‬ ‫عضله‬ ‫اي‬ ‫رقبته‬ ‫ف‬ ‫اليسار‬ ‫يدو‬ ‫حط‬ ‫اليسار‬ ‫جهه‬ ‫ع‬ ‫راسك‬ ‫حرك‬ ‫ليهو‬ ‫وقلتا‬ ‫بتوجعو‬ ‫رقبتو‬ ‫مريض‬ ‫??جاء‬ 
1/Right Sterno­cleido mastoid **
2/left Sterno­cleido mastoid
3/gastric
24/Maximum distance for lingual holding arch:
A.2 B.3 C.4**. D.5
25/patient with eroded palatal surfaces of upper anterior teeth what's the possibility reason?
1.Peptic ulcer
2.anorexia nervosa**
26/patient female 38 y old come to u for check up,with radiograph u found multiple radiolucency 
in anterior area periapicaly,no caries no pain wt cause?
A.periapical cemental dysplasia***
Pic cephalometric analysis pant with crowded lower jaw ANB+5
B.fibrous dysplasia
1. Class3 with skeletal class2
C.apical periodontitis 2.class3with skeletal class3.
27/Cephalometric analysis‫؟؟‬ 3.class2 with skeletal class2*

28/Pt with crowded lower teeth ANB 7?
29/medication cause gingival enlargement ?
a­phenytoin
30/two guestions with images for kennedy classes
31/patient came to you and you give him inf nerve block and he came to u the day after 
procedure with trismus from anethesia which space was affected ?
a­sub mandibular 
b­sub massetric
32/patient has mild stain by tetracycline what will you do ?
a­home bleach **
b­micro abrasion 
c­polishing
33/patient with badly decayed upper left central with swelling buccal to the buccal vestibule what 
is the x­ray u will do ?
a­periapical **
b­panorama
34/epinephrine in endo surgery with anethesia ?
a­prorlong time *
b­vasodilator to the tissue
35/pic with anesthesia tech ?
­inf nerve block
36/yrs child with swelling due to badly decayed lower right c with no crowding ?
a­extract canine 
b­extract the two canines**
37/What is most favorable place for streptococuc mutans:
a­pits and fissure **
b­proximal
38/Lefotr 1 injury
a/greater platine artery***
b/infra orbital artery
c/maxillary artery
39/In order to activation of periodontal instruments the blade should make angle with facial 
surface of the tooth 
a­ 45:90 **
b­ 90:180
40/ Most retentive crown is 
Full metal **
Veneer
41/ 4 yrs child has trauma cause intrusion 
a­extraction 
b­dont touch it for comfortable**
42/dentist at the end of the day want to pour alginate imp quikly how can he do that
a­increase powder/water ratio
b­hot water**
c­slurry water
43/chair time relining of denture 
a­wax
b­light cure acrylic **
c­>>>>
44/patient with prothetic valve before surgery 
a­prophylactic AB
45/Ttt of kerato cyst­ complete surgical removal **
46/5 yrs child swallow bottle of mouth wash contain 13mg of flouride 
a­refer to emergency immediately 
b­make him drick milk or some thing i dont remeber contain ca
47/patient can not localize the pain and u decided to do intraligament injection what is the firt 
place u will inject ?
a­up anterior
b­lowe ant 
c­upper post ***
d­ lower post
48/ X­ray of parotid gland if pt has iodine sensitivity 
A silogram **
B MRI
C CT
49/.long story about blade no.of stap incision for abcess drainage?
11**_12_15
50/.Pic. of lower complete over denture asking about decrease retention
51/pic.of diabetic patient with xerostomia??
caries,*** erosion,abrasion
52/non carious lesion may related to occlusal defect?? 
attrition,erosion,abfraction**
53/x­ray of 22 y old pt with midline shift in lower jaw with history of jaw lesion removal? 
dentegrous cyst
Ameloblastoma**
Odontogenic ker.tumour
54/pic of cephalometric which classification??
55/diameter of pulp arteriol?? 45­55 MICRONE ((50))
56/degree of enamel demineralisation??? PH 5­5.5
57/managment of cardiac pt in clinic with chest pain?
nitroglycerin**
other medication
58/long history of pt with notched central at Which stage syphilis affect teeth??
initiation
proliferation
histodifferentation
morphodifferention**
59/long story asking about flap retractor??
Minnesota
60/color of prilocaine cartidge?? yellow
61/Anaesthesia for pt with amide allergy??
lidocaine
Prilocaine 
Mepevicaine
Diphenhydramine hydrochloride ***
62/pt of mandible ask about suitable No. of implant regarding cost?
2**,3,5
63/Muscle of mouth opening??
masseter
Digastric
LATERAL Pterygoid**

Exam 32

1. What type of bone graft is used in treatment of furcation type II?? Corticocancellouse
2. What is the recommended amount of fluoride per liter?
• 0.2­0.5 mg/L (this was my answer bcoz we should drink at least 2L of water per day and 
recommended F dose is 1 mg/day)
• 0.5­0.1 mg/L 
• 1­1.5 mg/L
3. 12­year old patient with badly carious 36 & 46, you decide to extract. What do u do after the 
extraction?
• Treatment RPD (I chose this but not sure of the answer)
• Interim RPD
• Band and loop
• Another choice (forgot)
4. 28­year old female pt with missing or extracted 11, 12 and 21 are sound. What is the best txt 
option for her?
• Implant (I chose this because she is young, implant is the best choice, and the other teeth are 
sound)
• FPD 
• Other choices
5. Best in high caries individual
• Restoration after instructions 
• Instruction after restoration
• Temporary filling after scaling
5. Patient wearing upper CD for a few years, he has lower anteriors remaining and wants to do 
rpd for lower arch. What will you find in this patient?
• Maxillary anterior bone Resorption 
6. Mesodermal origin (something like that) of mesial part of upper lip is formed from:
• Medial part of maxillary something
• Frontonasal …
• 1st branchial arch
7. Tx of ranula:
• Marsipulization 
8. Posterior 1/3 of tongue is innervated by:
• Glossopharyngeal nerve IX
9. Difference between acute apical and periodontal lesion
• Vitality
• Size
• Pain
10. Anticoagulation test done for patient taking warfarin is:
• pt (chose this)
• aptt 
• Plasma level
11. Question about positive re­enforcement
12. Substantivity is a property of a material that attaches itself to multiple areas in the oral cavity 
and has antibacterial (or anti something ) effect that releases slowly throughout the day:
1. CHX*
2. fluoride
13. Nitrous oxide affect which vitamin
• B12
14. Type of cementum in coronal 2/3rd
• Acellular afibrillar
• Acellular extrinsic fiber**
• Cellular mixed stratified
15. When should the parents take their child to the orthodontist?
16. While giving anesthesia to 27 reaction happens..what structure was affected?
• Pterygoid plexus vein
17. Female pt with hx of trauma many years ago in central incisor, it
is dark in color. Tx :
• RCT + internal bleaching (my answer)
• Crown
• RCT
• Fpd
18. Question about Chroma..something like ceramic tooth, u want to make the cervical darker.. 
so I chose chroma coz it has to do with color saturation.
19. Functional orthodontic device ( I chose post bite block)
20. Type of dentin formed in response to caries: sclerotic dentin
21. For grade 2 furcation involvement what type if bone graft:
a­All are good 
b­Cancellous decorticated 
c­Cortical decorticated 
d­Freeze dried bone
22. What is the goal of maintenance therapy 
a­ To prevent recurrence of disease
b­ Evaluate tissue response
23. Anesthesia for maxillary primary molar which nerves: 
• post sup alv and greater palatine
• post sup alv and nasopalatine
24. While doing RCT, you penetrated the furcation area of roots what u will do?
• Mineral Trioxide Aggregate
25. Periodontal abscess is more common in which tooth:
• upper premolars
• incisors
• canines
• molars
26. Hypertensive patient on medication for 10 years, he come for complete denture construction. 
Bp is 145/100 What will interfere with the CD?
• dry mouth (my answer)
• mucosal changes (gingival recession)
27. Anticariogenic sugar substitute is : 
• Xylitol
• Mannitol
28. The tip of size 20 endo file is: 
• 0.02 mm.
• 0.2mm
29. Male pt who wears rpd for 5 years replacing lower molars, he complains that he cant tear his 
food properly. When placing pressure with finger on each side of the rpd, the opposing side does
not move (no rocking), what is the solution?
• Rebase/reline/or make a new denture? (I chose make a new one)
30. Laser used as substitute of blue curing light?
• Argon laser 
31. Picture of impression with implants, asking about the material around the impression coping :
gingival mask
32. Picture of band and loop 
33. Radiograph of well defined RO lesion near to the apex (but not attached to) of the lower 1st 
molar, the question was what is the lesion:
• osteoma 
• odontoma
34. Proximal caries in enamel of mesial side of 25. Non­cavitated. What is your tx:
• class II amalgam
• restore with composite (my answer)
• no tx
35. Amalgam filling on deep cavity. Sequence of applying materials :
• caoh, gic, varnish, amalgam 
36. Anti­cariogenic food..:
• high mineral content (my answer)
• high protein
37. Patient with yellowish based small ulcer and erythematous border, joint pain and eye lesion
• bahchet disease
• riter syndrome 
38. Most accurate pulp vitality test: 
48. cold *
49. hot
50. electric
51. cavity
39. Electric test measure the response of:
• Nerves *
• Pulp
40. During endodontic treatment the file broke ,when we can leave it and have best prognosis: 
• long part away from working length at early stage of mechanical preparation. 
• short part near the working length at early stage. 
• long part near the working length at late stage.**** 
• short part away from working length at late stage.
41. While anesthetizing upper molar, it turns white etc etc.. what is affected: 
Plexus vein
42. Old patient with severe resorption of lower ridge and sublingual glands are prominent ,wants 
to make complete denture, which type of impression is best: 
• Muco­compressive.  Old female patient with resilient
• active impression mucous membrane on the
• dynamic impression **** alveolar ridge need construction
of denture. What is the technic
43. 90% of fluoride is contained in : calcified structures of the body (bone) of impression?.
44. Leeway space in mandible per side: 3.5 a. Dynamic
45. Wax pattern carving grooves  b. Muco-compressive**
• pkt2
• pkt4 ****
46. Diabetic pt taking his meds, what is the acceptable blood sugar values to work on? 
1. 85­200 mg/dl (there was other options but this was my answer..the others seemed too high)
47. Pt has swelling from the sublingual gland related to a lower molar. You decide to make an 
incision. Where will you make the incision? 
→If the space contains pus, the usual treatment is by incision and drainage.
The site of the incision is intra­oral, made lateral to sublingual plica OR Intraoral parallel to 
Wharton's duct.
1. Why do older individuals have generally less caries than children?
2. Generalized sclerozing of dentin with age
3. What is the disadvantages of mcspadden technique in obturation:
4. Difficult in curved canals
5. 48. 80­year old patient want to give LA 2% with epinephrine. What is the dose in mg? 
6. 560 mg (80x7)
7. 
8. 49. Question about DMFT score, something is not accurate about this score..what is it? (forgot
the options)
9. 50. Female with only half of her face involved, mandible enlarging and spacing b/w molars.. 
what does she have?
10. Gigantism
11. Acromegaly 
12. Hemi something Hemifacial hypertrophy
13. 51. Patient with renal failure, what blood tests do you do?  Serium kertainin
14. Pedo pt with kidney & liver transplant, what is present in his mouth? I chose Periodontal 
disease(or changes) the other options were gingival recession..
15. 52. The base of en enamel caries is directed towards:
1. DEJ
2. Enamel Surface
3. 53. The particles in enamel which give its shiny appearance: Hydroxyapatite (my answer)
4. 54. 80­year old pt with bad oral and denture hygiene. Smoker. When he removes the denture 
there is a red lesion under the denture, what is it?
5. 55. 80­year old patient received denture 2 months ago presents with severe peri­auricular 
pain, severe wrinkles around corner of mouth. Pronunciation problems with some words. 
Mastication problem, reduced facial muscle tone. What is the problem?
6. Low vertical dimension
1. High vertical dimension
2. 
3. 56. patient who use to chew “Shams” but stopped a few years ago, has white lesion on the 
cheek..what is the Dx? Carcinuma situ
• Verrucous carcinoma 
4. 57. Female patient with gingival enlarged lesion on CI, what is it?
• Pyogenic granuloma (aka granuloma gravidarum)
Exam 34
1)FPD in mandibular molar , the preparation is short of lack of retention , we want to do 
provisional crown using zinc oxide eugenol cement what to do :
a. Thick , creamy mix of the cement***
b. Add petroleum to the cement
c. Maintain dry field until cement set
d. Remove hardened part of cement in interproximal area by using explore
2: Gingival margin trimmer used in
A. Finishing class5
B. Beveling class 2***
3: Which contraindicated with sickle cell anemia
A. Aspirin ***
B. Acetaminophen 
C. Local anesthetic containing vasoconstrictor

4:Last step in the insertion of PFM crown?
a Glazing of porcelain
b Polishing of metal***
c staining of porcelain
d contouring
5: When using buccal object rule where will lingual appear if we move cone mesialy
A distally
B. Mesialy***
C. Buccally
D.lingualy
6: Distance between two implants 
2mm
3mm***
4mm 
5mm
7:Cavosurface angle for amalgam
60
90***
70
110
8: Optimum crown to root ratio for fpd
1:2
2:3***
3:2
1:1
9: 8years Patient came to your clinic has impaired hearing, upon examination his mouth you 
found copper color lesion, notched incisor and mass on the occlusal surface of the molars. This 
patient has:
a) Gardner syndrome.
b) Congenital syphilis. *****
C turner hypoplasia
D Gorlin Goltz syndrome
10: At which age parent should go to the orthodontist for consultation:
1. When all permanent teeth erupt
2.when permanent centrals erupt***
3. when there is spacing present
4. when there is crowding present
11: Zinc polycarboxylate cement is better than zinc phosphate cement in:
a­ Compressive strength.
b­ Low solubility.
c­ Film thickness.
d­ More biocompatible ****
12: Root perforation is treated by:
1. MTA. ****
2. Ca (OH) 2.
3. Root canal with GP
.
13: The adverse effect of orthodontic treatment commonyl seen after treatment 
A enamel hypocalcification***
B root resorption
C tooth relaps
14: After trauma tooth is partial displaced coronally injury is 
A:intrusion
B:extrusion***
C Subluxation
15: Allergy from Methyl Methacrylate used in dental laboratory results in
a. Contact dermatitis***
b. Type I hypersensitivity reaction
c. Lung irritation and respiratory problems
D eye skin and mucusal irritation
16: During final inlay cementation which of the following you will do?
A. Polishing
B. Remove occlusal interferences 
C. Lowering occlusal surface.
D. Burnishing of peripheries of restoration ***
.
17: Old pt. has some upper molars overerupted, While lower jaw anterior has gingival recession, 
degree of mobility & Kennedy Cl 1:
a. acrylic RPD with occlusal adjustment of upper over erupted teeth
b. swing lock also with adjustment***
c. over denture
18: Pt came for prosthetic treatment , she has missing #24 #26#27#28 and #25 is left as peir 
abutment , it has lingodistal inclination , Mesiooccusal amalgam restoration and grade II mobility, 
what is ur treatment?
A­Extract #25 and RPD***
B­FPD on 23­25 and RPD on 26 27 28
C­Overdenture RPD on 25 
D­RPD
19: In complete dentures denture adhesive usually requires for which typenof palatal valut
A u shaped valut*
B flat
C v shaped
D ovoid
20: The degree of taper of crown prep. :
A. 3­5
B. 15
C. 6***
21: Type of orthodontic force applied to teeth with compromised attachment level:
a­ tipping
b­ light***
c­ heavy
d­ intermittent
22: Child has mild Tetracycline discoloration in permanent tooth what is the proper treatment:
1. Composite veneer
2. Home bleaching***
3. Pumice micro abrasion
4. Porcelain veneer
23: Hyperplasia of nerve fiber 
Calcitonin...
Prostaglandin + serotonin ****
Lysosomes enzyme 
Substance p...
23: When first expose child to use dental brush..?
A. Eruption their primary teeth***
B. At age of 2 y
C. At age of 5 y
D. When he entered primary school
25: Dental materials are classified as
A. ceramic, polymers and composite
B. ceramic, polymers and alginate.
C. ceramic, polymers. Cement
D. ceramics, metals, polymers,composites****
26: In primary tooth for restoration before putting the filling u put:
a. base.
b. calcium hydroxide.
c. varnish.
d. you put the filling after proper cleaning and drying.***
27:After usage of sharp scalpels, needles what's the best management:
1) throw in a special container of sharp instruments. ***( Sharp container ).
2) sterilize and reuse.
3) through in ordinary plastic waste basket.
28: The sequence in deep carious lesion close to pulp are 
a. GI base, varnish, caoh2 
b. Varnish, GI base, caoh2
c. caoh2, GI base, varnish****
d. caoh2, varnish, GI base
29: Patient come with complaint of loose denture when press Complete Denture, it bubbles in the
posterior part near the palatal seal, and good retention in the anterior Vestibule and Buccal 
vestibule, what is the problem:
A) Over post dam
B) Under post dam+++
C) Over extended
D) Under extended
30: Brushing technique in which sides of bristles takes active part
Modified stillman***
Charters..
Bass
31:Patient complains from pain in TMJ. During examination you noticed that during opening of 
the mouth mandible is deviate to the right side with left extruded. Diagnosis is:
a) Condylar displacement with reduction.
B) Condylar displacement without reduction. ***
C rheumatoid arthritis
32: the CBCT have the following property
a­ best to show TMJ disk*** 
b­ expose the patient to large amount of x­ray 
c­ use for routine radiographic examination
33: patient with interproximal caries detected by bitewing radiography, you want to detect caries 
that reach the pulp, what to use:
a­periapical****
b­bitewing with another angulation.
C occlusal
D Panorama
34: The water rins devices for periodontal therapy has a main goal which is:
a­ remove plaque
b­ prevent plaque attachment
c­ dilute bacterial toxin**
d­ remove dental pocket
33: Edentulous pt. class II kenndy classification 2nd premolar used as abutment when we 
serving
we found mesial undercut what is the proper clasp used:
1/ wrought wire with round cross section. ***
2/ wrought wire with half round cross section.
3/ cast clasp with round cross section.
4/ cast clasp with half cross section
34: Zirconium post has:
a­high compressive strength and low tensile str.
B­ low compressive str. And low tensile strength
c­High compressive str. And high tensile str.*****
D­ Low comp low tensile
35: What is meant by hyper­apnea
A increase in rate and depth of breathing ***
B increase in rate of breathing 
C increase in depth of breathing
D kussmaul breathing
36: Most commonly used blade use in oral surgeries 
Blade # 11 
Blade # 12
Blade # 10
Blade # 15****
37: most important biochemical consideration in implant:
a. abutment is made from titanium alloy.****
b.abutment be in 2 parts
38: Most biocompatible alloy when used intra oraly
A cr­co alloy
B titanium alloy***
C nikl chromium alloy
C cast metal alloy
39: 18 years old pt. The bacterial complex present in his mouth is:
A. Red complex
B. Green complex
C. Purple complex****
40: 18 year old patient have unilateral swelling of mandible which is painful when palpate
And there is x ray pic shows radiolucency around the crown of impacted lower thrid molar
Dentigerous cyst***
Ameloblastoma
Okc
Osteosarcoma
41: Young female patient with skeletal class 1 there is spacing present in anterior teeth. Her 
mother also have mild spacing due to missing upper lateral What is cause of spacing in patient
A hypodontia***
B avulsion
C congenital
D dileceration
42: Access opening of maxillary lateral with recessed pulp
A triangular
B oval****
C rhomboid
43: Access opening of mandibular molar 
A Rhomboid****
B round triangular with base towards buccal
C round triangular with base towards distal
44: Floor of mouth is mainly formed by
A diagestric
B genioglosus
C Mylohyoid****
D stylohyoid
45: patient came to you with multiple vesicles on the attached gingiva in the area of upper 
anteriors after
having extensive dental treatment the day before, what is the possible cause of the ulcers?
a. recurrent aphthous ulcer
b. recurrent herpes gingivostomatitis***
c. allergic stomatitis
d. mucositis
46:In FPD saturation of clour what does it means
A chroma****
B value
C hue
47: The percentage of simple caries located in the outer wall of the dentin (proximal sides of the 
tooth) which left without cavitations is around: 
A 60%.***
B 40%
C 10 %
D 50%
48: Preventive resin restoration PRR is mainly used to prevent
A initial caries lesion**
B caries progression

D
49: Organism which cuases initiation and progression of caries
Streptococcus mutans and lectobacilus
Xray picture of cliedocranial dysplasia
50: best describes caries progression?
A. Cant be reversed 
B. demineralization without remineralization
C. Demineralization with remineralization**
D can be arrested
51: foramen for accessory canal due to defect
A. Dental sac
B. Hertwig sheath***
C. Dentine matrix
52: Most favourable habitat for streptococcus mutans 
A pit and fissure****
B buccal
C lingual
D proximal
53: pa ent smoking 15 ­20 cigarettes a day for 8 years he com with
complain
of pain in gingival Diagnosed as chronic gingivitis
a­gum shows inflammation as former smoker
b­less intensity than unsmoker****
c­more intensity than non­smoker
d­same as non­smoker
54: A 21 year old man comes to the dental clinic. You notice he has plaque amd calculus 
accumulation. A SCaler with elliptical motion was used in this patient for scaling. Which type of 
scaler ?
A Piezoelectric
B MAgnetostrictive,*** 
C Sonic, 
D Ultrasonic
56: Which causes more spread of air born infection
A splater
B droplets
C spills
D aerosols***
56: The remnants are rest cells of srres of
A hertwig root sheath
B vestibular lamina
C dental lamina***
D oral epithelium
57: Anesthetic testing (intraligamentery injection)is most effective in localizing pain to which of 
the following?
Specific tooth
Mandible or maxilla ****
Across the midline of the face
Posterior tooth
58: which of these is used subgingival?
hoe
hachette
gracey curette***
chisel
59: Main advantage of glass ionomer to prevent caries
A release fluoride***
B less soluble
C wear resistance 
D aesthetic
60: Preparation of cavity for composite
A depends on caries****
B should be .5 mm in dentine
C sould be in .5 mm in enaml
61: Gates gliden 
A have numbers 3 to 9
B use to prepare coronal part of canal***
C breaks when bind in canal
D made of nikle titanium
62: Patient had anaphylactic shock due to penicillin injection, what's the most important in the 
emergency treatment to do:
a. 200 mg hydrocortisone intravenous.
b. 0.5 mg epinephrine of 1/10000 intravenous. 
c. adrenaline of 1/1000 intramuscular.***
63:Mentally ill child,the best way to apply fluoride:
a. Acidulated phosphate fluoride.
b. Natural sodium fluoride.
c. Fluoride varnish. ***
D. Stannous fluoride
64: 28 years old female is advised to brush her teeth forcefully to prevent calculus
why? 
A. Break plaque layer***
B break caculus

D
65: blood ,suction fluid and liquid waste should be disposed
A in rigid container
B in drain which is connected to sewerage ***
C in plastic bags
66: 6 year old child presents with bilateral facial swelling in ramus x ray shows well defined multi 
location and molars are anteriorly displaced
A fibrous displasia
B cherubism***
C cementoblastoma
67: main reason for unilateral cleft lip
A perforation of buccopharyngeal membrane
B failure to fuse medial nasal process with maxillary process*
C failure to fuse lateral nasal process with maxillary process
68:65 year old patient presented with pain on biting in left lower posterior teeth 47 have 
temporary root canal since 6 months and 46 have amalgam fillinf since 2 years. There is 5­6mm 
deep isolated pocket on lingual and buccal side of 47 with other wise healthy periodontal tooth
Diagnosis of pain
A Micro leakage in 47
B short root canal filling in 47 
C vertical root fracture**
D micro leakage in 46
.
69: Re treatment if endodontic treated tooth operator not reach to the all length no stick filling
A.Fracture instrument
B.Mud
C.Ledge***
D perforation
70: Fracture in neck of mandible which nerve affected 
Infer alveolar
Lingual
Mental
Auriculotemporal****
71: Rubber dam qs
A.Plastic retainer to be used for teeth with recessed pulp chamber
B.Retainer having four point contact on tooth****
C.Plastic retainer give best retention on teeth with less coronal structure
72:Which of the following may cause gingival enlargement :
1. Phenytoin ( Dilantin)***
2. Cyclosporine
3. Nifedipine
4.Aspirin

Exam 35
1. Intraligamentary injection
a.don't affect pulp circulation 
b.slightly decrease pulp circulation
c.will ceases circulation for 30 minutes ***
d.will increase pulp circulation

2. After root fracture best tissue healing
a.Interproximal connective tissue
b.interproximal bone
c.interproximal bone and connective tissue***
d.inflamed tissue

3. Type of orthodontic force applied to periodontally weak tooth
a.tipping
b.light***
c. heavy
d.intermittent

4. Enamel bonding agent (EBA) consist of:
a. Unfilled resin***
b.Primer and bonding agent
c. A mixture of resins in an acetone or ethanol solvent
d.A wetting agent and resins
5. Treatment of grade 2 furcation involvement:
a.Scaling root planing
b.GTR**
6. Concentration of flouride in water fluoridation ? 
a. 2 ppm
b. 1ppm**
c. 3ppm
d.None
7. Pt. blow in the anterior maxilla has lefort 1 where to put beak of rowens disimpaction forceps 
a. Zygomatic process
b. Canine eminence 
c. Maxillary tuberosity 
d. Hard palate along nasal floor***
8. Pt. come with broken nose, nasal septum fracture .what forceps should we use in this 
situation:
a. Walsham forceps **
b. Asch forceps
c. Boies fracture forceps
9. weeping canal we use :
a.GP
b.CaOH **
c.Formcresol
10. Disadvantage of ridge lap pontic
a.Tissue irritation***
b.Bad esthetic
c.Connector break
11. Most potent vasodilator? 
a.cocaine
b.procaine**
c.tetracaine
d.atricaine
12. Diffrence between Gracey currete and universal 
1­Gracey for specific tooth area , universal for all surfaces
2­Gracey has one side cutting , universal is both sides cutting
3­Gracey cutting end offset angle is 70 currete , universal is 85
4­gracey is semicircular cross section , universal is triangular 
a.1 and 2**
b.2 and 3
c.2,3, and 4
d.1,2 and 3
13. Hands instrument used to remove the subgingival calculus are???
a.Sickle
b.Hoe
c.Curette**
d.Curved probe
14. What is the sequence of treatment planning: 
a.Restoration , surgery, perio and ortho. 
b.restoration, perio , surgery and ortho. 
c.Perio, restoration , surgery and ortho.**
15. Curing time for sealant?
a.10­20 secs**
b.20­30 secs
c.40­50 secs
d.60­70 secs
16. Most surface of teeth more susceptible to caries
a.Labial of maxillary anterior
b.Palatal of max anterior
c.Buccal of max posterior ***
d.Palatal of max posterior
17. Access opening of lower canine is 
a.Triangle 
b.Square
c.Oval**
d.trapizoidal
18. Dental materials categorized to :
a.Metals, ceramics, polymers & Cements 
b.Metals, ceramics, polymers & composite**
c.Metals, ceramics, polymers & alginate
d.Metals, ceramics, polymers & stone
19. What is the goal of maintenance therapy 
a.To prevent recurrance of disease***
b.Evaluate tissue response
20. Arrange in descending order based on importance
a.Parallelism, height, free movement , retention**
b.Height, free movement, parallelism, retention
c.Retention, free movement, parallelism, height
d.Parallelism, free movement, retention, height
21. A patient with good RCT but coronal destruction with only 1mm crown remaining and biologic
width of 1.5mm. What will the dentist do:
a. Cast post and core, crown lengthening, restoration 
b. Cast post and core, restoration , crown lengthening
c. Cast post and core, restoration , orthodontic extrusion
d. Crown lengthening, cast post and core, restoration***
22. Angulation for scaling and Root planning is ?
a.45 degrees
b.45 to 90 degrees***
c.90 degrees
d.0 degrees
23. Amount of mercury ingested daily in the body from filling
a.1 ­ 3 μgs ***
b.10 ­ 15 μgs 
c.25 μgs
24. What's the most common 
a.cleft lip 
b.cleft palate 
c.cleft lip and palate***
25. Fracture in the neck of the mandible which nerve is injured:
a. inf. alv
b. Mental
c. lingual
d. Auriculotemporal***
26. Best Root Canal Irrigant 
a. NaOCl***
b. MTA 
c. Saline 
d. Chlorhexidine
27. Most biocompatible base to pulp and periodontal tissues
a.Zinc polycarboxylate***
b.Zinc phosphate
c.Calcium hydroxide 
d.Gic
28. pt present with acute apical abscess he is allergic to ester and amide local anesthesia . 
extremely need to use local anesthesia ,GA can’t be used .what local anesthesia given
a.nitrous oxide
b.benzocaine hydrate
c.1% hydrate
d.Diphynhydramine***
29. 50 yr old female have paroxysmal unilateral short pain which aggravates on touching
a.Atypical odontolgia
b.Atypical neuralgia
c.Trigeminal neuralgia**
30. Female patient with multiple erosions on palatal surface of her anterior teeth , common cause
a.Peptic ulcer
b.Alcohol consume
c.Anorexia nervosa**
31. the test not used primarly in testing pulp vitality
a. cold
b. hot
c. percussion
d. cavity test**
32. Long case with Oral mucosal pigmentation , adrenal insufficiency 
a.SCC 
b.Addisons disease **
c.Neurofibromatosis 
d.Peutz Jegher's syndrome
33. Shape of occlusal rest
a.box with sharp angle.
b.spoon shape.
c.concave **
d.convex
34. MTA superior over other sealer because 
a.more biocompatible
b.high tensile strength
c.high compressive strength
d.superior sealing properties***
35. Large resorption in the middle third of root after ortho.
a.Extract and implant
b.Extract and replant
c.CaOH at the site of resorption**
d.Single visit RCT
36. time for established gingivitis
a.1­2days
b.2­3 days
c.5­7 days
d.14­21 days***
37. Best radiograph for implant
a.Peri apical 
b.Panorama 
C.CT***
d.MRI
38. Patient came to you complain from discoloration in proximal of upper first premolar the clinical 
examination show tooth is intact and also in radiograph no­cavitation what your decision:
a.No treatment 
b.Fluoride application for comprised hydroxyapatite *
c.Composite restoration
d.Amalgam restoration
39. Pt. come with deep carious lesion ,radiographic examination show well­circumscribed radio­
opaque mass in 16 With radiolucent boundary , tooth border is visible and no other symptom
a. odontogenic tumor
b.condensing ostitis *
c.benign cementoblastoma
40.Maximum time for avulsed tooth to be brought to dentist is :
a.1hr*
b.24 hrs 
c.1week
d.8 to 9 hrs
41. caries progression in adults less rapid than in childen due to
a.Difference in ph
b.generelized dentin sclerosis by age*
c.increse in organic content of tubular dentin
42. Maxillary Tuberosity block 
a.Posterior superior alveolar nerve block*
b.Anterior superior alveolar nerve block 
c.Middle superior alveolar nerve block 
d.Maxillary nerve block
43. Child had trauma in the upper central and become intruded with loss of superficial layer of 
epithelium this is
a.Concussion and subluxation 
b.Laceration and subluxation
c.Laceration and luxation
44. child had trauma, the upper four anterior teeth are displaced
a.fracture of alveolar bone*
b.fracture of roots
c.luxation
45. Disadvantage of full thickness mucoperiosteal flap
a.Delayed secondary healing
b.Scar tissue formation
c.interdental papilla integrity*
46. Long case with Bulbous molars, obliterated pulp, short roots with Periapical pathosis
a.Amelogenesis imperfecta
b.Dentinogenesis imperfecta*
c.Regional odonto dysplasia
d.Dentin dysplasia
47. Cast post Indication :
a.Flared canal**
b.straight canal
c.wide canal
d.narrow canal
48. Patient with high masticatory force and need esthetic restoration in posterior area:
a.composite with no bevel**
b.composite with bevel
c.glass ionomer
49. Disadvantage of apex locator
a. Pregnancy 
b. short canal
c. curved canal
d. open apex*
50. H file
a.negative rake angle
b.More positive rake angle than K file*
51. While u are removing impression from a cast u need to be careful not to tear it which impression 
material
a.Polyether
b.condensation silicone*^*
c.Addition silicone
d.Polysulfide
52. Where are the guiding planes located in a partially edentulous prosthesis.
a.Cervical
b.Occlusal
c.Distal*
d.Lingual
53. to provide maximum strength of amalgam , cavosurface angles should be
A. Approach 75 with outer surface
B. Approach 90
C. Be supported by sound dentine
D. Be located in area free of occlusal stress
B+C+D*
54. instrument used to remove sharp bone after extraction 
a. rongeur
b. osteotome
c. bone file*
d. None
55. Patient not having secondary caries even after dislodgement of amalgam restoration ,underneath 
which GIC was given as a base due to :
a.sudden uptake of flouride 
b.enamel uptake of flouride*
56. Long case abt
Pt has 10cm ulcer in the palate for 3 weeks ,fibropurulent layer over it
a.minor aphtus 
b.major aphtus .*
c.erythema multiform 
d.herpatic ulcer
57. Which of the following MIGHT cause gingival enlargement
a) Phenytoin (Dilantin) *
b) Cyclosporine
c) Nifedipine
d) Aspirin
58. Diabetic patient came to clinic with pain, swelling & enlarged mandible, on radiograph it showed 
moth eaten appearance, your diagnosis is:
a)Acute osteomyelitis.
b) Chronic suppurative osteomyelitis.*
c) Focal sclerosing osteomyelitis.
d) Diffuse sclerosing osteomyelitis.
59. water absorption of restorative material cause
a.Decrease the volume of restoration
b.Soft resin
c.Change the color of restoration 
d.Increase the bond*
60. When you try to seat a crown on tooth you find a discrepancy of 
0.3mm at the cervical margin; crown well adapted to the cast and no other problem .you will: 
a.Reduce inner surface of crown 
b.Remake crown **
c.Smooth the enamel at the margin 
d.Hand burnish crown margins
61. Intraligamentary injection will make ...?
a.don't affect pulp circulation 
b.slightly decrease pulp circulation
c.ceases pulp circulation for 30 minutes*
d.will increase pulp circulation
62. Which epithelium is lacking in alveolar epithelium. ...
a.Stratum corneum..*
b.Stratum granulosum
c.Stratum spinosum
d.Stratum basale
63. Inferior alveolar nerve block not effective . So which of the nerve is anesthetized?
a.mylohyoid nerve*
b.lingual nerve.
c.mental nerve.
d.buccal nerve
64. Composite restn ,class 2, interproximal space after filling
a.Segmental matrix and wedge**
b.Circumferential matrix and wedge(amalgam)
c.Toflmeir matrix
d.Wedge
65. the maxillary sinus drains into 
a. Middle meatus +*
b. Inferior meatus
c. Superior meatus 
d. Spheno ethmoidal recess
66. Materiel that has substantivity
a.chlorhexidine 
b.flouride*
67. Young pt. came without any complain. During routine X­ray appears near the apex of the two 
lower molars lesion diameter about 2mm. & extend laterally and inferiorly about 5­6mm with sharply 
irrgular border...its shows radiolucency with not will defined peripheral radioopacity . What’s the type 
of cyst: 
a.Dentigerious cyst 
b.periapical cyst 
c.Simple bone cyst**
d.compound osteoma
68. Fractured tooth to alveolar crest, what's the best way to produce ferrule effect:
a.Restore with amalgam core sub­gingivally.
b.Crown lengthening.
c.Extrusion with orthodontics.**
69. Material for Chair Side relining of denture?
a.Soft liner
b.Light cure acrylic resin..*
c.Wax
d.Acrylic
70. If a deep incision made medial to lower eyelid while performing..what structure most commonly to 
be injured?
a.lacrimal gland
b.lacrimal canaliculi.**
c.infraorbital nerve
d.inferior lateral nasal cartilage
71. case about pain on touching the root surface­­­­dentinal hypersensitivity
72. For a child of age 10, what all teeth are present
73. Case about contact stomatitis
74. Child 10yrs came to dental clinic. Behavior explained. What's the mental age?­­­­3yrs
75. Long case, LA maximum dose , 80kg patient. ­­­­560mg
76. Long case abt white discoloration on teeth surface, sound enamel
a.active caries***
b.inactive caries
77. After periodontal surgery which pack is given
a.eugenol containing 
b.non­eugenol*
78. Steps in resistance and retention form
79. 14 yr old patient, which x­ray to see unerupted teeth­­­­­panoramic radiograph
80. Long case ,a mother complains of her 4 month child with no teeth­­­­­ child is normal
81. Factor deficiency causes­­­­hemophilia
83. Long case , pain and swelling on having food­­­­­­Sialolithiasis
84. Water in handpiece­­­­­dentinal heating
85. Ultrasonic cleaners­­­­reduce bacterial load
86. What is needle hub?yes the part of needle that attaches with the syringe
end
87. Cast post preferred in
a.good hygiene 
b.circle root canal cross section**

Exam 36

Vous aimerez peut-être aussi